סיקור מקיף

הצוות ה-35 הגיע לתחנת החלל בשש שעות במקום ביומיים כמקובל עד כה

החללית סויוז TMA-08M שנשאה שלושה חברים חדשים לצוות ה-35 של תחנת החלל הבינלאומית עגנו בסוף השבוע בתחנה לאחר מסע מושלם במהלכו הגיעו לתחנה בפחות משש שעות במקום ביומיים כמו כל טיסות הסויוז והמעבורות עד כה.

החללית סויוז TMA-8M מתקרבת לתחנת החלל, 28 במרץ 2013. צילום: NASA TV
החללית סויוז TMA-8M מתקרבת לתחנת החלל, 28 במרץ 2013. צילום: NASA TV

מפקד הסויוז פאבל וינוגרדוב, ועמיתיו אלכסנדר מיסורקין וכריס קאסידי, שוגרו מבייקונור ביום שישי בבוקר והגיעו באותו היום לתחנה. הסויוז הגיעה לתחנת החלל לאחר ארבעה מסלולים בלבד במקום יומיים אחרי השיגור המסורתיים.

המסלול החדש נוסה לראשונה בטיסה מאויישת. קודם לכן נעשה בו שימוש בשלוש טיסות המטען האחרונות של חלליות מדגם פרוגרס.

השלושה הצטרפו לכריס הדפילד הקנדי, טום משבורן מנאס”א ורומן רומננקו מרוסיה השוהים במעבדה מאז ה-21 בדצמבר 2012. כל ששת חברי הצוות השתתפו בטקס קבלת הפנים בשיתוף בכירי הסוכנויות ובני משפחתם של האסטרונאוטים שהתאספו במרכז הבקרה הרוסי בעיר הכוכבים שליד מוסקבה.

הצוות ה-35 יפעל בהרכב מלא של שישה אנשים עד מאי כאשר הדפילד משבורן ורומננקו ישובו לכדור הארץ בחללית סויוז TMA-07M. עזיבתם תהפוך את השלושה שהגיעו היום לחברי הצוות ה-36 תחת פיקודו של וינוגרדוב שישארו בהרכב מצומצם עד שיגיעו חברי הצוות הבא בסוף מאי. קסידי, וינוגרדוב ומיסורקין צפויים לשוב לכדור הארץ בספטמבר. הם יבצעו 137 ניסויים בחלק האמריקני של התחנה ו-44 בחלק הרוסי. 430 חוקרים מכל העולם יהיו מעורבים במחקרים הללו שיעסקו במחקרי התנהגות אנושית, מחקרים בתחום הביולוגיה והמדעים הפיסיקליים, פיתוח טכנולוגיות, תצפית על כדור הארץ וחינוך.

קאסידי, קצין בצי האמריקני טס לחלל בפעם השניה. בפעם הקודמת הוא ביקר בתחנה במעבורת אנדוור בטיסה STS-127 ביולי 2009. אז גם בצע שלוש הליכות חלל שהסתמו ביחד ב-18 שעות מחוץ לתחנה. עבור וינוגרדוב זו המשימה השלישית. הוא היה חבר צוות מיר למשך 197 יום בשנים 1997-8 ובילה 182 יום בתחנת החלל הבינלאומית בשנת 2006 כמהנדס הטיסה של הצוות ה-13.

למיסרוקין, סא”ל בחיל האוויר הרוסי זו טיסת החלל הראשונה. הוא הוכשר כקוסמונאוט ניסוי ב-2009.

334 תגובות

  1. אז בוא נניח שבדרך חזרה, בפרק זמן של חצי שעה על פי שעונו, הטמפרטורה נופלת ב300 מעלות, 10 מעלות בדקה, והכל מצולם בוידאו.

    אבל שים לב, באותה חצי שעה הוא בעצם במנוחה. יתרה מזאת, אילו היה זוג תאומים נוסף שחונה צמוד אליו, והם עדיין לא התחילו את התהליך, הרי שמבחינתם הם במנוחה ולכן הזמן מתקתק בקצבו הרגיל והטמפרטורה בקושי משתנה.

    מצב פרדוקסלי, לא?

  2. ישראל שפירא
    לא סתם שאלתי בקשר לשיטת מדידת החום. חשבתי ששתבין – לא משנה.
    אכן ימצא את היקום קר יותר.
    אני אזניח את זמן ההאצות.. זמן זה יכול להיות קצר ביותר או ארוך ביותר, ובשני המיקרים זה זניח.
    במהלך התנועה, אני באמת לא יודע מה יקרה…..

  3. אנא ערף, רוב האנשים שאני מכיר מצליחים למדוד את הטמפרטורה עם מדחום או מד קרינה. רק מעטים משתמשים בברומטר או מד לחות.

    אז הנה השאלה, עם תאומים כמו שאתה אוהב:

    אם התאום הצעיר, שנפרד מאחיו כאשר היקום היה בן 400,000 שנים, שהה בחלל שעה ובינתיים אחיו הזדקן ב10 מילארד שנים (אילו תאומים זריזים ומאריכי חיים במיוחד).

    האם כאשר ישוב הצעיר ממסעו בן השעה, הוא לא ימצא את היקום קר בהרבה מאשר כשעזב אותו?

    אם הוא יצלם בוידאו את שעונו ומדחומו, מתי יראה הוידאו את ירידת הטמפרטורה המהירה ביותר:

    בתקופת ההאצות?

    בתקופת המסע מן התאום הנשאר?

    בתקופת המסע חזרה לתאום הנשאר?

  4. ישראל שפירא
    איך המדחום שלך מודד טמפרטורה? אם אתה מתכוון למהירות הממוצעת של מולקולות זה דבר אחד. אם אתה מתכוון לגיל היקום זה דבר אחר.

  5. ניסים

    הדבר היחיד שאני שואל הוא על הטמפרטורה. אינני שואל על פרדוקס התאומים. רק זאת: כאשר ג׳יל חולפת על פני ג׳ק, מה יראו המדחומים שלה ושלו?

  6. ישראל שפירא
    רציתי שתראה שהתאוצה לא ממש משפיעה על הזמן. עניין של שסדרי גודל.
    מה שאני חושב זה ככה: כשהתאום חוזר לכדור הארץ, למרות גילו הצעיר, הוא יסכים שהשעון של כדור הארץ הוא נכון. הוא יודע שהוא שינה מערכת יחוס פעמיים ולכן יחשוב שהוא חזר לעתיד.
    מה שמרתק הוא שכבר עניתי לך על זה…… 🙂

    אבל – האם אתה שואל מה יקרה בזמן התנועה? זו שאלה אחרת ובאמת לא ברורה (לי) מאליו. אצטרך לחשוב על זה….

  7. לא יודע איזה חישוב אתה רוצה שאעשה. האם את החישוב שמסביר את פרדוקס התאומים במסגרת היחסות הפרטית? הבאת את זה קודם והסכמתי איתך. אז מה? מה הקשר לצריכת אנרגיה ושות?

    אם אינך מאיץ, אתה עומד, ולכן אינך יכול להחליף מערכות יחוס.

    אבל זה כבר שייך להסטוריה. שאלתי אותך שאלה שלא ענית עליה: בדוגמת ג’-ג’: האם כאשר ג’יל חולפת על פני ג’ק שניהם מודדים את אותה הטמפרטורה לחלל החיצון?

    אולי יתמזל מזלי ותענה הפעם 🙂

  8. ישראל שפירא
    התאוצה היא דרך לשנות מערכת יחוס. כדי לספק תאוצה צריך כוח. כדי לייצר כוח צריך לצרוך אנרגיה. האם צריכת האנרגיה היא ההסבר לפרדוקס?
    אני רואה שלא עשית כל חישוב. ואני שואל את עצמי – למה? אתה בטח לא רוצה שאענה לעצמי 🙂

  9. ישראל שפירא
    למה להעליב? 🙂
    כמה פעמים אפשר להסביר שפרדוקס התאומים לא קשור לתאוצות? אולי תעשה את החישובים במקום להתווכח סתם? תניח מרחק מסויים ותניח תאוצה קבועה (חיובית ואחרי זה שלילית). הגדלת התאוצה מקטינה את זמן התאוצה ולכן, בקירוב ראשון, לגודל ההשפעה אין השפעה. מצד שני – הכפלת המרחק מכפילה את הפיגור בזמן.

    הנה ציטוט מויקיפדיה – The standard textbook approach treats the twin paradox as a straightforward application of special relativity

    וכמו שאמרתי …. אין כל סתירה בין המפץ הגדול לתורת היחסות. להיפך – איינשטיין הרי ניבא את תורת המפץ הגדול.

    ישראל, אם אתה רוצה שיחה מועילה – תעשה בבקשה כמה חישובים פשוטים. וקרא לפחות בויקי 🙂

  10. יובל

    אבא כמובן צודק, אבל מה הטעם בלצדוק אם אינך מסוגל להצדיק את צדקתך?

    העלתי את השאלה פה ובמקומות נוספים מספר פעמים. כולם מסבירים לי במבינות שאני פשוט טועה, אבל כל אחד טוען שהסיבה היא שונה. עד כה לא קיבלתי הסבר קוהרנטי ועקבי מאף אחד חוץ מאנונימי, שטוען בקוהרנטיות ועקביות שההסבר הוא שאני פשוט טמבל, מה שמובן מאליו גם בלעדיו.

    קח לדוגמא את ההסבר האחרון של ניסים הנעלם: “שניהם מסכימים שג’יל שינתה מערכת יחוס וחזרה למערכת היחוס המקורית”.

    טעות. ג’יל לא שינתה מערכת יחוס. בניגוד לפרדוקס התאומים, ג’יל אינה מאיצה 3 פעמים, למעשה היא אינה מאיצה בכלל. במשך כל ההרפתקה היא מבחינתה עומדת במקום.

    הסבר שני המקובל מאוד עיי רבים וטובים: כל צד רואה את שעוני השני כנעים לאט יותר.

    גם כן טעות במקרה שלנו, וטעות זאת מוסברת בדוגמאת ג’-ג’.

    הסבר שלישי: שעוניה של ג’יל מפגרים יחסית לשעוני ג’ק כיוון שהוא במנוחה יחסית לCMBR והיא בתנועה יחסית אליה.

    הCMBR לא הוזכרה בדוגמא שלנו. היא אמורה לעבוד גם כאשר המצב הוא הפוך, דהיינו כאשר ג’ק נמצא בתנועה יחסית לקרינה וג’יל במנוחה.

    הסבר רביעי: טמפרטורת היקום היא גורם שמחוץ למערכת.

    אילו היה הדבר נכון, לא היינו מקבלים התאמה מושלמת בין זמן 0 הקוסמולוגי לזמן 0 המחושב על פי נוסחאת פרידמן.

    ההסבר הראשוני הפשוט אותו אתה מחפש של איך ומדוע מעסיק גם אותי, וניסיתי להביא הסבר משלי בכתבה הארוכה בקוסמו. אין זאת אומרת שהוא נכון, למרות שהוא מסביר יפה גם תופעות נוספות.

    לדעתי קיים הסבר בתוך המיינסטרים ואני פשוט לא מכיר או מבין אותו. לכן אני ממשיך להעלות את הנושא בתקווה שאולי מישהו שמתמצא בנושא יוכל להסביר. אולי אבא?

  11. ישראל – בהתנצלות על העיכוב בתגובה.

    כאמור, אינני שותף לדעתו של אבי. נראה לי שאתה מבין על בוריה את היחסות. ברור גם שנהירים לך מודלים פיסיקלים נוספים. אתה תוהה על הסתירות המתגלעות בין המודלים השונים, ובצדק. הם אכן אינם מושלמים, ועל זה כבר הנחתי את האצבע לפני הרבה זמן.

    לא נזהרתי בלשוני בביקורת על דרך עבודתך. אמנם היא לא מסודרת לטעמי, אך אולי היא עדיפה על שיטות העבודה שלי. כל עוד אף אחד מאיתנו לא הגיע אל האור, אל לנו לשפוט.

    אולי לא הבנת את השימוש שלי במילה “מתפיסיקה”. לא התכוונתי להעלאת רוחות באוב אלא למציאת מודל פשוט המסביר את כל התופעות הפיסיקליות הידועות היום ומיישב את הסתירות בין המודלים השונים. את הדיון בפיסיקה “פרופר” כבר מיצינו וכעת אנו רק דשים בזה שוב ושוב.

  12. ניסים

    האם כאשר ג’יל חולפת על פני ג’ק שניהם מודדים את אותה טמפרטורה לחלל החיצון, או שכל אחד מהם מודד טמפרטורה שונה?

  13. ישראל שפירא
    ממש לא. גם ג’יל וגם ג’ק מסכימים ששעונה של ג’יל מפגר. בסיום שניהם מסכימים שג’יל צעירה יותר. כלומר – ג’יל נוחת בעתיד. המקרה הזה הוא בדיוק פרדוקס התאומים. שניהם מסכימים שג’יל שינתה מערכת יחוס וחזרה למערכת היחוס המקורית.

  14. ניסים

    בפרדוקס המקורי כל תאום יכול להגיד שהזמן שלו הוא הנכון.

    תוכל לראות את זאת טוב יותר בדוגמת החלליות הקודמת. כזכור, גם ג׳ק וגם ג׳יל מסכימים ששעונה מפגר. האם פרוש הדבר שאילו בזמן 0 המשותף לשניהם שניהם באותו הגיל, אז בזמן המפגש ג׳יל צעירה מג׳ק כמו בפרדוקס התאומים?

  15. ישראל שפירא
    אני מבין את מה שאתה אומר. התופעה של פרדוקס התאומים אכן קיימת – על כך אין ויכוח. זה במיקרה של הבדלים קטנים בזמן שאין להם השפעה על גיל היקום הנמדד.
    שאלה טובה היא מה יקרה בתנאים קיצוניים יותר, נניח הפרש של מאות מילוני שנה, שזה משהו שמדיד. אבל – אני לא רואה בעיה. התאום חוזר בעתיד הרחוק ויגלה יקום שהזדקן בהרבה – בדיוק כמו שאחיו הזדקן. לכן – שניהם יסכימו על גיל היקום.
    אין כאן, להבנתי כל סתירה.

  16. ניסים

    בפרדוקס התאומים בזמן המפגש החוזר ביניהם, שעונו של כל אחד מהם מראה זמן שונה, וזאת למרות שכאשר הם נפרדו השעונים הראו את אותו הזמן.

    גילו של היקום לעומת זאת, הוא אחד ויחיד, גם כשנפרדו וגם כשנפגשו.

    אז איך אפשר להגיד שלכל אחד יש את הזמן שלו, ושזמן זה לגיטימי בדיוק כמו של אחיו?

    לפני כמה ימים חדתי חידה. מהותה: האם ניתן לסנכרן שעונים בחלל ללא קשר בין המסנכרנים.

    אילו היית עונה עליה, היית רואה את ההבדל בין הידע של איינשטיין ב1905 כאשר הגה את היחסות, והידע כיום.

    יובל.

    לא ראיתי שום תגובה של אבי (סתם) אבל הוא כמובן צודק: אינני מבין את היחסות כנראה.
    אז אולי אבא יסביר את הסתירה כביכול בין הזמן המוחלט של המפץ והזמן היחסי של היחסות?

    בעניין דרך עבודה מסודרת – אתה צודק יותר ממה שאתה יודע (איזו עבודה אגב?). אבל כן, הראה לי איפה דרך עבודתי אינה מסודרת, למען אטיב דרכי בעתיד.

    מתפיסיקה כבודה במקומה מונח, אך אני מתעניין יותר בפיסיקה פרופר. מה גם שלא נראה לי שהמתפיסיקה תפתור את דילמתינו. אבל אם יש לך משהו מעניין שמתקשר לנושא, שוט.

  17. ישראל, אני מעריך את הענוה שאתה מגלה בסיפא של דבריך: “קיימת תשובה לסתירה כביכול זו, ואני פשוט לא מכיר או מבין אותה”. אבי ציין כי “קרוב לוודאי שאינך מבין את היחסות הפרטית/כללית”. אמנם אינני שותף להערכתו, וזאת כי נוכחתי לדעת שאתה מגלה בקיאות (אם לא מלאה, אז לפחות חלקית) ברזי המיסתורין של התאוריות דנן, אך נראה לי שחסרה לך דרך עבודה מסודרת. יתכן שאינני רואה את הדברים נכוחה, ואהיה מאושר אם תוכיח לי ההיפך. על כל פנים, כדי לבדוק ולהשוות כמה תאוריות (בעיקר כאלה הנראות כסותרות זו את זו) יש, לדעתי, להיכנס אל תוך המתפיסיקה שלהן. עלינו להבין את המכניזם היוצר את התופעות שהתאוריות האלה מייצגות.
    בתחילת דברי כאן אמרתי שנראה לי שחסרה לך דרך עבודה מסודרת. אם תרצה, אביא דוגמאות. אך אין לי ספק שהשאלות שאתה מעלה הינן רלוונטיות וכבדות משקל. לכן, אם יש במה שאמרתי כדי להוציא את הרוח ממפרשיך, אנא התעלם.
    שבוע טוב 🙂

  18. ישראל שפירא
    המפץ הגדול הינו ניבוי של תורת היחסות. אני לא רואה ביניהם סתירה. ולא הבנתי מה הקשר לפרדוקס התאומים – הרי אנחנו מסכימים שאין כאן שום פרדוקס.

  19. קל ופשוט – אך לא מביא לסתירה עם היחסות או המפץ.

    הרי היחסות טוענת שלכל אחד יש את השעון שלו ואת הקצב שלו. אז למה שלא נסנכרן שעונים על פי שעון בעל קצב טקטוק מהיר או איטי, או אחד שנקודת ה0 שלו היא אתמול בצהריים?

    הצרה היא שבשעונים ממהרים או מפגרים אורך היממה אינו 24 שעות ובשעונים שזמן ה0 שלהם שונה מן הזמן הקוסמולוגי, יקומינו זקן או צעיר מגילו האמיתי.

    ניתן לראות זאת יפה בפרדוקס התאומים. אם התאום הצעיר ממש ממהר, אז כאשר הוא נפגש עם אחיו מאריך הימים, הלז הזדקן במיליארד שנים. אולם כאשר שניהם משווים את שעוניהם, לא יוכל הצעיר לטעון כי הזמן שלו לגיטימי כמו זה של אחיו כפי שטוען איינשטיין. עובדה: היקום כעת קר יותר, ולא בשבריר המעלה אלא בפער ניכר.

    שונה הדבר אצל איינשטיין. מכיוון שאין תחילה וסוף לזמן, הרי כל תאום יכול לטעון שהזמן שלו לגיטימי בדיוק כמו זה של אחיו.

    האם זה אומר שהיחסות שגויה? לא. רק שהיא אינה מסתדרת עם תאוריית המפץ, או מה שסביר יותר, שקיימת תשובה לסתירה כביכול זו, ואני פשוט לא מכיר או מבין אותה.

     

  20. אתה מתפרץ לדרך פתוחה. ברור שקיימת בעיה עם הנחות היסוד. לא היינו צריכים את כל הטרטור המרתוני הזה כדי להיווכח בכך. על אנרגיה אפלה, למשל, שמעת?
    בין שנינו אני העצלן ואתה החרוץ. לסנכרן את השעונים בדרכים שהצעתי קל ופשוט יותר מאשר בדרך שלך.

  21. לא צריך מאפנן. יש היום ספריי מיוחד נגד נודניקים, אבל כל הכיף זה לזרוק עליהם נפטלין נפטלין.

    חוץ מזה שאני רואה שאבי כבר מחק את הודעות ההתחזות והשטנה (תודה אבי).

    גם לא צריך CMBR. היא הובאה רק כדוגמא לזאת שכל השעונים יכולים להסתנכרן לזמן מסויים וטבעי (רגע המפץ) אם רק ירצו. אבל הרבה יותר פשוט להשתמש בשעונים אוניברסליים, המורכבים ממשדר מרכזי המשדר ליקום את הזמן שלו (עדיף כמובן את רגע המפץ, כדי שנקודת ה0 לא תהיה שרירותית), וכל שעון מורכב ממקלט, מחשב ומד דופלר. תמיד תוכל לסנכן שעונים בדרך זו כמו שצפרדעים בביצה יכולות לסנכרן שעונים עיי מדידת טמפרטורת הביצה המתקררת לאיטה בלילה.

    אם אתה מקבל את זה, תראה שקיימת בעיה עם הנחות היסוד של היחסות.

  22. ותיקון קטון: CMBR זוהי קרינת המיקרוגל הקוסמית שברקע, ואשר לה אין מוקד ידוע, אין לייחס לה שום מהירות תנועה יחסית לשום גוף והיא מייצגת נאמנה (עד כדי פרידמן) את גיל היקום שנולד במפצהגדול. אפשר להשתמש בה לסנכרון שעונים באופן שהצעתי (קליטת אות המכיל זמן שרירותי והטמפרטורה הלוקלית באתר השידור והשוואתו עם הטמפרטורה המקומית של החללית הקולטת).

  23. אז זהו בדיוק! קרינת הרקע הקוסמית היא זו שאליה התכוונתי כשהצעתי להשתמש בה לסנכרן שעונים. תחילתה בסדרה בה מככבים שלדון ולאונרד 🙂 והיא דועכת הומוגנית בערך באותו קצב בה דועכת הפופולריות של האחרונה – שלא בצדק 🙁

  24. יובל.
    מעט מאוד ניתן לעשות כנגד פחדן טיפש שלא מזדהה ומשנה את שמו כל הזמן, מפריח הבלויות וניבולי פה ואינו מוכן להעמיד שום נושא לדיון אובייקטיבי.

    אך כנראה שמדיניות אתרינו היא להרשות לטפילים שכאלו להציק למגיבים לגיטימיים. יש את אילו שלא מתייחסים ונותנים לנודניקים האלו (טוב, כולנו יודעים במי מדובר) להציק להם ללא מענה. אני מעדיף להשתמש בהם בתור שק חבטות לתרגול. גם הוא סך הכל מעניק כושר לא רע.

    אל תדאג, שנתיים שאני כבר כותב פה ועדיין לא התגוללתי אפילו על מגיב אחד אלא כתגובה. לא נראה לי שאשתנה.

    לעניינינו. יש שני סוגי קרינה – הקרינה הקוסמית, וקרינת הרקע הקוסמית. עסקינינו באחרונה.

    אם תסתכל בלינק:

    http://en.wikipedia.org/wiki/Sun

    תמצא שם:

    the Sun’s resultant velocity with respect to the CMB is about 370 km/s in the direction of Crater or Leo.

    וכפי שציינתי, הצורה שמודדים מהירות זו בחלל היא עיי השוואת ההסטה לכחול ואדום בכיוונים שונים.

    קרינת הרקע הקוסמית היא במידה רבה מערכת היחוס האבסולוטית אותה אתה מחפש (שששש, לא אמרתי אתר!). אולם שים לב שלעניינינו, זמן מוחלט, היא איננה תורמת. אילולא היא הרי שלא היתה כל בעיה פשוט למדוד את טמפרטורת החלל, ולהפוך אותה לזמן מוחלט באמצעות נוסחת פרידמן. בגללה עלינו לשקלל את הדופלר, אך עדיין נגיע ללא בעיה לזמן מוחלט.

  25. ישראל, שבת שלום.
    ראשית, אנא אל תתייחס למנבלי הפה ושוג הכתיב; אני חושש שמא תהפוך להיות אחד מהם.
    שנית, נדמה לי שאתה מערבב מושגים. עד כמה שזכור לי משיעורי האסטרופיסיקה באוניברסיטה, מה שמגיע אלינו מכיוון רגולוס (שנמצא במזל אריה – “קונסטלציית לאו”) זו לא קרינת הרקע הקוסמית אלא זרימה של חלקיקים המהווה סימן לכך שמערכת השמש שלנו נעה לעבר הכיוון ההוא. קרינת הרקע הקוסמית היא קרינה אלקטרומגנטית בתדר מיקרוגל המגיעה אלינו מכל הכיוונים בטמפרטורה (כמעט) שוה.

  26. לא מספיק שאתה בור קשקשן ופחדן, אז נוסף לכל אתה גם עצלן.

    הרי טענת שחידת מכוניות השעטנז שגויה. מה קרה, עוד הפעם השתפנת?

    אני רואה שאבי בליזובסקי מחק תגובה שלך שבה הגזמת בניבולי הפה שלך. נקווה שהוא ישים קצת יותר לב לכל המתחזים והטפילים שזוחלים לאתר שלו מכל הפינות ומציקים למגיבים הלגיטימיים, לפני שיהיה מאוחר מדי וכולם פה ידבקו במגיפת הטרוליות.

    יאללה, תתחזה עוד פעם, תקלל עוד פעם, תגיד את המילה האחרונה ותתעופף.

  27. צ׳אנס אחרון, נודניק.

    או שתסביר מה לא בסדר בחידת מכוניות השעטנז – או שאאלץ לסיים את הדיון איתך לתמיד.

  28. עוד הפעם ליצן, עוד הפעם שפירע, עוד הפעם מפלצת הספגטי המעופפת, עוד הפעם נחיתה רכה…

    מתי כבר תחדש משהו, נודניק. לא הגיע אולי הזמן להתקשר עם הבן דוד הצעיר והדביל שזכה בכל הפרסים?

  29. “אז או שתתחיל לדבר לעניין או….” – הנה עוד בדיחה מבית היוצר של הליצן שפירע.
    אתה יודע, אם תשאל את מפלצת הספגטי המעופפת יהיה לך קל יותר לסנכרן את השעון של החללית שלך עם הזמן של כדור הארץ.
    נחיתה רכה

  30. בטח שאני יותר חכם מאיינשטיין, אתה צריך לדעת. אתה הרי יותר חכם מכולנו.

    הייתי ממשיך את הויכוח הנואל איתך הזה ברצון, אבל סבא עליו השלום לפני שנפטר השביע אותי לא לנהל דיונים עם נודניקים אנונימיים (עאלק) שמכנים את עצמם בשמות כמו שמעון שקשוקה, פלאפל ודנג’רוס.

    אז או שתתחיל לדבר לעניין, או שתתחפף. אחרי הכל, סבא…

  31. בטח שעשיתי. כל יום אני מבצע שניים שלושה ניסויים. לפעמים גם ארבעה.
    מה הפואנטה? איינשטיין הגה את כל היחסות מבלי לצאת מהמשרד. ניסויים מחשבתיים, עאלק. האם לכן אנו יכולים להפסיק לחשוש מהפצצה האיראנית?

    רוץ למשביר דנג’י, שמעתי שמוכרים היום דמיון במחירי מבצע.

  32. ישראל
    עשית ניסוי?
    איך בדיוק מודדים קרינת רקע קוסמית?
    גם השלג בטלויזיה זאת הקרינה הקןסמית. איך תמדוד? צריך כלים כמו טלסקופ למשל שיודע להפריד רזולוציות תדרים.. איך תעשה זאת?
    אתה מתכוון לשגר לחלל טלסקופ?

  33. לקרינת הרקע הקוסמית יש מהירות וכיוון (כ300 קמ/שנ לכיוון קונסטלציית ליאו יחסית לכדה”א). כאשר מודדים אותה באותו כיוון, מקבלים שבגלל המהירות יש הסטה לאדום או כחול, תלוי אם אתה מודד עם או נגד כיוון הקרינה. כך אתה יכול לדעת מהי מהירותה, ומכאן את מהירותך יחסית אליה, אך עליך למדוד בשני הכיוונים.

    אצל השעון האוניברסלי שהצענו האות האלקטרומגנטי מגיע רק מכיוון אחד, מקומו של המשדר. לכן הדופלר הוא חד כיווני.

  34. אין מניחין בתגובה אחת יותר מקישור אחד שמא תנפול חסימה.
    יכול מגיב לעשות “הבה נתחכמה לו” ויתן קישור נוסף תחת שם עצמו.

  35. יופי, עברה. ננסה שנית:

    למה לסבך? אין צורך בתקשורת בין א’ וב’. פשוט הולכים ללינק:

    http://hyperphysics.phy-astr.gsu.edu/hbase/astro/expand.html#c3

    זוהי נוסחת פרידמן לקשר שבין טמפרטורה לזמן שחלף מהמפץ הגדול.

    עכשיו לך ללינק:

    http://www.fourmilab.ch/etexts/einstein/specrel/www/

    המאמר המקורי של איינשטיין על היחסות,

    ותראה שהפרק הראשון שבו מוקדש לסינכרון שעונים.

    זו בעיה לגיטימית ב1905 כאשר האמונה היא שהזמן נמשך ממינוס אינסוף לאינסוף, והדרך היחידה היא להחליט על זמן שרירותי ולהסתנכרן. היום המצב שונה, אחרת לא היינו יכולים לדבר על יקום בן 13.7 מיליארד שנה.

    את הדרך שהצעת הצענו כבר בעבר וקראנו לה “שעוני יקום”. אולם שעונים אוניברסלים מטקטקים באותו הקצב של שעוני טמפרטורה, אך נקודת ה0 שלהם היא שרירותית, בעוד שנקודת ה0 של שעוני הטמפרטורה היא טבעית, המפץ, שלפניו פשוט לא היה זמן. היתרון של שעונים אוניברסלים הוא בפשטות הפעולה, ודופלר חד כיווני.

    בעתיד, אם ייושמו שעוני היקום, יוכל כל נוסע חללי לדעת את הזמן המוחלט כפי שאתה יכול היום לדעת את זמן ניוארק עיי הצצה בסלולרי.

  36. למה לסבך? אין צורך בכלל לתקשר בין א’ וב’. פשוט הולכים ללינק:

    http://hyperphysics.phy-astr.gsu.edu/hbase/astro/expand.html#c3

    זוהי נוסחת פרידמן של הקשר בין הטמפרטורה לזמן שחלף מאז המפץ הגדול.

    יש למטה מחשבון קטן, שאם תזין בו את הטמפרטורה, תקבל את הזמן שחלף מאז המפץ.

    לכן אין כל בעיה לסנכרן שעונים.

    אם תסתכל בלינק:

    http://www.fourmilab.ch/etexts/einstein/specrel/www/

    המאמר המקורי של איינשטיין על היחסות,

    תראה שהפרק הראשון בו מוקדש לנושא הסינכרון. אולם ב1905 לא ידעו על טמפרטורת הקרינה הקוסמית ועל המפץ, וחשבו שהזמן נמשך ממינוס אינסוף לאינסוף. לכן לא היתה משמעות למושג הזמן המוחלט, ורק לזמן מוסכם.

    את הדרך שהצעת הצענו כבר קודם וכינינו אותה “שעונים אוניברסליים”. אול אין הבדל מבחינת קצב הטיקטוק של שעון אוניברסלי שכזה ושעון טמפרטורה, ונקודת ה0 שלו היא שרירותית, בשעה שנקודת ה0 של שעון הטמפרטורה היא טבעית, כי לפני המפץ פשוט לא היה זמן.

    היתרון של שעון אוניברסלי היא שהדופלר שלו הוא חד כיווני. אילו ירצו בעתיד לבנות מערכת שכזו, אז כל תייר בחלל יוכל לדעת את זמן כדה”א עיי מדידה פשוטה, כמו שאתה יודע מה השעה עיי זאת שתציץ בטלפון הנייד שלך.

  37. אז הגענו למסקנה שהיום ישנה דרך לסנכרן שעונים, וזאת באופן הבא: א שולח אות המכיל זמן שרירותי (נגיד 0) וקריאת טמפרטורה; ב מקבל את האות, מוצא את ההפרש בין הטמפרטורה ששידר א לבין הטמפרטורה המקומית שלו ומוסיף את ההפרש לזמן השרירותי של א.
    הידע הזה לא היה קיים בשנת 1905, אך לא חסרו כלים אחרים. למשל, אם ידועה עוצמת השידור של קרן האור (ועליה אפשר להסכים מראש), ניתן לחשב על פיה ועל פי עוצמת האור הנקלט את המרחק שהקרן עברה בדרך מן המשדר אל המקלט ומכאן גם את הזמן שחלף.
    מה דעתך?

  38. משתמש אנונימי (לא מזוהה)
    הוא צודק בתגובה האחרונה שלו. הוא לא צודק בזה שהוא טוען שאי אפשר לפתור את פרדוקס התאומים רק בהנחות תורת היחסות הפרטית.

  39. ניסים: אם. כאן הואצודק בכל מילה, אז למה ניסית כל הזמן לשכנע אותו שהוא טועה? בכלל לא ברור

  40. אחד אחר
    אתה בהחלט צודק בכל מילה כאן. נורא חבל שאתה לא מנסה לקרוא את מה שכתבתי כבר הרבה פעמים.
    הסימטריה נשברת מהעובדה שמרחק התנועה בשתי מערכות היחוס שונה. אם במערכת היחוס של כדור הארץ והכוכב המרוחק המרחק הוא 40 דקות אור, ואם מהירות החללית היא 0.8c אז מרחק התנועה במערכת צירי החללית היא 24 דקות אור. 0.8 ברבוע זה 0.64. 1-0.64= 0.36. שורש 0.36 זה 0.6. 0.6 כפול 40 זה 24
    זה החישוב… פשוט 🙂 שני הצופים מסכימים לחישוב זה ושניהם מסכימים בשוני בזמן – בהחלט אין באמת פרדוקס.

    אגיד את זה שוב – קרא את הלינק של ישראל שפירא. מוסבר שם יפה מאוד.

  41. מצטער- לא ממש מכיר את הנוסחאות של יחסות כללית.
    אני גם לא ממש זוכר את טרנספורמציות לורנס בעל פה.
    אני כן זוכר אבל את זה –
    וכבר אמרתי כבר הרבה פעמים- כן מה שאתה אומר נשמע הגיוני- אבל לא זה לא מה שמתקבל מהנוסחאות.
    ככה זה- גם הגיוני שאם תשגר טיל במהירות חצי ממהירות האור מחללית שנוסעת במהירות חצי ממהירות האור- הוא יזוז במהירות האור- ובכל זאת זאת לא המציאות.
    תאוצה משנה כאן היא הדבר היחידי ששובר את הסימטריה. אם זה לא מה שהיה אז איך אתה היית פותר את הפרדוקס? פיסקה לא סובלת פרדוקסים- היחסות הפרטית והכללית לא היו יכולים להתקיים אם משהו כל כך בסיסי לא היה יכול להפתר.

  42. ישראל שפירא
    תכפיל את “זמן המנוחה” – מרחק המסלול יוכפל ולכן גם הפיגור בשעון יוכפל.
    תכפיל את עוצמת ההאצה – זמן ההאצה יירד לחצי ותהיה הפשעה מאוד קטנה על הפיגור.

    הסיבה לפיגור בזמן היא השינוי באורך המסלול ב-2 מערכות הצירים. תבדוק אותי….

  43. יובל.

    קיבלת שאין הסחה? עוד יותר טוב. אז אין צורך למדוד את הדופלר, וכל שני צופים החולפים זה על פני זה יסכימו תמיד על הטמפרטורה ולכן על הזמן, ולא משנה מה מהירותם היחסית, לא?

    האם תהיה להם בעיה אז לסנכרן את הקרניים בחידה שלי ולהצליח לפתוח את תיבת חיי הנצח?

    והאם יכלו לעשות זאת לפי הידע ב1905?

    ניסים.

    “הטיסה הרציפה” אליה אתה מתייחס היא בעצם חוסר תנועה מכל בחינה שהיא. מה שאחד אחר טוען (אם הבנתי נכון) הוא: אם פרקי הזמן של התאוצות מהתאום הנשאר וחזרה הוא זניח, הרי שרוב הזמן שני התאומים בילו בעצם בתנועה אינרציאלית שהיא בעצם מנוחה. אם שניהם נחים רוב הזמן, מדוע אחד מזדקן ואחיו לא? האם יש הבדל בין שתי מערכות אינרציאליות? מה עם פוסטולט א’?

    שוברת השוויון היא התאוצה, אותה חווה האח הצעיר בלבד.

    נכון הדבר שאם ג’יין תהפוך את כיווניה היא תהיה עוד יותר צעירה מג’ק, אבל כל היפוך כזה משמעותו תאוצה.

    לכן כשאתה כותב: “אבל גם מהירות יש השפעה לא זניחה”, אתה יכול באותה מידה לכתוב: “גם לחוסר תנועה יש השפעה לא זניחה”. כי אין הבדל בין מהירות קבועה למנוחה.

  44. ישראל שפירא ואחד אחר
    ישראל – קרא שוב את הלינק ששלחת. בסוף המהמלך – הפוך כיוונים. תקבל שג’יין צעירה מג’ק ב-4 שניות. זה נכון שתאוצה מאטה את הזמן – אבל כמו שהסברתי שוב ושוב, וגם אתה ציינת (!) – גודל ומשמעות התאוצה חסרות משמעות לעומת זמן הטיסה הרציפה.
    אחד אחר – אתה צודק שיש לגרביטציה השפעה גדולה על הזמן. אבל גם מהירות יש השפעה לא זניחה.
    בחייכם – תנסו לפתור את זה במשוואות של יחסות כללית – תראו למה תגיעו. למה לדבר סתם?

  45. ישראל, ניסיתי לכייל את שעון הטמפרטורה תחת דופלר ולא עלה בידי. פשוט, בכל מהירות שנסעתי, ובכל כיוון, לא נרשמה הסחה. מזה אני מסיק שכנראה קרינת הרקע הקוסמית איננה נובעת ממקור אחד או מקורות בודדים (שזה בעצם תנאי לקיום תוצא דופלר) אלא שהקרינה הזאת נוצרת בחלל “הריק” בכל נקודה ונקודה ביקום. למעשה, זו אחת התעלומות שהפיסיקאים טרם פתרו (לצד החומר האפל והאנרגיה האפלה). קרינת הרקע הקוסמית זהה בכל כיוון ומרחק שמהם היא מגיעה. בלי להיכנס כרגע לפרטים, זה מצביע על תנועה של מידע במהירות העולה בהרבה על מהירות האור – דבר שהוא עבירה חמורה על חוקי מדינת פיסיקה.

  46. התגובות נמשכות ונמשכות, אבל לדעתי שניכם צודקים.

    וגם שניכם טועים.

    ניסים – אינך יכול לדבר על פרדוקס התאומים בלי לערב תאוצה. הדוגמא שנתתי שבה אתה רואה את התארכות הזמנים גם במערכות אינרציאליות, נפתרת עיי סנכרון שעונים. השאלה ששאלת בקשר לחלליות אינה שלמה, כפי שב׳ ציין באחת התגובות שלו. אם תרצה, אראה לך למה.

    אחד אחר – נכון שאין פרדוקס תאומים ללא תאוצה, אך משך הזמן שבו שוהה התאום הצעיר בתאוצה זניח ביחס למשך הזמן שבו הוא בתנועה אינרציאלית, שהיא הגורם העברי להבדל בזמנים בין התאומים.

    אילו הייתם מנסים לפתור את חידת חיי הנצח שלי, הייתם נתקלים אולי בבעיה חדשה לגמרי בנושא התארכות הזמנים.

  47. א)
    לא קשור- האירוע הוא גם ככה או ביחידת הקצה אצלך או בלווין- האפקט נגרם גם בגלל הפרשי גרביטציה- שזה בדיוק כמו תאוצה.
    ב) כן -ולכן ניסוי מחשבה הוא לא לגמרי הדרך לעבוד כאן.
    ג) במקרה הזה- כן – לגמרי- פרופסור לאסטרופיסקה זה לא משהו זניח- הוא מתעסק הפיסקה הזאת כחלק מהמחקר השותף שלו כל הזמן- זה סביר להניח שהוא יודע על מה הוא מדבר. האם אתה פרופסור לאסטרופיסקה או משהו אחר בפיסקה שרלונטי?
    ד) נכון – זה הפרדוקס- והפתרון לפרדוקס הוא שהתאוצה/כבידה היא שונה ולכן אין סימטריות במערכת.
    זמן שונה במערכות שונות זה לא אותו דבר.
    פרדוקס התאומים הוא בדיוק הסיפור על התאומים ולא תסריט אחר עם מערכות אחרות- מה שאתה מדבר עליו זה פשוט יחסות פרטית רגילה. וזה לא פרדוקס התאומים.

  48. אחד אחר
    א) לא נכון. אנחנו יודעים שהשעונים בלוויני GPS רצים במהירות שונה מהשעונים במקטע הקרקעי.
    ב) תורת היחסות היא באמת לא אינטואיטיבית. גם החוק הראשון של ניוטון אינו אינטואיטיבי. כדי לבדוק את זה תשאל ילד קטן מה יקרה לטיל בחלל שכה לו המנוע.
    ג) אז מה שנכון זה בהכרח מה שאומר מישהו בעל יותר השכלה?
    ד) הפרדוקס הוא שהחללית נעה לעומת כדור הארץ באותה המהירות יחסית – אבל עובר יותר זמן בכדור הארץ מאשר בחללית.

    נכון – אפשר להתעלם מהתאוצה ולהתחשב רק במהירות. הראיתי שהחישוב מסתדר יפה מאוד. העניין הוא שהמרחק שנע החללית שונה בשתי מערכות הצירים. המרחק במערכת צירים של כדור הארץ-מאדים קבוע אבל המרחק שרואה החללית תלויה במהירות שלה. לכן ההפרש השעונים. ללא כל קשר לתאוצה. המהירות שווה ב-2 מערכות הצירים + המרחק שונה ==> הזמן יהיה שונה.
    זה הכל 🙂

  49. ניסים-
    א) אפשר לבדוק הפרש זמנים רק באותה מערכת באותו מקום.
    ב)יחסות פרטית וכללית הם לא תמיד אינטאיטיבים (סולם באסם מזכיר לך משהו?) ופשוט להשתמש בניסוי מחשבה פשוט כזה לא תמיד יתן לך תוצאות נכונות.
    ג) תאוצה היא ההסבר שאני מכיר- מי שלימד אותי את זה לא היה אתר באינטרנט אלא פרופסור לאסטרופיסקה מהטכניון.
    ד) עדיין לא ברור לי איפה הפרדוקס בתסריט שלך.
    הסברת את פרדוקס התאומים כמו שאני מכיר אותו ואיך הוא נפתר בבמודל של פיסקה יחסותית.
    ממה שהבנתי ממך אתה מסכים אם מה שאמרתי ואם זאת אומר שהתיחסות לתאוצה היא איכשהו לא הפתרון.
    אז מה כן הפתרון?

  50. tאחד אחר
    לא נתת הסבר. אמרת שאתה זוכר משהו…..אני נתתי הסבר מפורט. נתתי לךהסבר ללא כל תאוצה, והראתי שגול התאוצה חסר שממעות.
    המשפט האחרון שלך נכון. אבל זה פרדוקס הפוך 🙂 קח שעון לחלל, במהירות נמוכה. תחחזיר אותה לאט. אתה תראה שהזמן בשעון שבחללית מראה שעה מאוחרת יותר. כלומר – בחלל מזדקנים מהר יותר. ופה אין לזה שום קשר למהירות.

    בבקשה – קרא כאן להבין http://www.einsteins-theory-of-relativity-4engineers.com/paradoxes-of-relativity.html

  51. אני פשוט אומר לך מה אני זוכר מהקורס.
    ואני די בטוח שזה הולך ההפך- כל עוד אתה לא מוצא בהסבר שלי שגיאה אתה אמור לקבל את הטענה שלי.
    אמרתי לך- כן התאוצה זה מה שמשנה.
    וניסים
    לפי תורת היחסות- אין הבדל בין גרביטציה לתאוצה – מי שנמצא בתוך חללית לא יכול להבדל אם החללית מאיצה או שהחללית עומדת על פלאנטה עם כבידה מתאימה.

  52. אחד אחר
    חבל שאתה לא מתייחס למה שכתבתי. כל עוד אתה לא מוצא בהסבר שלי שגיאה אתה אמור לקבל את טענתי.

    ודרך אגב – גם בתורת היחסות הפרטית יש תאוצות. תורת היחסות הכללית מוסיפה כבידה.

  53. אז זהו – שלא- התאוצה היא הכל.
    זה רשום אצלי במחברת איפשהו וזה לא כזה פשוט- אבל התאוצה זה מה שמשנה כאן-
    זה לא אינטאיטיבי- אבל יחסות פרטית היא גם לא אינטואיטיבית במיוחד.
    הכל מגיע מנוסחאות ומקבל אישושים בכל מיני דרכים-
    זה בדיוק התחום שהוא מעבר ליחסות פרטית אבל עדיין לא בדיוק בעולם של יחסות כללית.

  54. אחד אחר
    בוא תניח שהחללית בכל זאת עוצרת לי מאדים. נניח שהמסע ארוך שנים רבות והפרש הזמנים הוא של 10 שנים. עכשיו חשוב על האטה במשך חודש, יום, שניה או אלפית שנייה. יש כאן הפרש עצום בהאטות. תחשוב אפילו על מצב שהחללית, בשלב האחרון של הטיסה מאיצה למהירות עצומה, בולמת, נוסעת אחורה, קדימה, הצידה, וכל זה במשך חודש — המקסימום שזה יכול להשפיע זה רק חודש…..
    אבל אם נכפיל את במרחק באותה מהירות – נקבל כבר הפרש של 20 שנה.
    כלומר – התאוצה/תאוטה כמעט חסרת משמעות.

  55. אם אני זוכר נכון- השוואת גילאים היא חסרת משמעות כל עוד המערכות לא מתאחדות
    מערכת שזזה ליד מערכת בלי להאיץ או להאיט- אין לא הזדמנות להשוות גילאי תאומים ולכן זה לא הפרדוקס.

  56. ניסים: אתה לא רואה איך אפשרלסנכרן שעון עם gps?
    אתה באמת טמבל.
    הרי הgps עובד על בסיס שעון אטומי.
    טוב.
    נראה לי שחטפת מספיק בחיים… נעזוב אותך בשקט

  57. ישראל שפירא
    אני לא רואה איך תסנכרן שעון ל-GPS. מתי תדע מה רגע יציאת האות? מכשיר GPS משתמש בלווין רביעי כדי לפתור בעיה זו. המידע שמספק המקלט הינו לאחר עיבוד ואני לא מכיר דרך לדעת את זמן ההגעה…

  58. tאחד אחר
    בוא תסתכל על המצב הבא. יש שלושה צופים. אחד על כדור הארץ אחד על מאדים ואחד בחללית. הצופים על כדור הארץ ומאדים מסנכרנים שעונים. עכשיו – החללית חולפת ליד כדור הארץ וברגע החליפה הצופה בחללית גם מסנכרן שעון. הוא טס במהירות גבוהה (וקבועה כל זמן הניסוי) למאדים. כשהוא יגיע למאדים יהיה הפרש בזמנים בין החללית לשני הצופים האחרים.
    כלומר – הצופה בחללית הזדקן פחות מהצופה במאדים (ששווה בגילו לצופה שבכדור הארץ). אבל – כדור הארץ “התרחק” מהחללית בדיוק כמו שהחללית התרחקה מכדור הארץ – והרי לך הפרדוקס – בלי שום תאוצות ושום נעליים.

    ובכן – קיבלת ניסוי ששקול לניסוי הקלאסי וראית שאכן יש פער בגילאים – ללא תאוצות.

    לזה אני בטוח שגם ישראל שפירא מסכים 🙂

  59. יובל

    כאשר אתה מודד את טמפרטורת הקרינה, אתה משתמש במד קרינה. אם תרצה לדעת את מהירותך יחסית לקרינה, אתה יכול למדוד אותה בשני כיוונים, קדימה ואחורה, ועיי השוואת ההסחות לאדום וכחול לחשב את מהירותך יחסית אליה. החישוב של טמפרטורת הקרינה הנייחת במקום בו אתה נמצא הוא קל לחישוב, במיוחד אם אתה משתמש במחשב של מקינטוש.

    לכן כל שני נוסעים החולפים זה על פני זה יכולים תמיד להסכים על זמן משותף, הזמן שחלף מאת המפץ הגדול המחושב באמצעות נוסחת פרידמן הרלוונטית.

    ולכן אין להם כל בעיה לסנכרן את שעוניהם לאותו הזמן, וזאת בניגוד לידע שהיה ב1905. אז, באמת לא הייתה קיימת כל אפשרות לסנכרן שעונים אלא באמצעות קשר ביניהם – איינשטיין הציע לעשות זאת באמצעות קרני אור, שמהירותם זהה לכל מודד, ומכאן היחסות.

    אולם אין זאת אומרת שלא קיים זמן אוניברסלי, שניתן תמיד למדידה באמצעות מדידת קרינת הרקע הקוסמית, דהיינו השעון הקוסמולוגי.

    ניסים – תודה על התשובות המפורטות. אני מנסה לבנות מתקן שיוכל למדוד במדוייק את זמן ההגעה של סיגנל מלווין GPS. מכיוון שכל מכשיר GPS עושה בדיוק זאת, נראה לי שזה לא אמור להיות מסובך במיוחד.

  60. ניסים- דבר ראשון ויקפדיה היא לא מקור מאה אחוז מהימן לשום דבר- בין ויקפדיה ופרופסור לאסטרופיסיקה- אני מעדיף את הפרופסור.
    אתה זה שהבאת את המקור של ויקפדיה בעיברית ואני רק ציטטתי אותה.
    דבר שני זה לא בהכרך יחסות כללית אבל התאוצה היא שובר השיוויון כאן- הקורס שלמדתי נמצא במעבר בין פרטית לכללית. ואתה לא אמרת אחרת- באמת שלא ברור לי מה ה אמרתי שאתה לא מסכים איתו – אתה כל הזמן אומר שאני טועה ואז אומר משהו שהוא די בדיוק מה שאני אמרתי.

  61. tאחד אחר
    מדהים!!!! זה באמת מה שכתוב בויקי בעברית!!! אבל בוא נקרא באנגלית:

    Resolution of the paradox in special relativity

    The standard textbook approach treats the twin paradox as a straightforward application of special relativity. Here the Earth and the ship are not in a symmetrical relationship: regardless of whether we view the situation from the perspective of the Earth or the ship, the ship experiences additional acceleration forces. The ship has a turnaround in which it undergoes non-inertial motion, while the Earth has no such turnaround. Since there is no symmetry, it is not paradoxical if one twin is younger than the other. Nevertheless it is still useful to show that special relativity is self-consistent, and how the calculation is done from the standpoint of the traveling twin.

    Special relativity does not claim that all observers are equivalent, only that all observers at rest in inertial reference frames are equivalent. But the space ship jumps frames (accelerates) when it performs a U-turn. In contrast, the twin who stays at Earth remains in the same inertial frame for the whole duration of his brother’s flight. No accelerating or decelerating forces apply to the twin on Earth.

    There are indeed not two but three relevant inertial frames: the one in which the Earth-based twin remains at rest, the one in which the traveling twin is at rest on his outward trip, and the one in which he is at rest on his way home. It is during the acceleration at the U-turn that the traveling twin switches frames. That is when he must adjust his calculated age relative to the Earth-based twin.

    Minkowski diagram of the twin paradox. Time is relative, but both twins are not equivalent (the ship experiences additional acceleration due to changes in the direction of travel).
    In special relativity there is no concept of absolute present. The present from the point of view of a given observer is defined as the set of events that are simultaneous for that observer. The notion of simultaneity depends on the frame of reference (see relativity of simultaneity), so switching between frames requires an adjustment in the definition of the present. If one imagines a present as a (three-dimensional) simultaneity plane in Minkowski space, then switching frames results in changing the inclination of the plane.

    In the spacetime diagram on the right, drawn for the reference frame of the Earth-based twin, that twin’s world line coincides with the vertical axis (his position is constant in space, moving only in time). On the first leg of the trip, the second twin moves to the right (black sloped line); and on the second leg, back to the left. Blue lines show the planes of simultaneity for the traveling twin during the first leg of the journey; red lines, during the second leg. Just before turnaround, the traveling twin calculates the age of the Earth-based twin by measuring the interval along the vertical axis from the origin to the upper blue line. Just after turnaround, if he recalculates, he’ll measure the interval from the origin to the lower red line. In a sense, during the U-turn the plane of simultaneity jumps from blue to red and very quickly sweeps over a large segment of the world line of the Earth-based twin. The traveling twin reckons that there has been a jump discontinuity in the age of the Earth-based twin.

    The twin paradox illustrates a feature of the special relativistic spacetime model, the Minkowski space. The world lines of the inertially moving bodies are the geodesics of Minkowskian spacetime. In Minkowski geometry the world lines of inertially moving bodies maximize the proper time elapsed between two events.

    באנגלית ההסבר נכון. ואין שום צורך ביחסות כללית.

  62. ניסים- לא ברור לי איפה מה שאמרת הוא סותר למה שאני כתבתי.
    אבל אני אמשיך ואצטט מויקפדיה:
    “אולם למעשה אין סימטריה בין השניים. התאום שנשאר על כדור הארץ נשאר כל הזמן באותה מערכת ייחוס – זו של כדור הארץ. לעומתו, התאום הנוסע החליף מערכות ייחוס. בתחילה הוא היה במערכת שנעה במהירות v+ יחסית לכדור הארץ, אבל לאחר מכן הוא הפך כיוונו ועבר למערכת ייחוס שנעה במהירות v-. בסיבוב הזה זה היה הוא שהפעיל את המנועים, ולא כדור הארץ, ולכן בפרק הזמן הזה הוא היה בתאוצה, בעוד אחיו התאום היה בתנועה במהירות קבועה. כאשר הוא היה בתאוצה חלו עליו חוקי תורת היחסות
    הכללית ולא הפרטית, והם בעצם אלו שגרמו להזדקנות המואטת.”

  63. ישראל שפירא (אני מניח)
    אני לא מבין בדיוק מה אתה רוצה. למקלט יש מספר ערוצים. צריך לפענח לפחות 4 ערוצים כדי לקבל מיקום (כי לנו אין שעון אטומי ביד….). וכל הערוצים הם באותם שני תדרים…..
    השלב הראשון, כמו שאמרתי, הוא שימוש באות רעש (כאילו אקראי). לאחר מכן מקבלים בלוקים של מידע באורך 30 שניות ובתדר של 50 הרץ. כלומר 1500 ביט. בבלוק כזה יש 3 דוגי מידע – נתוני שעון, נתוני אלמנך ונתוני אפמריס. האלמנך מכיל מידע על כל הלווינים והאפמריס מידע (בתדר גבוה יותר) על הלווין הספציפי.
    אם אתה רוצה לראות את המידע עצמו אז יש מספר פרוטוקולים מוכרים. אני עבדתי עם NMEA 0183 ויש פרוטוקול נוסף שנקרא TIPY.

    ואפשר לכתוב כאן עוד הרבה מאוד …….

  64. מר ניסים הנכבד
    בבקשה לא את ויקי וכל החסימות שלי שם
    שוב לא אחד על הארץ ושני בחלל – שני תאומים סימטרים בחלל
    תודה

  65. אחד אחר
    כל הכבוד לפרופסור שלך … אבל ההסבר שלי הוא נכון.
    חבל שלא ניסית להבין את ההסבר שלי. הוא באמת פשוט. אם משהו לא ברור שם – תשאל. אם יש טעות בהסבר – תגיד מה הטעות.
    אני יודע שאני מבזבז את הזמן שלי – אבל הנה תחילת הערך בויקיפדיה
    In physics, the twin paradox is a thought experiment in special relativity involving identical twins, one of whom makes a journey into space in a high-speed rocket and returns home to find that the twin who remained on Earth has aged more.
    ואם זה קשה לך …. הנה הערך בעברית

    “פרדוקס התאומים הוא ניסוי מחשבתי פרדוקסלי בתורת היחסות הפרטית לפיו אדם נוסע במהירות קבועה, הזמן שהוא ימדוד בין שני אירועים בהם הוא נמצא פיזית (ולכן עבורו קורים באותו מקום) יהיה קצר יותר מהזמן שימדוד כל אדם הנמצא במערכת ייחוס אחרת, בה שני האירועים קורים במקומות שונים.”

    בהצלחה ……..

  66. אחד אחר הנכבד שוב שלום
    אם אחד רואה מהצד שלו שחור והשני רואה מהצד שלו לבן אז כנראה הם בעולמות מקבילים
    בכבוד תודות

  67. אחד אחר הנכבד שלום
    בקשר לפרדוקס התאומים – תקח דוגמא סימטרית לגמר תראה שהיא סימטרית ותבין שיש סתירה .
    לא אחד על הארץ ושני בחלל – שניהם בחלל ומאיצים אחד ביחס לשני .
    יום טוב ותמשיכו להתווכח אחר כך על זמן שלילי אם בא לכם כי זה ועולמות מקבילים הוא הפתרון לפרדוקס .
    תודה

  68. אחד אחר
    אז ככה…. ההסבר שקיבלת על פרדוקס התאומים היה כנראה מסובך להפליא. יש הסבר מאוד פשוט לפי הנחות תורת היחסות הפרטית. התיאור של הפרדוקס שאתה נותן הוא נכון וזה באמת נראה פרדוקסלי. אבל הנחת הסימטריה היא שגויה. הסברתי כבר כמה פעמים אבל אסביר שוב.
    נניח שהחללית טסה לנקודה במרחק 20 שנות אור במהירות 0.8c
    מבחינת התאום ששכחו אותו בבית – הזמן שהוא עובר הוא 50 שנה.
    מבחינת התאום הנוסע – המרחק לנקודה מתקצר והוא רק 12 שנות אור. כלומר הזמן שעובר מבחינת התאום הנוסע הוא 30 שנה.
    כלומר – יש הפרש גילאים כם ללא כל תאוצה.

    עכשיו שים לב – בוא נניח תאוצה חזקה ביותר כך שהמהירות עולה ל 0.8c בזמן קצר ביותר, וההאטה בהתאם. בתאוצה עצומה זו הזמן כמעט נעצר. כלומר בזמן קצר ביותר התאום לא מזדקן. לכן נוצר הפרש קטן מאוד בגילאים. סה”כ נקבל אותה תוצאה כמו קודם.
    כלומר – תאוצה חזקה ביותר לא משפיעה על המצב ולא נראית רלוונטית לפרדוקס.

    ואם כרב רוצים להכניס את תורת היחסות הכללית לסיפור – זכור שהתאום שנשאר בבית נמצא בשדה כבידה וזה דווקא יקטין את הפרש הגילאים.

    אתה מקבל את ההסבר?
    —————————-
    לא- נכון שזה נשמע לא אינטואיטיבי – גם לי זה לא נשמע ככה- אבל זה מה שזה – מי שהאיץ- אפילו לרגע קצר כרצוננו- הוא זה שיושפע מהאפקט ולא השני- מדבור כאן על התחום שבין יחסות פרטית וכללית .
    (קורס מרחבי זמן וחורים שחורים אצל נועם סוקר- ואני ממש לא זוכר את המתמטיקה- אבל ככה זה.)
    בלי להתיחס לתאוצה האחים הם סימטרים- בלי תאוצה או גרביטציה- מבחינת האח בחללית האח השני על כדור הארץ זז במהירות קרובה למהירות האור ונסע הוא 10 שנות אור וחזרה.
    ומכאן הפרדוקס.

  69. ניסים
    אני רוצה לראות את המידע שמתייחס לנ”צ שבהם אני נמצא.
    מהם התהליכים (צעד אחר צעד) שקורים כאשר מתקבל המידע במכשיר ואם אפשר, “בפרוטרוט”.
    שמעתי שאתה מומחה בgps נשמח לשמוע…

  70. ישראל שפירא
    גם האות מכל לווין אינו נקי. הבעיה היא עוצמה חלשה ביותר. והפתרון זה לקלוט מידע לאורך זמן ולחבר את האותות. בחיבור אותות יחס האות לרעש משתפר (נדמה לי לפי שורש מספר הדגימות). האות בתדר מאוד גבוה מעל 1 גיגה. אם תרצה לבצע דמודולציה תצטרך מעגל מתאים. ובכל מקרה צריך את 2 התדרים. איזה מידע אתה רוצה לראות?

  71. אז כן התכוונת לזה 🙂
    דופלר? כלומר למדוד בשני מדחומים? איפה תתקע אותם?
    אם במשך חיי הנצח שיוענקו לי אצטרך להשתתף בדיונים מרתוניים באתר הידען, אני מוותר 🙁

  72. יובל

    שששש.. אל תלשין. תן לג’מע הזדמנות לזכות בחיי נצח.

    אבל מכיוון שכבר שאלת – מה הבעיה לשקלל את הדופלר הדו כיווני כדי לדעת מה מהירותך יחסית לקרינה?

    ניסים

    תודה על האינפורמציה.

    אז אתה אומר שבלוק מידע משודר מכל לווין כל 30 שניות? מדוע לא אוכל להתביית על שידורי לווין בודד לדוגמא? הרי כל מכשיר GPS עושה זאת, ועם כמה לווינים.

  73. ישראל שפירא
    האות מ-GPS הוא חלש ביותר והוא משודר בצורה כזאת שצריך לבצע סכימה לאורך זמן כדי לקבל מידע. יש שני גלים (בתחום האזרחי). תדר השידור עצמו הוא בערך 1.2GHz ו 1.5Ghz. המידע עצמו, מאופנן בצורה מוזרה אמנם …. הוא ב 50Hz ובלוק מידע משודר כל 30 שניות (1500 ביט). ויש 2 סוגי מדע – אלמנך ואפמריס.

    יש פרוטוקולים סטנדרטיים שהם המידע שמוציא המקלט. למשל NMEA 0182 (אם אני זוכר נכון). זה פרוטוקול טורי שקל להבין אותו.

    אל תצפה לראות משהו בסקופ ….. במקרה הטוב תראה גל רועש מאוד בתדר 1 MHz

  74. ישראל, בהתחלה חשבתי שאתה מתכוון למדחום המודד את טמפרטורת קרינת הרקע הקוסמית, הזהה בכל מקום ביקום ויורדת עם הזמן. גם אם נקבל כעובדה מוצקת שקיימים מכשירים המאפשרים את תרגום הטמפרטורה הנ”ל לקריאת זמן בשעון תקני (לפי נוסחת פרידמן), עדיין לא ברור אם בגלל תנועת החלליות קריאת המדחום אינה מתעוותת. שהרי טמפרטורה היא מדד לאנרגיה קינטית, וזו מתבטאת במהירות תנועתם של חלקיקים. לפיכך אני מבין שלא לזה התכוונת. אנא תקנני

  75. “האמת היא שאנחנו צריכים קצת בדיחות קרש, שיהיה לל”ג בעומר”.

    אתה יכול להשתמש בזאת.

  76. לפעמים זה נוח לא לכתוב נקודות, לכן איני מוטרד מכך ואין לי בעיה להישאר באנונימיות מתי שנוח לי. בכל מקרה מזהים כך שזה לא משנה.
    לגבי המכוניות אין צורך להיכנס שוב לויכוח.
    לגבי חיי הנצח… רק בתנאי שתסדר לי חללית שבהילוך ראשון המהירות שלה הוא שיא

  77. אנו,
    לא מאמין שיש טעויות בחידת מכוניות השעטנז. אם כן, הראה. את חידת החלליות החדשה אתה מוזמן לנסות לפתור. אולי גם תזכה לחיי נצח.
    ואתה יכול כבר לצאת מהאנונימיות.

  78. ישראל,
    בחידת המכוניות יש טעויות.
    על החידה של חלליות לא דיברתי.
    והחידה האחרונה מתאימה בדיוק לניסים.
    :))

  79. ניסים העובדה שאתה פולט שטויות לא תהפוך אותך למשכיל. עם ההגיון שלך תוכל רק ללכת לאיבוד בתוך הסרט שאתה חי. אם יש לך בעיה עם המציאות תלך לפסיכולוג. אתה מעורר רחמים.

  80. ניסים

    בתור אחד שמבין בGPSים, ידוע לך אולי מה פרק הזמן הטיפוסי של פולס המגיע מלווין GPS?, האם וכיצד אוכל לקלוט אותו באמצעות מקלט רדיו כלשהו? או איך אוכל לחלץ אותו ממקלט GPS כך שאוכל להציג אותו על אוסציליסקופ?

    תודה.

    וביינתיים, מכיוון שמייקל קצת מחפף בנושא החידות, הנה לכם החידה היומית:

    אי שם במעבה החלל, במרחק אלפים רבים של שנות אור מכדור ארצינו, באיזור שזכה באתרינו לכינוי “טיז אל נאבי” ישנה קוביית חלל ריק שמקצועה שווה ל10 שנות אור.

    בטיז אל נאבי אין כלום – לא שמשות, לא פלנטות, לא שביטים, לא מטאורים. רק חושך על פני תהום. בעצם אפילו לא תהום (בטח שלא תהום – הרי אין שם כלום!).

    יש שם רק דבר אחד, במרכז טיז אל נאבי: תיבה סגורה ובה סוד חיי הנצח.

    אולם אין זה כה קל להגיע לאותו הסוד: כדי לפתוח את אותה התיבה, יש להאיר עליה משני כיוונים, משתי ספינות חלל שונות, במרחק רב מהתיבה כשאין אינן רואות או מתקשרות זו עם זו, בתוך פרק זמן שלא יעלה על שעה אחת הנמדד עיי השעון האטומי שבתיבה.
    על הספינות לעשות את דרכן במסלולים שונים דרך גלקסיות שונות, במהירויות ותאוצות שונות וללא כל קשר ביניהן, ואסור להן לצבור נתונים על הדרך שעברו.

    מאות רבות של הרפתקנים יצאו לדרך כדי לזכות בפרס הנכסף. אף אחד מהם לא הצליח. תמיד גילו שכאשר הגיעו לאיזור התיבה, שעוניהם יצאו לגמרי מסנכרון בגלל המהירויות והתאוצות השונות שחוו בדרך, ולכן אין באפשרותם לסנכרן את הקרניים כך שיפגעו בתיבה בתוך השעה המתבקשת.

    מהות החידה:

    נאמר ששתי ספינות מצויידות במיכשור מדוייק ככל שיחפצו. האם הן תוכלנה לעמוד במשימה ולזכות בחיי נצח?

    1. עם ברשותם הידע שהיה לאיינשטיין ב1905 עת הגה את היחסות.

    2. עם הידע המצוי בידינו כיום.

    נסו אתם מגיבים חביבים, לפתור את החידה, ואולי תזכו גם אתם לחיי נצח.

  81. ניסים:
    צר לי לומר זאת אבל דבריך הם פשוט קישקוש!
    האם באמת עשית את הניסוי עם אכילס והצב?
    רמז לתשובה: לא!
    לכן מדובר במודל.
    האמת היא שגם לו עשית את הניסוי היה מדובר במודל שהוא מה שמוחך מפרש אודות המציאות.
    אבל באמת שאין לי כבר כוח.

  82. אחד אחר
    אתה כמובן צודק שבפיסיקה אין הוכחות.
    בקשר לאכילס והצב – אתה צודק בהבחנה שלך שטור וסידרה זה דברים שונים. זה בלבול מאוד נפוץ גם אצל מרצים בטכניון….
    בכל מקרה, לדעתי זה לא הפתרון לפרדוקס הזה. אבל זו רק דעתי….

  83. אחד אחר
    אז ככה…. ההסבר שקיבלת על פרדוקס התאומים היה כנראה מסובך להפליא. יש הסבר מאוד פשוט לפי הנחות תורת היחסות הפרטית. התיאור של הפרדוקס שאתה נותן הוא נכון וזה באמת נראה פרדוקסלי. אבל הנחת הסימטריה היא שגויה. הסברתי כבר כמה פעמים אבל אסביר שוב.
    נניח שהחללית טסה לנקודה במרחק 20 שנות אור במהירות 0.8c
    מבחינת התאום ששכחו אותו בבית – הזמן שהוא עובר הוא 50 שנה.
    מבחינת התאום הנוסע – המרחק לנקודה מתקצר והוא רק 12 שנות אור. כלומר הזמן שעובר מבחינת התאום הנוסע הוא 30 שנה.
    כלומר – יש הפרש גילאים כם ללא כל תאוצה.

    עכשיו שים לב – בוא נניח תאוצה חזקה ביותר כך שהמהירות עולה ל 0.8c בזמן קצר ביותר, וההאטה בהתאם. בתאוצה עצומה זו הזמן כמעט נעצר. כלומר בזמן קצר ביותר התאום לא מזדקן. לכן נוצר הפרש קטן מאוד בגילאים. סה”כ נקבל אותה תוצאה כמו קודם.
    כלומר – תאוצה חזקה ביותר לא משפיעה על המצב ולא נראית רלוונטית לפרדוקס.

    ואם כרב רוצים להכניס את תורת היחסות הכללית לסיפור – זכור שהתאום שנשאר בבית נמצא בשדה כבידה וזה דווקא יקטין את הפרש הגילאים.

    אתה מקבל את ההסבר?

  84. ניסים
    אם אתה רוצה להתוכח עם הפרופסור לפיסיקה שהסביר לי את זה בקורס מיוחד שמתיחס לנושא הזה כחלק מהתואר רשאון בפיסיקה-
    תרגיש חופשי- יכול להיות שלא הסברתי את הרעיון כראיו אבל פרדוקס התאומים כמו שאני מכיר אותו הוא כזה:
    יש שני תאומים – אחד עולה על חללית ונוסע קרוב כרצוננו למהירות האור במשך 10 שנים- ואז הוא משנה כיוון ונושא עוד 10 שנים בחזרה הביתה-
    לפי הנוסחאות מעט זמן עבר על התאום בחללית ו20 שנה על זה שנשאר- ה”פרדוקס” הוא בכך שמבחינת האח בחללית במערכת היחוס שלו- זה התאום שעל כדור הארץ שהתרחק ממנו במהירות השואפת למהירות האור וחזר- אז לא הגיוני שהוא זה שבקושי יזדקן?- ה”סימטריה” כאן נעלמת שמבינים שהאח בחללית חווה תאוצה גדולה-שזה לפי יחסות כללית בלתי ניתן להבדלה מכבידה מבחינת התאום- וזה מה שעושה את כל ההבדל ופותר את ה”פרדוקס”.
    בקשר לאכילס והצב- הבעיה נובעת כמובן מכך שהזמן מתכנס ביחד עם אכילס ולכן אין התקדמות.
    אכן אין דבר כזה סכום של טור אינסופי אלא רק גבול של טור אינסופי- אבל זאת בעיה של המתמטיקה ולא של המציאות לכן המציאו את החדו”א כדי שיוכלו למצוא את הגדלים האלו ולהתמודד אם הפרדוקס הזה.
    דבר אחרון, הוכחה זה לא מונח ממדע אמפירי ומחקרי- רק במקצעות מתמטים ולוגים יש הוכחות-
    בפיסיקה יש רק אישושים.

  85. מיכאל רוטשילד
    מה שאתה אומר נהיה יותר ויותר שגוי. אכילס והצב אינו פרדוקס בלוגיקה!
    הבעיה אינה עם מודל של המציאות. הבעיה היא עם המציאות 🙂 הרי אכילס מנצח את הצב במציאות. זה מה שמפריע לנו – זה הפרדוקס! במודל האבסטרקטי אכילס כלל לא מגיע לצב. במציאות כן.
    הנחנו הנחות:
    1. במציאות – המרחק (או הזמן) הם רציפים
    2. במציאות – אכילס מנצח את הצב

    להזכירך, הפתרון של דיוגנס היה פשוט לקום וללכת…… והפתרון שלך (ושל רבים) זה להגיד שסכום הטור האינסופי הוא סופי. שניכם לא מבינים, לטעמי, את הבעיה.

  86. ניסים:
    אינך היחיד שחושב כך וזה לא שייך לדיון.
    למעשה זה לא שייך לשום דיון אבל נעזוב את זה כרגע.
    האם אתה חולק על כך שמה שאנחנו דנים בו הוא מודל של המציאות (כפי שאנחנו מדמיינים אותה במוחנו)?
    האם אתה מסכים עם זה שהמודל הוא אבסטרקטי?
    אם כן – הרי שלטענה שאין בעולם דבר אבסטרקטי – אין כל רלוונטיות לדיון (מלבד העובדה שהיא טוענת שהוא וכל המחשבות הכרוכות בו כלל לא מתקיימים).
    מעולם לא טענתי שלא ייתכן שאין אינסוף אירועים. טענתי רק שמחשבה זו היא הצעה למודל אחר של המציאות ולא הסבר לפרדוקס.
    קרא את הקישור שהצבעתי עליו ותראה שפרדוקס, כפי שכבר אמרתי, הוא ביטוי ששייך לתחום הלוגיקה.
    המציאות כלל לא שייכת לעניין!
    השאלה היחידה בהבנת הפרדוקס היא האם יש כאן טעות לוגית ומהי אותה טעות לוגית.
    אתה מוזמן להמשיך לדבר על המודל הקוונטי של הזמן אבל דע לך שכל עוד אתה עושה זאת אינך מתייחס לפרדוקס.
    אגב – אינני יודע איזו סיבה יש לך לחזור לקוונטיות של הזמן כי אין לי וויכוח אתך בעניין. אני לא יודע אם הזמן הוא קוונטי (וגם אתה אינך יודע זאת. למעשה אין בעולם אף אחד שיודע זאת).
    הנ”ל הוא גם התייחסות לטענתך שאיננו מסכימים למושג אינסוף.
    אתה מסכים למושג אינסוף אבל אתה רק חושב שמספר אירועים ברצף הזמן לא יכול להיות אינסופי.
    זו טענה על (המודל של) המציאות ולא על המושג אינסוף.
    שים לב שאתה לא מוכן לקבל אינסוף אירועים ללא שום קשר לפרדוקס. זה וויכוח שיש לך עם מודל כלשהו של המציאות וזה וויכוח שיכול להיות מוכרע רק בניסוי.
    ניסוי כזה לא נעשה בינתיים והעובדה שיש לבעיה פתרון מתחום הלוגיקה והמתמטיקה מוכיחה שגם הניסוי של אכילס והצב אינו מהווה עדות לכך שהזמן הוא קוונטי.
    זהו.
    יש גבול למאמץ שאני מוכן להשקיע מבלי שבכלל מנסים להבין מה אני אומר.

  87. מיכאל רוטשילד
    אז אני חושב שאנחנו לא מסכימים למושג “אינסוף”. הבעיה היא התרחשותן של אינסוף ארועים. הטענה שלי היא שייתכן והפתרון הוא שיש הנחה שגויה – ההנחה שאכן יש אינסוף ארועים.
    הטענה שלי לא סותרת שום דבר שאני מכיר בעולם.
    מה לא בסדר כאן?
    והכוונה שלי ב”יש טענה כך וכך” היא שאני לא היחיד שחושב ככה ……..לא יותר מזה.

  88. tאחד אחר
    אתה פשוט טועה. קרא את המאמר שישראל מפנה אליו. אין שום קשר לתאוצות בפרדוקס התאומים. אשמח להראות לך את החישוב, ללא כל צורך בתאוצות.
    דרך אגב – זה הוכח בניסוי לפני הרבה שנים. ובניסוי לא היו תאוצות….

  89. ניסים:
    המונח “פרדוקס” נועד לתאר בעיה בלוגיקה.
    http://en.wikipedia.org/wiki/Paradox
    סתירה להתנהגות העולם יכולה לנבוע מלוגיקה שגויה או ממודל לא נכון אבל היא אינה בהכרח פרדוקס.
    ברגע שאתה מציע מודל אחר לעולם אינך מתמודד עם הבעיה הלוגית ולכן אינך פותר את הפרדוקס.
    לוגיקה ומתמטיקה הן אותה הגברת בשינוי האדרת ופתרון הבעיה המתמטית הוא לכן הפתרון היחידי לפרדוקס.
    יש פרדוקסים שעוסקים בעולם הפיזיקאלי אבל הזכות שלהם להיקרא פרדוקס נובעת מן הבעיה הלוגית שנוצרת. אם לא נוצרת בעיה לוגית אלא רק סתירה עם הטבע אז זה לא פרדוקס אלא רק מודל שגוי של הטבע.
    במקרה שלנו יש פשוט טעות לוגית בשיקול המתעלם מן העובדה שטור הזמנים מתכנס ומרשה לעצמו להשתמש בביטוי כמו “אף פעם” (המתייחס לכל ציר הזמן) אחרי שראה אינסוף אירועים שכולם לפני השעה 10:00
    זו טעות מתמטית והיא הסיבה לתחושת הפרדוקס (כי באופן חישוב אחר אנחנו רואים שאכילס מגיע לצב ואף עובר אותו).
    מה שאתה מציע זה להתעלם מן הטעות המתמטית, לומר שהשיקול השגוי הוא נכון, ובעקבות זאת להחליף את המודל של הפיזיקה.
    זו טעות מן ההתחלה ועד הסוף.
    אגב, אני לגמרי רגוע.
    ועוד אגב: מעולם לא אמרתי שלפילוסופיה אין מקום בעולם אבל פילוסופיה היא אהבת המחשבה או החוכמה ולא אהבת הבירבורים. חלק נכבד מאלה המתקראים כיום פילוסופים אינם ראויים לתואר. בגלל “פילוסופים אלה” אומרים שכדי להיות מדען צריך נייר, עיפרון, מחק ופח אשפה וכדי להיות פילוסוף נחוצים רק נייר ועפרון.
    ועוד כמה דברים: “יש טענה שכך וכך” אינו טיעון. זה במקרה הטוב ציטוט ובמקרה הרע נפנופי ידיים.
    יש בעולם כמעט כל טענה שבני תמותה מסוגלים להוציא מפיהם וזה לא אומר כלום.
    הטענה הפילוסופית הספציפית שאתה מתאר היא דוגמה לפילוסופיה גרועה כי היא לא אומרת כלום – פשוט כלום.
    ללא כל קשר למה שיש בעולם – מה שיש במוחנו הם מודלים אבסטרקיים בלבד (כלומר – ברמת ההפשטה הראוייה – ברמה בסיסית יותר כמובן שיש במוחנו רק חלקיקים אלמנטאריים) ורק על המודלים האבסטרקטיים האלה אנחנו מסוגלים לדבר.
    אגב – לומר שמחשבותינו (כולל המודלים האבסטרקטים שבהן) לא קיימות בעולם זו סתם שטות.

  90. “פרדוקס” התאומים זה בדיוק על תאוצה (או כבידה) אפשר להראות את זה על ידי שימוש בנוסחאות.
    (לא שאני זוכר את זה בעל פה- המחברת נמצאת אצלי איפשהו).
    אני לא כל כך מצליח לעקוב אחרי הבעיה שאתם מדברים עליה- אבך פרדוקס התאומים הוא בהכרח נופל בכך שתאום אחד מאיץ והתאום השני לא.

  91. ישראל שפירא
    פה אתה טועה 🙂 בתורת היחסות הפרטית בהחלט יש תאוצות. אבל, פרדוקס התאומים כלל אינו מדבר על תאוצות. תחשוב שג’יל עוצרת בנקודה C2. אם הם היו תאומים לפני הטיסה אז יש ביניהם עכשיו 2 שניות הפרש.
    הדוגמא שהבאת בדיוק ממחישה את מה שאמרתי. נוצר הפרש גילאים ללא שום קשר לתאוצה.

    בצורה פשוטה – תורת היחסות הפרטית טוענת ש c קבוע בכל מערכת יחוס (גם במערכות צירים מואצות).
    תורת היחסות הכללית מראה שקילות בין תאוצה לכוח כבידה.

  92. ניסים
    אם הבנתי נכון את דבריך, אז בגלל הסימטריה ג’יל וג’יין תפגשנה בנקודה C2 ושעוניהן יראו 8.
    שעונו של ג’ק יראה 10 כמו מקודם.

    זה אינו פרדוקס התאומים למחצה – בפרדוקס התאומים אותם תאומים נפגשים. אתה הראית את פתרון הפרדוקס על פי היחסות הפרטית, שלא כולל תאוצות. אך הפתרון הכללי מדבר על תאוצה, שלא כלולה בדוגמאות שדברנו עליהן עד כה.

  93. אנונימי

    קרא שנית את הדוגמא. אין פה איפוס שעונים בנקודת המפגש – הוא בוצע קודם, לפני 8 שניות על פי שעון ג’יל.

    במקרה שלנו נקודת המפגש היא ג, או ד, המצויינות בתגובה שלך, ושם נערכו התצפיות.

    הבעיה היא שניסים טוען שכל צד יראה את שעונו של השני כמפגר בנקודת המפגש, בניגוד להגיון ולכתוב במאמר.

  94. ישראל שפירא
    הבנתי את הנקודה שלך. זה בדיוק “פרדוקס התאומים” – אבל רק חצי מהדרך. ג’ק הזדקן ב-10 שניות וגיל הזדקנה ב-8 שניות. שניהם מסכימים לזה. אתה צודק בזה ב-100%.

    אבל … אני חשבתי על מצב שונה. כך את הדוגמא שנתת, ובו נשכפל את זה בצורת ראי סביב C2. ג’יין יוצאת מהנקודה החדשה בצד ימין בצורה סימטרית לגמרי. השאלה שלי – מה תראה ג’יל בשעון של ג’יין ולהיפך? התחושה שלי שתגיד ששניהן תסכימנה שעברו 8 שניות. ואני חושב שאתה צודק ……

    אבל …. (תמיד יש אבל….) – האם ג’יין וג’יל ייפגשו בנקודה C2 ??? אני חושב התשובה היא כן …. למרות שזה נוגד מאוד את האינטואיציה שלי 🙂

  95. ישראל:
    אם שתי מערכות חולפות זו ליד זו אז בנקודת האפס המשותפת של זמן ומרחב יתקבלו אותם תצלומים . אותם קריאות שעונים. אבל בנקודות המרוחקות מנקודת האפס יתקבלו קריאות אחרות.
    בנקודה המרוחקת מרחק (מרחבי) כלשהוא מנקודת הראשית קריאות הזמנים יהיו שונות מקריאות הזמנים בנקודת הראשית.
    אם א חולף על פני ב אז תמיד הם יכולים לאפס את השעונים בנקודת המפגש שלהם . ואז כמובן קריאת השעונים תהיה זהה כלומר אפס בשני השעונים. אבל אם לגבי ג ו ד הנמצאים במרחקים מסויימים מן הראשית הקריאות יהיו אחרות.

  96. ניסים

    אז מה אתה טוען – שבדוגמא שהבאתי כאשר ג’יל חולפת על פני ג’ק היא תראה את שעונה מראה 10 ושעונו 8, וזה גם מה שיראה צילום של שני השעונים מחלליתה?

  97. ר.ח רפאי.ם
    1. יש בפילוסופיה טענה כזו. יש לה גם שם. ויש שמאמינים בה. ויש מאמרים בנושא.
    2. לא טענתי שטענה זו פותרת את הפרדוקס.
    3. ניכר שאינך מבין מה אני מבין ומה אני לא מבין.

  98. טוב.
    אפשר להגיד עוד כמה דברים, בעניין הפיגור…
    “יש טענה פילוסופית שאין בעולם אובייקטים אבסטרקטיים.” – ניסים, טענה פילוסופית לא פותרת פרדוקס בעולם בדיוק באותה המידה שבה תרגיל במתמטיקה אינו פותר את הפרדוקס בעולם.
    ניכר עליך שאינך מבין את העובדה שאנשים הם אלו שפותרים את הפרדוקסים תוך שימוש בידע, וכלים כמו מתמטיקה או פילוסופיה או ווטאבר….

  99. ישראל שפירא
    שים לב למשפט שאת מצטט: שתי תמונות מאותו מקום באותו זמן. זו בדיוק הבעיה – מה זה אותו זמן במערכות ייחוס שונות?
    אשאל אותך שאלה פשוטה. נסתכל על השעון בחללית נמוכה ומהירה – האם אתה טוען שהשעון בחללית יראה כמו השעון על כדור הארץ? בוא נניח חללית שעוברת מעל לצופה במדוייק כל שעה. האם גם השעון בחללית יתקדם בשעה?
    ורק שתדע, ניסויים שהראו את הפיגור בזמנים נעשו כבר לפני 40 שנה…..

  100. מיכאל רוטשילד
    תרגיע. כל מה שאני אומר זה שתרגיל במתמטיקה לא פותר פרדוקס בעולם. יש טענה פילוסופית שאין בעולם אובייקטים אבסטרקטיים.
    אז אתה אומר שלפילוסופיה אין מקום בעולם. אז כנראה שאין לנו מה לדבר יותר.

  101. ניסים.
    יאוֹש יאשתני יאוּש.

    בפעם האחרונה: איני מתווכח איתך על הנושא “האם יש או אין התארכות זמנים”, אלא על השאלה: האם שני נוסעים החולפים זה על פני זה יראו – או יצלמו – תמונות זהות של מצב השעונים.

    קישרתי אותך לדוגמא רלוונטית, הזהה לדוגמא שהבאתי על החללית החולפת על פני הארץ בדרכה למאדים.

    באותה דוגמא, ג’יל חולפת על פני שעון המסונכרן עם שעונו של ג’ק ונייח יחסית אליו (כמו הארץ ומאדים בדוגמא שלי). שעונה ואותו שעון מראים 0 כאשר היא חולפת על פניו (כמו בדוגמא שלי). כאשר מגיעה ג’יל אל ג’ק, שעונה מראה 8 שניות ושעונו מראה 10 (כמו בדוגמא שלי).

    אילו היה ממש בטיעונך, הרי שג’יל היתה רואה את שעונו של ג’ק כמפגר, דהיינו היא הייתה רואה זמן 10 בשעונה וזמן 8 בשעונו.

    אבל הפלא ופלא! שניהם מסכימים על כך שהם רואים – ועל כן מצלמים -ששעונה מפגר אחרי שעונו!

    איך אם כן טוענת היחסות שכל אחד רואה את שעון השני כנע לאט יותר?

    כאן מופיע במאמר הסבר מלומד, מתמטי בעיקרו (ונכון), על כיצד הם מגיעים למסקנה זו.

    אולם ללא ספק, בשני הצילומים – מצידה ומצידו – שעונה מפגר יחסית לשעונו.

    ורק על זאת שאלתי.

    ליתר ביטחון, אם תמשיך בקריאת המאמר, תראה ש:
    two snapshots taken from the same place at the same time must show the same thing!

    אז אחת מהשתיים: או שתסביר לי מה לא הבנתי במאמר, או שאגיע למסקנה הבלתי נמנעת שהחלטת לשגעני שָגוע.

    דוד – אני מקווה שהבנת מדברי לניסים את הפואנטה. השאלה הייתה מה יראו צילומים מטווח קצר, לא חישובים הכוללים מרחק. התשובה: הם יראו את אותה התמונה.

    רפאים:

    מעולם לא הייתה לי בעיה להודות בטעותי, אם זהיתי אותה. אינני רואה כל טעות בטיעון שלי כאן, וגם לא בחידת מכוניות השעטנז.

  102. ניסים:
    המדענים נקראו בעבר הפילוסופים של הטבע.
    בשלב אינפלציוני מסוים החליטו אנשים נוספים שאינם מבינים במדע שכדאי שגם להם יקראו “פילוסופים”.
    כל השאר – היסטוריה ופוליטיקה.
    יש הרבה אנטישמים וזה לא משכנע אותי בצדקתם.
    יש המון מוסלמים וזה לא משכנע אותי בצדקתם.
    יש המון נימוקים חסרי שחר והם אינם משכנעים אותי בצדקת טועניהם.
    משום מה, אתה מוכן לקבל את ההתכנסות בבעיה מתמטית והיא נראית לך כאבסורדית כאשר נותנים לפרמטרים שלה שמות.
    זו בעיה שהסיבה היחידה לה יכולה להיות דעה קדומה וזה שאתה מתחמק ממענה על שאלותיי מעיד כאלף עדים על מצב של היעדר נכונות לדיון במסקנה.
    על איזו משאלותיי אינך יודע להשיב?
    אם זה על אלו שנוגעות להארטי הרי מדובר במצב בו ניסית לשכנע אותי באמצעות ספר שלא קראת או לא הבנת.
    אם מדובר על עצם איזכורו של הארטי (שלשיטתך אומר שהמתמטיקה אינה נחוצה למדע) רק כדי להגן על מודל מתמטי אחד ולפסול מודל מתמטי אחר הרי שמדובר בדמגוגיה.
    אם מדובר על הסתירה בין אי נכונותך לקבל בטבע מסקנה כלשהי לבין נכונותך לקבל את אותה מסקנה במודל מתמטי שאין כל הבדל בינו לבין תיאור הטבע על ידי מי שהגה את הפרדוקס הרי מדובר בפסילת ההסבר מטעמים שאינם נוגעים לבעיה בה דנים – שאלה שהיושר האינטלקטואלי מחייב אותך לתת לעצמך תשובה עליה.

  103. ישראל,
    ב”כל אחד רואה את שעוני השני כנעים לא יותר” הכוונה היא לשעונים עצמם. ז”א הנוסע בחללית רואה את שעון כדה”א כמתקדם לאט יותר, והצופה בכדה”א רואה את שעון החללית כמתקדם לאט יותר. ז”א, אם נניח שבכדה”א יש טלסקופ שעוקב אחר החללית, רואה את השעה שמוצגת שם, מתחשב במרחק שהאור היה צריך לעבור מהחללית לכדה”א ומשווה את השעה לשעונו הוא יראה את השעון בחללית כמתקתק לאט יותר. ולהפך, אם בחללית יש טלסקופ שמכוון לכדה”א, צופה בשעון שנמצא שם ומתחשב במרחק שהאור היה צריך לעבור עד החללית (המרחק הפעם קצר יותר!) הוא יראה שהשעון בכדור הארץ מתקתק לאט יותר. להכניס את מאדים לסיפור סתם יוצר בלבול.

  104. ניסים: נראה שחוץ מבב”לת, אינך יודע דבר.
    אתה ממשיך להתחמק. אולי תענה על השאלות ששאל אותך מ.ר? מעניין אותנו לדעת היכן הוא טעה, כדבריך. (ומעבר לביקורת עליך- אני באמת מקווה שלא תיקח את זה קשה מדי- יש גם עצה: תהיה קצת כמו ישראל שפירא [ר’ חידת מכוניות השעטנז], הוא לפחות מודה כאשר הוא טועה).

  105. ישראל שפירא
    אני שמח שאתה רואה היגיון כלשהו בבעית הרציפות בעולם האמיתי 🙂

    דוד
    אנחנו חושבים בדיוק אותו דבר. אולי אתה תשכנע את ידידנו ישראל? 🙂

  106. ישראל שפירא
    בקשר לרכבות/חלליות אתה טועה. זה מה שהתיאוריה אומרת – הצגת לך את החישובים: התקצרות המרחק זה בדיוק מה גורם לפיגור בזמנים. מעבר לכך, הסברתי לך שזה גם קורה במציאות.
    ישראל – אנחנו רואים את הפיגור בשעונים במציאות. אין לי מושג למה אתה מתווכח עם עובדות שאתה כנראה לא מכיר.
    בקשר למאדים – התאום המתקרב אלינו מתקצר. אתה מבלבל בין מרחק לאורך – ואסביר. אם נצלם את את כוכבים בזמן החליפה, הם יהיו באותו מרחק. זה נכון כאשר הצופה מכדור הארץ רואה חליפה.
    אבל – לא חשוב מתי נצלם, אנחנו נקבל שהכוכב בתנועה מקווץ. זה די מסובך, כי צילום במהירויות כאלה אינו פשוט בכלל. בוא נניח שאנחנו מפעילים הבזק אור מכל מיני נקודות על הכוכבים ברגע החליפה. אבל, אנחנו ניתקלים בבעיית החוסר בו-זמניות במערכות בתנועה.

    בוא נפשט – נפעיל 2 הבזקים בלבד, בשני קצוות הכוכב הנע ברגע מסויים. האם הצופה הארצי יראה אותם בו זמנית? מה אתה אומר?

  107. מיכאל רוטשילד
    לי מפריעה המחשבה שאפשר לסיים סידרה אינסופית של ארועים. אם אתה לא רואה את הבעיה אז אתה לא רואה את הבעיה. לא באתי לשכנע אף אחד.
    זה שפילוסופים מכובדים רואים בזה בעיה לפחות מראה שהנושא לא פתור כמו שאתה אומר בכזה בטחון.

    ושוב – הצגת שאלה שאני לא יודע עליה את התשובה. מה אתה מנסה להסיק מזה???

  108. ניסים:
    זה שיש בתחום הפילוסופיה עוד אנשים שלא מבינים את הבעיה ואת הפתרון לא ישכנע אותי.
    נימוק לכך שזה לא עובד לא הבאת וגם על השאלות האחרות שהצגתי לך לא ענית.
    לא נראה לי שאתה מתדיין ותגובותיך נראות לי כהתעקשות גרידא.

  109. ניסים
    אתה מכריח אותי לטרוח – ועוד בשבת! קח את זה על מצפונך.

    הנה מה שהשבתי לך בעבר על שאלתך, שים לב לתאריך:

    לשאלתך: כל הצילומים יראו זמן זהה בשעוני 2 הרכבות המחושב עיי טרנספורמציית לורנץ הרלוונטית.

    אנא הסבר את בעייתיות פרדוקס זנון.

    אפריל 24th, 2013
    תגובות ישנות

    זה שהחלפת רכבות בחלליות, לא משנה את מהות השאלה.

    לגבי החלק השני של שאלתך, אכן כל חללית רואה את השניה כנעה בערך ב0.97C, אך המרחק ביניהם מתקצר עוד יותר (שוב לורנץ), מה שמפצה על 3 הדקות הנעלמות.

    אם התיאור האיכותי אינו מספיק, אמרתי לך אתמול שאם תתעקש אערוך עבורך את החישוב הכמותי (אתה מתעקש? שבת, מצפון, עמאיית….).

    האם הכל ברור גם לי? לא בדיוק. בזמנו שאלתי פה ללא מענה, את השאלה הבאה:

    נאמר שתואם מאדים חולף על פני מאדים במהירות גבוהה בכיוון הארץ. אנו מצלמים מהארץ את רגע החליפה.

    עכשיו, על פי לורנץ, המרחק מתקצר רק בכיוון התנועה. לכן נקבל בתמונה שקוטרו של התואם זהה לזה של מאדים, אך מרחקו קצר בהרבה.

    מה נראה בתמונה? האם בגלל הפרספקטיבה יראה התואם גדול בהרבה ממאדים? האם הוא יסתיר אותו בגלל גודלו העצום?

    לא קבלתי תשובה. אולי בהזדמנות זו שמייקל קפץ לביקור, נשמע את דעתו.

    אגב – אם שמת לב כולנו אומרים מ*כאל, או מייקל, או מקל חובלים. רק לא את השם המפורש. זה כדי שלא ימתינו אותנו.

    ובעניין הדיון שלך עם מייקל – מבין ומזדהה איתך. גם אני תמהתי לא פעם מדוע האינטגרל, עצם מתמטי הבנוי על אינפיניטסימל השואף ל0, חייב לעבוד גם על עצמים פיזיקלים שאינם שואפים ל0 אלא קוואנטיים במהותם ולא קטנים מעבר לגבול מסויים של גודל פלאנק (זמן הוא דוגמא יפה).

    דוד – כמובן שהתשובה שלך היא הנכונה. הבעיה היא: מה הכוונה ב”כל אחד רואה את שעוני השני כנעים לאט יותר”. אם אינך מסכים שזה מבלבל ביותר, ראה את תשובתו של ניסים – ועוד רבים וטובים – שמאמינים שמשמעות הדבר הוא כפשוטו: צילומים שנעשו באותה נקודה ובאותו הרגע של אותו אירוע, יראו דברים שונים.

    מייקל – באילו פטנטים מדובר?

  110. ישראל,

    ברשותך אצטרף לדיון. בתשובה לחידתך על הארץ ומאדים. שאלת:
    כאשר תחלוף החללית על פני מאדים, מה יראו הצילומים של שני השעונים במצלמות ממאדים ומהחללית?
    תשובתי היא שקודם כל שניי הצילומים יהיו זהים, והצילומים יראו את שעון החללית מפגר אחרי שעון מאדים. ז”א שעון מאדים מראה שעתיים ושעון החללית יראה זמן קצר יותר. לא נוצרת סתירה משום שמבחינת החללית שעוני כדור הארץ ומאדים אינם מסונכרנים.

  111. מכאל רוטשילד
    אמרתי שהפתרון של תרגיל מתמטי הוא גבול. אני לא חושב שיש “גבולות” בעולם האמיתי.
    רק שתדע – בתחום הפילוסופיה, הפרדוקס על אכילס והצב אינו נחשב לפתור. אני מציע לך לקרוא ב SEP וללמוד שהמצב לא בדיוק כפי שאתה כותב.

    ואני לא מתחמק משאלתך. קרא את הספר…….

  112. ניסים:
    אני מסכים לא להסכים.
    אני נוהג לא להסכים עם טעויות ולכן זה קל לי.
    להגיד שהפתרון הוא גבול זה לא לומר כלום אבל אם אמירה זו מספקת אותך כפתרון לבעיה המתמטית אז מדוע אותו פתרון לאותה בעיה – כשההבדל היחיד הוא ש t הוא הזמן ושתי הפונקציות הן מיקומיהם של אכילס והצב – אינו מספק אותך?
    אני חוזר ואומר: אינני טוען שהזמן הוא לא בדיד. אין לנו כרגע מידע על הנושא וה”פרדוקס” הנדון לא מוסיף לנו כל מידע על הנושא, כך שדוגמת החשמל מתפרצת לדלת פתוחה בבניין אחר. השאלה כאן היא האם במודל הפיזיקלי שעל פיו הזמן הוא רציף יש כאן פרדוקס והתשובה היא שלא – גם אם הזמן הוא רציף אין כאן כל פרדוקס. יש כאן בסך הכל ניסיון – מוטעה מבחינה לוגית – להסיק על כל ציר הזמן מסקנה המבוססת רק על אירועים שכולם מתרחשים לפני השעה 10:00.
    אני יכול להצביע לך על מישהו שכתב שהעולם ייחרב ב 2012 כך שלהצביע על מישהו שכתב משהו זה באמת לא טיעון (אגב – יש איזה רבי שטוען טענה דומה לגבי הערב!!).
    אשמח אם לא תתחמק משאלותי ותסביר לי איך אותו הארטי מתאר את מסלול כדור הארץ סביב השמש ואיך הוא מציע לקבוע מתי והיכן בדיוק יתנגשו שני גופים הנעים זה לקראת זה.

  113. מכאל רוטשילד
    אז בוא נסכים לא להסכים ….
    הדוגמא שלך היא תרגילבמתמטיקה. הפתרון הוא הגבול כמובן. ההפרש בין 2 הפונקציות שואף ל-0.
    הנה עוד דוגמא – מתחום החשמל. זרם הוא נגזרת של המטען ליחידת זמן. היום אנחנו יודעים שהזמן דיסקריטי. פעם לא ידעו את זה ….

    אופס … הארטרי פילד. הוא כתב ספר שנקרא Science without Numbers.

  114. אגב, חיפשתי את הארטי פילד בויקיפדיה ולא מצאתי אותו.
    אינני יודע מיהו ואינני מבין מה עשה.
    איך הוא יכול היה לציין מהו מסלול כדור הארץ סביב השמש ללא שימוש במונח המתמטי “אליפסה”?
    איך הוא הציע לקבוע את הזמן בו שני אובייקטים שנעים זה לקראת זה יתנגשו ללא מתמטיקה?
    כל הסיפור נראה לי מוזר ביותר.

  115. ניסים:
    קודם כל – אם אתה חושב שאפשר להתייחס לעולם הפיזיקלי ללא מתמטיקה אז אתה פחות או יותר היחיד בעולם שחושב כך.
    המצב האמיתי כל כך רחוק ממה שאתה מתאר שיש אפילו אנשים שחושבים שהעולם איננו אלא מתימטיקה ואתה מוזמן לקרוא על כך כאן:
    https://www.hayadan.org.il/ground-problem-of-metaphysics-part-2-0704103/

    המצחיק הוא שגם המודל שאתה מציע הוא מודל מתמטי ככל המודלים. פשוט – אינך מבין את הפתרון במודל שעליו מדברים אז אתה אומר – נעבור למודל אחר.
    התחמקות כבר אמרנו?
    אנסה להבהיר לך את האבסורד בהלך המחשבה שלך על ידי דוגמה שהיא מתמטית נטו.
    יש בה רק פונקציות פרמטריות והדמיון באותיות אינו מקרי.
    במשל המתמטי – t הוא רק מספר שמשמש כפרמטר.
    הפונקציה ( T(t היא סתם פונקציה באותו מרחב מתמטי רציף וכך גם הפונקציה ( A(t אלו סתם שתי פונקציות.
    נניח ש T(t)=10+t
    נניח ש A(t)=10t
    ה”פרדוקס” יוצג בצורה הבאה:
    לא יתכן ש ( A(t יהיה גדול מ ( T(t כי אם נתחיל t=0 אז T(t)=10 ו A(t)=0
    אם נגדיל את t ל 1 אז ( A(t אמנם יהיה שווה ל 10 אבל ( T(t כבר יגדל ל 11

    וכן הלאה – כמו באכילס והצב – רק לא באכילס ולא בצב ולא בעולם אלא רק במרחב רציף דמיוני לחלוטין.
    מסקנתך כאן תהיה: טוב, נו, דיברתם על מרחב רציף אבל שיקרתם.
    הייתם צריכים לדבר על מרחב בדיד כי שם אני מבין את הפתרון והוא נראה לי הרבה יותר מעניין.

  116. מיכאל רוטשילד
    אני חושב שכן אפשר לדבר על העולם הפיסיקאלי ללא מתמטיקה. אכילס והצב זו דוגמא פשוטה לכך. לפני מספר שנים הארטי פילד פיתח את המכאניקה של ניוטון ללא מתמטיקה.
    ולי אישית יש בעיה עם סידרה אינסופית של “ארועים”. אני מבין ששימוש במילה “ארוע” כאן הוא בעייתי, כי לא ברור שתנועה היא בכלל קבוצה של ארועים.
    בקיצור – הפתרון הדיסקריטי נראה לי מעניין יותר….

  117. ניסים:
    זו אינה בעיה פיזיקאלית אלא בעיה עם מודל של הפיזיקה.
    מודל הוא מתמטי (תמיד! הרי אין לך כל אפשרות לדבר ישירות על העולם. אתה תמיד מדבר על מודלים).
    השאלה היא אם יש לבעיה במודל פתרון (ויש לה פתרון) ולא אם יש למציאות מודלים אחרים.
    לפעמים – כשלבעיה אין פתרון במודל קיים, מהווה הדבר אינדיקציה לכך שהמודל אינו מייצג את המציאות ויש להחליפו. לא זה המצב בשאלה זו.
    במודל המתמטי אני אכן מניח זמן רציף (כפי שהניחו מנסחי הבעיה) וזה פשוט כן עובד.
    אינני מבטל את רעיון הזמן הבדיד ואתה מוזמן לקדמו ולהביא לו ראיות כאוות נפשך אבל על תעשה זאת על חשבון הבנת הבעיה הנוכחית ופתרונה.

  118. מיכאל רוטשילד
    לדעתי, “פתרון” מתמטי לבעיה פיסיקאלית זו ההתחמקות. בפתרון המתמטי אתה מניח שהזמן רציף, וזה פשוט לא עובד, לדעתי. אתה עדיין צריך לעבור אינסוף ארועים.
    מצד שני, אני לא מבין למה אתה רואה בעיה במושג של זמן דיסקריטי. זה לא נוגד תצפיות וזה כן פותר לפחות בעיה אחת. למה לבטל את הרעיון בלי סיבה?

  119. מייקל

    עברתי קצת על הבלוג שלך.

    שאלה: האם היה לך בעברך רקע דתי או מסורתי?

    טוב, אני חייב ללכת לעבודה. רווחתם של הורי אמנם מובטחת בינתיים בקיבוץ, אבל אנו בני התמותה ללא הפטנטים חייבים להתפרנס.

  120. ישראל:
    לשמחתי – אני תמיד בחופש.
    למעשה – בחופש חייתי מאז ומתמיד אבל מזה כמה שנים אני גם בחופשה אחרי שמכרתי מספר פטנטים ששחררו אותי מעול הפרנסה.
    הבעיה היא שנושאים אחרים טרדו את מנוחתי (וחלקם ממשיך לטרוד אותה עד היום).
    זה התחיל בהשקעת זמן רב בנסיונות להחזיר את המדינה לשפיות במסגרת הפעילות במפלגת אור וזה המשיך בהשקעת הזמן הרבה שעלי להשקיע ברווחתו של אבי הקשיש בעקבות פטירתה של אמי.
    זה גם עניין של סדרי עדיפויות. אחד הדברים שעשיתי לאחרונה זה לכתוב כמה דברים בבלוג שפתחתי באתר “אחד נגד כל הדת”.
    שם הבלוג הוא “מעגלתו של חילוני” ואני ממליץ לכולם לעיין בו.

  121. ניסים
    עלינו להבחין בין שני נושאים: נושא הדיון, דהיינו האם שני צופים החולפים זה על פני זה יסכימו ביניהם על תמונת מצב אחת ויחידה, ונושא חישוב הזמן על פי היחסות.

    בנושא הראשון אני מאמין שהצלחתי להראות לך שאין בינהם ויכוח. בלינק שהבאתי מצויין בפירוש ששניהם מסכימים ששעונה של ג’יל מפגר אחרי שעונו של ג’ק.

    הנושא השני – החישוב עצמו – הוא עניין טכני. אם תתעקש, אשתדל לערוך את כל החישובים הרלוונטים, אבל אני מאמין, וגם אתה מאמין, שלא נקבל שום סתירה עם היחסות.

    מייקל

    עזוב שטויות עכשיו, יחסות, שמחסות, דמי חסות. ספר איך היה בחופש.

  122. ניסים:
    ואולי בכלל לא ייתכן עולם? אני חוזר ואומר – זו אינה אלא התחמקות מן הבעיה. זה כמו לענות לבעיית תנועה העוסקת באדם שיוצא מעיר A לכיוון עיר B את התשובה שבכלל אין ערים בשמות הללו.
    אין כל פסול בתיאור מתמטי של עולם בו הזמן רציף.
    אני חוזר ואומר – אין לשאלה זו כל קשר לטיב הזמן בעולמנו.

  123. מיכאל רוטשילד
    את החישובים אני מכיר לעומק 🙂 לא טענתי לרגע משהו שונה. מה שטענתי זה שתופעות האלה ניצפות. זה לא חישובים ללא יכולת אישוש, כמו שטוען ישראל שפירא (להבנתי את דבריו).

  124. מיכאל רוטשילד
    אולי בכלל לא ייתכן עולם שבו הזמן רציף? אני לא חושב שהרעיון עד כדי כך טיפשי. אנחנו יודעים שאי אפשר לקבוע בו זמנית גם מהירות וגם מיקום בדיוק גבוה. אז – כשאכילס קרוב מאוד לצב אנחנו כבר לא בעולם הקלאסי.

  125. ניסים:
    אמנם המהירות מאיטה את מהלך הזמן בשעוני הלווינים ב 7 מיקרושניות, אבל הכבידה הנמוכה דווקא מאיצה את הזמן בהרבה יותר (45 מיקרושניות ביום) ולכן הזמן בלווינים מתקדם דווקא מהר יותר (ב 38 מיקרושניות ביום).
    תוכל למצוא את החישובים כאן:
    http://www.dtic.mil/cgi-bin/GetTRDoc?Location=U2&doc=GetTRDoc.pdf&AD=ADA516975

    ישראל:
    התווכחתי אתך על הנושא די והותר אז לא אחזור לוויכוחים אלה שוב.

  126. ישראל שפירא
    אבל שים לב…. בוא נניח עכשיו 2 חלליות. אחת יוצאת מכדור הארץ והשניה יוצאת מהכוכב במרחק 40 דקות אור. מהירות כל אחת – נניח 0.8c. הם יוצאים לדרך יחד וניפגשים אחרי 25 דקות (לפי שעוני הכוכבים). איזה שעה רואה צופה בחללית? הוא בחצי הדרך ולכן אצלו עברו 15 דקות.
    אבל מה הצופה יראה בחללית השניה? המהירות היחסית היא בערך 0.97c .בוא נניח שזה c – טעות קטנה. המרחק – 12 דקות אור, כלומר – השעון בחללית השניה יפגר ב-3 דקות.
    זה כמובן נכון לשני הצדדים.
    אתה רואה כאן טעות?

  127. ניסים:
    תשובתך אינה נכונה כי היא מתייחסת למציאות אחרת מזו שהשאלה מניחה.
    זו אינה אלא התחמקות מן המציאות הנדונה בשאלה.
    גם אם הזמן מורכב מיחידות בדידות (ובינתיים אין לנו כל עדות לכך) הרי שהשאלה יכולה להישאל כשאלה היפותטית על עולם בו הזמן רציף והתייחסות אמיתית לשאלה צריכה, לכן, להיות מסוגלת להתמודד גם עם עולם כזה (מה גם, ששוב, כאמור, אין כרגע שום אינדיקציה לכך שעולמנו איננו כזה).

  128. ישראל שפירא
    שוב – זה לא המצב שתיארנו. המצב הזה, שבציור, הוא בדיוק כמו הדוגמא שתיארתי בהתחלה. הדוגמא הייתה של טיסה לכוכב במרחק 40 דקות אור, במהירות 0.8c – לגבי צופים על כדור הארץ ועל הכוכב יעברו 50 דקות, אבל לגבי נוסע בחללית יעברו רק 30 דקות.
    כלומר – אנחנו אומרים אותו דבר – במקרה הזה

  129. ניסים

    אפילו לשניה אחת לא היה לי ספק שתגיד שזה מה שאתה אומר.

    שמעתי שלפי שפינוזה, שלילי זה חיובי, מעגל הוא ריבוע והעקוב הוא מישור.

    שים לב: דיברנו על צילומים. הסתכל מתחת לתמונה השניה וקרא מה כתוב (להלן תרגום):
    כאשר ג׳יל חולפת על פני ג׳ק שניהם רואים ששעונו מראה 10 ושעונה 8 שניות.

    ההסבר מדוע זה לא כך מגיע אחר כך, והוא מתמטי. אך אין ספק ששניהם רואים – או מצלמים – ששעונה מפגר יחסית לשלו.

    מ.ש.ל.

  130. ישראל שפירא
    הדוגמא שהפניית אליו מדבר על משהו שונה לחלוטין – 2 צופים מסתכלים על אותו שעון. להבנתי, דיברנו שכל צופה מסתכל על שעון אצל השני.

    בוא נקרא ביחד את הפסקה הראשונה:
    The object of this exercise is to show explicitly how it is possible for two observers in inertial frames moving relative to each other at a relativistic speed to each see the other’s clocks as running slow and as being unsynchronized, and yet if they both look at the same clock at the same time from the same place (which may be far from the clock), they will agree on what time it shows!

    הכתבה אומרת בדיוק מה שאני אומר.

  131. ישראל שפירא
    לא. זאת לא הטענה שלי. הטענה שלי היא שבכל מהירות יש פיגור בזמן, אבל פיגור כזה קטן ביותר ב-20 קמ”ש, וגדול ב-200,000 ק”מ לשנייה. בחיי יום יום יכול להיות שזה נכון, תלוי בעניין שלנו.
    זה לא המקום היחיד שזה קורה. בחיי יום יום מהירות הסיבוב של כדור הארץ חסר משמעות. כשמשחקים בייסבול אין לזה שום משמשעות. כשמטוס זורק פצצה לטווח של מספר קילומטר יש לזה השפעה קטנה אך לא זניחה וצריך לתקן לסיבוב זה. וכשמדברים על מזג אוויר אז יש לזה השפעה מכרעת – זה מה שגורם להוריקאנים וגם לשינוי מהירות הרוח כתלות בגובה (הרוח מסתובבת ימינה כשמטוס מטפס לגובה).
    כך גם לגבי זמן. המקום היחיד שיצא לי להתעסק עם זה, זה בשעונים אטומיים בלווינים, במיוחד לוויני GPS. לווינים אלה נעים בסביבות 4 ק”מ לשנייה, וזה נותן כ-7 מיקרושניה ביום. לא זניח בכלל.

  132. ניסים

    אז לטענתך במהירות של 20 קמ״ש הצילומים יהיו זהים מבחינת הזמנים בשעונים אך במהירות של 200,000 קמ/שניה הם יתהפכו?
    זה מחלק את המהירויות לשני תחומים: תחום המהירויות הנמוכות שבהם הצילומים זהים, ותחום המהירויות הגבוהות שבה הם מתהפכים.
    התוכל – אתה, איינשטיין, או כל מי שבסדר גודל שלכם, להצביע על נקודת או איזור התיחום, שמתחתיה נוכל להגיד: ממהירות זו ומטה הצילומים זהים, מעליה הם מתהפכים?

    הכנע ניסים. זו אינה התשובה. היא מובאת בדוגמא הבאה:
    http://galileoandeinstein.physics.virginia.edu/lectures/time_dil.html

    שים לב שבדוגמא זו, הזהה לשלי, אין ויכוח על מצב השעונים בין שני הצדדים, שניהם מסכימים ששעון אחד מפגר. הטענה שכל מערכת מפגרת יחסית לשניה מתבצעת באמצעות חישוב, לא צפיה או צילום. הצילום תמיד יראה אותו הדבר. זו בדיוק הטענה שלך על ההבדל בין המהירות האמיתית, הפיזיקלית, לבין המהירות המתמטית.

    מייקל, ברוך שובך. אור אולי לא חוללה גדולות ונצורות, אך נראה שלפיד מיישם את משנתה באדיקות. אז אולי הגיע הזמן לחזור לדברים האמיתיים: ויכוחים פילוסופיים אינסופיים על מהות התנועה והבורא?

    וכולנו גם מתגעגעים לחידות, רצוי בפיזיקה.

  133. אחד אחר
    אתה צודק שמספר הנקודות הוא אינסוף שאינו בר מנייה (עוצמת הרצף). אבל – החלוקה החוזרת לשניים של המרחק יוצר קבוצה שהיא כן בת מנייה. הרי אנחנו יודעים לחשב את מיקומו של כל נקודה.
    מה שאני מציע כפתרון (אני מניח שזה לא במקור שלי אבל אני לא מכיר טענה כזו) זה שהקבוצה היא פשוט סופית…

  134. מיכאל
    זה יכול להסביר את ההבדל ….אז אני מבין מדברייך שאכן ניתן להגדיר רשימה של אינסוף ארועים ולעבור את כולן. אני חושב שהפתרון הוא אחר – הרשימה פשוט סופית. אני לא רואה למה טעות לחשוב ככה…

  135. ניסים:
    הפתרון שלך שונה משלי כי הפתרון שלך שגוי.
    בין היתר – שגויה בו הטענה ש”אין סוף” פירושו “לעולם לא נגיע לסוף”. זו טעות מכיוון ש”אין סוף” מדבר על מספר הרגעים ולא על סכומם וזה בדיוק מה שהסברתי.

    אחד אחר:
    אין לעניין כל קשר לנושא עוצמת קבוצת האירועים (בת מנייה או לא). קרא מה שכתבתי.

  136. ניסים- איך שאני מבין את זה – אכילס צריך לעבור אינסוף בלתי ניתן למניה של נקודות במרחב- אבל יש לו אינסוף בלתי ניתן למניה של נקודות בזמן לעשות את זה. לכן אכילס משיג את הצב.
    זה הפרדוקס שעושה תרגיל מתמטי ומחלק משהו סופי לחלוטין לסידרה מתכנסת אינסופית.

  137. מכאל
    קראתי את תגובתך ואני מסכים בהחלט על הדרך שצריך לפתור פרדוקס. זו בדיוק הטענה שלי – צריך למצוא את הטעות בטיעונים. הפתרון שלי שונה.

  138. ניסים:
    אין שום בעיה בכך שאכילס צריך לעבור דרך אינסוף נקודות.
    גם בלי הפרדוקס הזה כל צעד של כל אדם עובר דרך אינסוף נקודות.

  139. אני מסכים אם מה שאתה כותב. אבל, לדעתי זה לא פותר את הפרדוקס. עזוב את נושא הצעדים. אם הזמן והמרחב רציפים, אזי יש טור של אינסוף נקודות שאכילס צריך לעבור דרכן.
    אתה רוצה להגדיר את המרחק מהגבול? אני רוצה לעבור את הגבול …..
    אתה צודק שאין משמעות לאיזה רגל. זה רק מחדד את הבעיה.

  140. לא לגמרי ברור לי על מה הויכוח כאן
    אבל מהירות היא תמיד משהו יחסי. (גם מהירות האור רק שכאן זה נהיה מסובך והיחסות נכנסת)
    בקשר לתאוצה- בעיקרון אפשר להגיד שאם מישהו נמצא בחללית ומרגיש כוח כובד של 10 מטרים לשנייה בריבוע
    אין לו דרך לדעת לפי הכוח הפועל עליו בלבד אם פועל עליו כוח כבידה של 10 מטר לשנייה בריבוע או שהחללית מאיצה.

    תנע ואנרגיה נשמרים במערכת שלהם הם לא נשמרים בכל המערכות- לאותו עצם יהיו תנע ואנרגיה שונים בכל מערכת יחוס.

  141. ניסים:
    אתה אינך טועה. אם אכילס אמנם ירוץ בצעדים שתיארת הוא לא ישיג את הצב. הוא רק יכול להגיע קרוב אל הצב בדרגה כלשהיא שתיקבע באופן שרירותי.
    להלן מספר שגיאות נפוצות:
    1)נהוג לומר אכילס “לעולם” לא ישיג את הצב. זו אמירה שכוללת בתוכה הנחה שגויה שקשורה לזמן. קיים זמן שבו אכילס ישיג את הצב. אבל בתנאים של הבעיה שתוארה. אכילס והצב לא יגיעו לנקודת זמן זו.
    לתיקון השגיאה יש לומר: אכילס לא ישיג את הצב במספר צעדים סופי כלשהוא. תיקון זה אינו מתיחס לזמן אלא למספר הצעדים של אכילס.
    2)בפרדוקס מדובר על “אכילס” שהוא לא בנאדם. אכילס בתיאור המתמטי אינו קיים בטבע. אין אפשרות לבצע משימה כזו כי מספר הצעדים לשניה מוגבל אצל יצורים ביולוגיים.
    3)אין משמעות לשאלה באיזה רגל יגיע אכילס המתמטי אל הגבול. זאת משום שאכילס המתמטי לא יגיע אל הגבול. יש רק איזור מסויים של הגבול שאכילס המתמטי יגיע אליו לאחר מספר מסויים של צעדים.
    רק לאחר שיוגדר המרחק מן הגבול ניתן יהיה לחשב את מספר הצעדים. לפי זה ניתן לומר שאכילס יגיע אל הצב בשתי הרגליים. משום שבכל מרחק שנגדיר , קטן ככל שיהיה, אכילס ידרוך שם בשתי רגליו. במילים אחרות המרחק בין שתי הרגליים שואף לאפס.

  142. יובל חייקין
    אני עדיין לא רואה את הטעות שלי בפרדוקס אכילס והצב. רגע הגעתו של אכילס לצב בא אחרי סידרה אינסופית של רגעים. סכום הרגעים אמנם סופי, אבל בכל זאת משהו לא מסתדר בראש. אלא אם כן עושים קוואנטיזציה לזמן, או מערבבים כאן את אי הודאות…..

  143. ישראל שפירא
    אני לא מבין מה העניין עם רכבות ב 20 קמ”ש. הנה דוגמא אחרת: בשביל לבנות בית על מגרש של דונם כדור הארץ ייחשב כשטוח. בשביל לבנות גשר ארוך צריך להתחשב בזה שכדור הארץ עגול. בשביל לנווט למרחקים של מאוד ק”מ כדור הארץ הוא כבר אליפסואיד. בשביל לחשב מיקום של לווין GPS כדור הארץ הוא גיאויד.
    זה אותו דבר לגבי מהירויות יחסיות.

    הנקודה שלך שמהירויות ניתנות לטיפול רק בדרך המתטמטיקה אינה נכונה,לדעתי. מתמטיקה היא כלי שבני אדם המציאו. ווילארד קוואין טען, והילארי פאטנאם טוען שמתמטיקה אכן חיונית להבנת העולם הנצפה. הארטי פילד למשל, טוען להיפך, וכך גם אני.

    ישראל – תיארת נכון את הגישה שלי. נראה לי שאתה צודק שמהירות יחסית היא אכן מושג אמיתי, והוא גם שונה מתנע: מהירות יחסית משפיעה על תצפיות (ורק על תצפיות) ותנע נשמר, גם במהירויות רלטוויסטיות. ושים לב למשהו מעניין: המהירות שהיא קבועה בכל מערכת ייחוס אינה בדיוק מהירות האור, אלא מהירות העברת מידע, כלומר המהירות של תצפיות…..ואל תשכח שלא רק אור נע במהירות האור ……
    אבל!!! אין לזה קשר למתמטיקה.

  144. אכן כך – תנע הוא מה שנשמר.
    מהירות היא תכונה של החץ כמו שמיקום היא תכונה של החץ- זה שיש בינהם קשר מתמטי שאחד הוא נגזרת של השני והשני הוא אינטגרל של הראשון- זה מאוד מהותי להבנה ולחישוב- אבל זה לא מה שמגדיר אותה- נגזרת מגדירה מהירות רגעית בעוד שאפשר להגדיר מהירות גם כמהירות ממוצעת- נגזרת זאת רק דרך למדוד את המהירות-
    אפשר להגדיל את המהירות הרגעית כהמרחק שהעצם יעשה בשנייה אם בדיוק באותו רגע כל הכוחות שפועלים על העצם יחדלו לפעול.

  145. אינעל דרבקום האייפון הזה, שנחזור כבר למחשב נורמלי, שאפשר גם לראות מה כותבים לפני ששולחים.

  146. גם מתיישב וגם מתחשב.

    אם הבנתי נכון, ניסים מנסה להראות שהניסיון שלנו לטפל במהירות – מהות פיזיקלית – באמצעות המתמטיקה מוביל לפרדוקסים, בגלל הבעייתיות של הגדרת המהירות בנקודה אפס.

    זו בעיה ידועה באינפי. כל הרעיון של נגזרת היא שאתה מגדיר אותה כחלוקה באינפיניסטימל השואף לאפס – בשעה שהחלוקה באפס אסורה לפי המתמטיקה.

    ניסים שואל ( כך הבנתי): מה הרלוונטיות של אותו טיפול מתמטי רעוע – המהירות, שהיא נגזרת מרחק/זמן לפי המתמטיקה, אך עומדת בפני עצמה בעולם הפיזיקה?

    ועל כך השבתי: בעולם הפיזיקה אין מהירות, רק מהירות יחסית. אין לכך קשר ליחידות si. למעשה, תוכל להגדיר מהירות כגודל בסיסי, ואת הזמן כחלוקה מרחק במהירות.

    ומשום כך, אל לנו להטריד את מוחנו הקטן והעסוק בטבעה הפיזי של המהירות או התנועה האינרציאלי בנקודת האפס. היא פשוט לא קיימת.

    מהירות יחסית לעומת זאת קיימת גם קיימת, והמתמטיקה היא הכלי הנכון לטיפול בה.

    מהירות האור לעומת זאת אינה גודל יחסי, וכמוה התאוצה. שתיהן מוחלטות.

    אם אינך מאמין, כנס לאוטו ושים גז. נסה לשכנע את גבך הדואב שיחסית למכונית המאיצה בדיוק כמוך אתה בעצם במנוחה. ע״ע מכוניות שעטנז.

  147. ישראל, קצת סדר:
    העתקתי וגם הדבקתי מויקיפדיה:
    במערכת SI ישנם שבעה גדלים פיזיקליים בסיסיים, שנמדדים באחת משבע היחידות הבסיסיות (בסוגריים): אורך (מטר) , מסה (קילוגרם), זמן (שנייה), זרם חשמלי (אמפר), טמפרטורה (קלווין), כמות חומר (מול) ועוצמת הארה (קנדלה).
    ישנם גדלים פיזיקליים רבים, שאינם בסיסיים: שטח, תאוצה, קיבול ועוד, גדלים אלה נגזרים מהגדלים הבסיסיים. כך למשל מדידת שטחים היא הכפלת אורך באורך ולכן אינה בסיסית. בהתאם, יחידות המידה למדידת שטחים אינן יחידות בסיסיות והן נגזרות מיחידות המידה של אורך.

    אם האמור לעיל מקובל עליך, איך זה מתיישב עם תוכן תגובתך האחרונה?

    ובעניין הרומן שלי עם ניסים: נגמר בטרם החל 🙁 אבל לא נורא. אנ’לא לוקח ל♥

  148. ניסים, לתגובתך זו:
    https://www.hayadan.org.il/new-crew-aboard-station-after-express-flight-010413/comment-page-7/#comment-406932
    א: יש שתי תורות יחסות, הפרטית והכללית. בבקשה על תערבב ביניהן. אנחנו מתעסקים כאן רק בתה”י הפרטית. הדוגמאות שהבאת הן מתה”י הכללית.
    ב: במונח “שמרנות” התכוונתי לגישה של פרמנידס. בעיני, זנון הוא זה שמייצג גישה מהפכנית שתוצאותיה הן חישובי גבולות והתכנסויות, “המצאת” מספרים שהאינטואיציה שלנו אינה מכירה ועוד פיתוחים חדישים.
    ג: כבודן של טענות פילוסופיות במקומו מונח. אך למען הסדר הטוב גם אותן יש להביא בקוהרנטיות וללא נפנופי ידיים.

    חוצמזה, בלי כוונה להעליב, אני כבר לא מבקש ממך שתכתוב אלי.

  149. ניסים
    בפיזיקה אין דבר כזה מהירות – יש רק מהירות יחסית, מהירות האור ותאוצה.

    כל גוף שאינו בתאוצה נמצא במהירות 0 מבחינתו. לכן הדרך היחידה לטפל במהירויות היא באמצעות המתמטיקה, ומכאן הסדרות המתכנסות.

  150. אותן הדבר? חשבתי שאתה טוען שכל אחד יראה את שעוני השני כנעים לאט יותר..

    כי אם זה אותו הדבר, אז שניהם מסכימים שחיפה מפגרת אחרי תל אביב, לא?

  151. יובל חייקין
    תיארתי טיפה את נושא לוויני ה-GPS. גם הזכרתי את נושא משיכת זוגות תייל נושאות זרם. אלה שתי דוגמאות מתחום ההנדסה שמאששות את תורות היחסות – הן מבחינת פיגור בזמן והן מבחינת התכווצות באורך. אמפירי לחלוטין 🙂

    אני לא כל כך מבין למה אתה אומר שהתפיסה שלי שמרנית. כל ילד בתיכון לומד את הפתרון הקלאסי לפרדוקס אכילס והצב – סכום סידרה אינסופית מתכנסת לתוצאה סופית, וגם קל לחשב סכום זה.
    אני הבאתי טענה פילוסופית: הנחות יסוד והסקת מסקנה. אם המסקנה לא הגיונית אז איפה השגיאה? בהנחות היסוד או בדרך הסקת המסקנה? יש פתרונות אפשריים לפרדוקס, אבל לזרוק “זה סכום טור גיאומטרי” לא עובד.

  152. ניסים

    אתה מרבה לדבר על סימטריה, אבל שים לב לחוסר הסימטריה בינינו: אני עונה על כל שאלותיך, אתה רק מתפלסף ואינך עונה בבהירות.

    אז הנה אציג לפניך את השאלה פעם נוספת. אם תענה בצורה ברורה, יש טעם לדיון. אם לא, עדיף אולי שנסיים.

    בואו ניקח את דוגמת הרכבת לחיפה שחולפת על פני הרכבת לתל אביב. המהירות היחסית ביניהן – 20 קמ״ש. ברגע שהקטר חולף על פני המאסף הזמן בשעוני הרכבת לחיפה – 1442. בשעוני הרכבת לתל אביב – 1753.

    הזמן מוצג על חלון הקרון והקטר כתצוגת לד דיגיטלית, כך שאין בעיה לצלם את הזמן בשתי הרכבות ביחד ממצלמה אחת מכל רכבת.

    שאלה:

    האם גם במקרה זה, שיכול לקרות כל יום ואין לו קשר ליחסות, יראו הצילומים זמנים שונים בשעונים? האם הצילום מרכבת חיפה יראה: חיפה 1442 תל אביב 1753, ואילו הצילום מרכבת תל אביב יראה: חיפה 1753 תל אביב 1442?

    במידה ותשובתכם היא חיובית, ראו הוזהרתם: אין כל בעיה לבצע ניסוי פשוט כזה עם מכוניות, שעונים ומצלמות. אני מוכן להתערב עם כל מי שמעוניין על כל סכום שירצה שהצילומים יראו את אותם הזמנים בשעונים, לא משנה מאיפה צולמה התמונה.

    מישהו רוצה להתערב?

  153. אחד אחר
    יש בעיה של הגדרת מהירות בנקודה. אם אנחנו מסתכלים על המהירות כנגזרת, אז המהירות היא גבול והיא מוגדרת אך ורק לפרק זמן ששונה מאפס.

    מתמטיקה היא כלי נפלא לתאר את הטבע. אבל היא רק כלי. חוקי מתמטיקה אינם חוקי הטבע.

    וכשאני למדתי פיסיקה, מה שהיה באמת בסיסי זה דווקא תנע. הרי אין חוקי שמירת מהירות, נכון?

  154. ישראל שפירא
    כתבת ששני הצופים החולפים יסכימו על הזמן…. ואז כתבת ששעון אחד מפגר. איזה מהשעונים יפגר? בילבלת אותי לגמרי, חשבתי שהמצב סימטרי…

    אשאל אותך שאלה אחרת: כל צופה אמור לראות את הרכבת השניה כקצרה יותר. כאן אין בעינייך בעיה?

  155. עוד הפעם אייפון. על פי איינשטיין והיחסות, שני הצופים החולפים זה על פני זה יסכימו ביניהם על הזמנים הנצפים, וששעון אחד מפגר.

    בהצלחה עם יובל.

  156. ניסים

    אולי כדאי באמת שנעזוב. אם אתה ממשיך להתעקש שכל מצלמה תצלם תמונה שונה, הרי שנשמט הבסיס לדיון פיזיקלי בינינו.

    רק בקשה אחת – אל תנסה לנכס לעצמך את איינשטיין. זה לא מה שהוא טוען. על פי איינשטיין והיחסות

  157. ניסים מהירות היא לא רק נגזרת של המקום- היא גם תכונה אמיתית של החץ.
    זה משהו שהוא בסיסי לכל מי שלומד פיסיקה.
    והיא ועוד איך מוגדרת בנקודת זמן.
    גם אינטגרל זה גבול- אז אם נגיד שמיקום זה אינטגרל של המהירות- האם זה סביר להגיד שמיקום זה גבול ולא מוגדר בנקודת זמן?

  158. ניסים, איבדת את סבלנותך? במקום להתעצבן ולהגיד סתם “איינשטיין טעה” נסה להבין לעומק את טענתו של ישראל. אני, אגב, לא משתתף, וזה כי החלטתי להפגין עמדה אמפיריציסטית וטרם קיבלתי תוצאות של ניסוי מעשי בשטח.
    וטענתך שאי אפשר לחבר אינסוף מספרים איננה רלוונטית, וזאת משום שאפשר להגיע לתוצאות רצויות גם בעזרת פעולות אחרות חוץ מחיבור. למשל, ההתכנסות שאתה מפגין כלפיה סלידה שמרנית. גם אמירתך שמהירות איננה מוגדרת בנקודת זמן מדגימה את שמרנותך שאינה במקומה. זנון הצביע על הבעייתיות שבקביעות של פרמנידס, ואילו לא נפל עליו רוגזו של מורו הרי שהחשבון האינפיניטסימלי לא היה מחכה לניוטון ולייבניץ אלא היה נוסד כבר לפני למעלה מאלפיים שנה.

  159. יובל חייקין.
    אי אפשר לחבר אינסוף מספרים. הנוסחה של טור הנדסי היא גבול!!!! הטור שואף לנוסחה – הוא אינו שווה לנוסחה. אתה בעצמך כתבת “טורים המתכנסים לגודל סופי” – שכחת כבר מה משמעות המילה “מתכנסים”? 🙂

    דרך אגב – זה לדעתי הפתרון לפרדוקס החץ של זנון: מהירות היא גבול, כלומר מהירות אינה מוגדרת בנקודת זמן.

  160. ישראל שפירא

    החישובים מתחשבים בזה שהמערכת מואצת
    החישובים מתחשבים במרחק בין התחנות ללווין.

    בוא נעזוב את זה. אתה צודק. איינשטין טעה. התצפיות שגויות.

  161. מן הסתם כבר הבנת שעד אוהיו אין לי מה להוסיף. ובעניין זנון, הניסוח מטעה: אכן אכילס מתקדם אינסוף קטעים, ו”אינסוף” מתקשר בתודעת רבים עם “לעולם לא”. אך האסוציאציה הזו אינה רלוונטית למקרה הנדון. לשם כך, אגב, לא צריך לגייס תותחי חדו”א כבדים ודי באנליזה נומרית בסיסית: סכום אורכי הקטעים וסכום פרקי הזמן הם טורים המתכנסים לגודל סופי, ודי להסתבך.

    ניסים! זנון ערער על קביעותיו של פרמנידס, אך האוטוריטה של האחרון הכריעה והיום זוכרים את זנון בעיקר כבדיחה, שלא בצדק. אני סבור שפרמנידס טעה והטעה, ומשום כך יש לי עניין רב בזנון. הפיתוח היפה שהבאת (וגם המאמר שהפנית אליו) מציג התאמה של זנון למציאות הפרמנידית, אך היות שלדעתי פרמנידס טעה הרי ההתאמה הזו מיותרת בעיני ואני מנסה לפתח תמונת עולם מסודרת המבוססת זנון. כל תרומה תתקבל בברכה.

  162. ניסים
    ראשית, לווינים נעים במערכת מואצת. הנושא שלנו הוא מערכת אינרציאלית.

    שנית, קריאת השעונים אותה הצגת היא ממרחק רב ולכן תקבל השעיה דו כיוונית בגלל מהירות האור המוגבלת. שונה הדבר בצילום מקרוב.

    בעניין זנון – בפרדוקס המקורי אין צעדים ואכילס לא מצליח לעבור את הצב לעולם.

  163. ועכשיו לידידינו זנון מאלאה, תלמידו של פרמנידס
    בקיצור …. אכילס צריך לעבור אין סוף צעדים. פירוש המושג “איi-סוף” – שלעולם לא נגיע לסוף. האם יעלה על הדעת שבאחד הצעדים אכילס יגיע לצב?
    יש הבדל בין לבצע מניפולציה מתמטית כדי לחשב טור לבין לבצע את הסכימה עצמה.
    עכשיו – כדי לפתור פרדוקס – צריך למצוא טעות באחת מהנחות היסוד או בדרך ההיסק. נוסחה נותנת לנו את הזמן שאכילס מגיע לצב אך הוא אינו פותר את הפרדוקס.

  164. ישראל שפירא
    אז פה אני חושב שאתה טועה.
    לדוגמא: כל לווין GPS מקיף את כדור הארץ אחרי 12 שעות. אם אני מבקש מהלווין את השעה כל 24 שעות (ואז הוא באותו נקודה בשמיים) – אני רואה כל יום פיגור של 7 מיקרו. אם הלווין מבקש ממני את השעה כל 24 שעות – גם הלווין רואה שגיאה של 7 מיקרו. זה יותר מסובך מזה, בגלל שדה הכבידה, מהירות סיבוב כדור הארץ ועוד….
    אבל -שגיאה זו נמדדת וחייבים לטפל בה.
    זה לא תיאוריה – ככה זה עובד.

  165. יובל
    לא שכחתי אותך, אני בדרך. ניפגש באוהיו.

    ניסים

    המאמר אכן מתאר, אך הוא אינו ניסוי או עדות.

    לדעתי בכל מהירות צילומים משני גופים החולפים זה על פני זה יראו אותו הזמן בשעונים.

    לשאלתך: כל הצילומים יראו זמן זהה בשעוני 2 הרכבות המחושב עיי טרנספורמציית לורנץ הרלוונטית.

    אנא הסבר את בעייתיות פרדוקס זנון.

  166. ישראל
    אותו מאמר שהבאתי מתאר בדיוק התקצרות האורך. אמנם למדתי את זה לפני 35 שנה (לא נעים ….) אבל אני זוכר בפירוש שהחישובים של משיכה של זוג תייל נושא זרם מראה שהמרחק בין האלקטרונים חייב להתקצר. הרי אין כזה דבר “כוח מגנטי”.

    אז כנראה שלא הבנתי את ניסוח הבעיה שלך. כתבתי שלדעתי, במהירות נמוכות, כל הצילומים של אותו ארוע יראו אותו דבר.

    אני רוצה להבין משהו על הבנתך (ןעל הבנתי) את תורת היחסות: בוא נניח שתי רכבות יוצאות בשעה 10:00, אחד לכיוון השני. המרחק בין נקודות היציאה היא 40 דקות אור ומהירות כל רכבת, במערכת הייחוס הנייחת היא 0.8c. נעמיד צלם באמצע הדרך – ונבקש מכל 3 הגורמים לצלם את כל השעונים בזמן המפגש. בלי לדייק – מה נראה ב-3 הצילומים?

    ובקשר לפרדוקס של זנון – בבקשה תקרא את המאמר. הוא מסביר יותר טוב ממני. ואגיד שוב – זה שיש הסבר מתמטי לתופעה פיסיקאלית לא אומר שפתרנו את הפרדוקס. אם אתה חושב שכן, אז אתה מפספס משהו מאוד עקרוני. אנסה להסביר אם זה מעניין.

  167. ניסים

    אתה נכנס פה לתחום של תורת הנסתר – אתה מתיימר לדעת מה אני מתאמץ, מה אני מבין, מה אני עושה… דברים שרק אני אמור לדעת.

    המאמר שהפנית אותי אליו עוסק באלקטרודינמיקה של גופים נעים – אותו נושא שבגללו הגה איינשטיין את היחסות. מה הקשר שלו לנושא שלנו, צילום של זמנים בשעון על ידי שני גופים החולפים זה על פני זה?

    הטענה שלך לגבי מהו פרדוקס יכולה אולי לעניין את מי שמתעניין בפילוסופיה. הפרדוקס של זנון נחשב לפרדוקס פיזיקלי אמיתי, עד שהחדו״א פתרה אותו והוא הפסיק להיות פרדוקס. את פתרון פרדוקס התאומים במסגרת היחסות הפרטית הבאת אתה.

    השאלה שהבאתי פה – איך זה ש״כל צופה רואה את שעוני השני כנעים לאט יותר״ לא נפתרה עיי התשובות שלך ושל ב, שהן תשובות בלתי אפשריות מבחינה פיזיקלית. קיים פיתרון, ואם תרצה אביא אותו, אך הוא אינו מה שאמרתם.

    אף אחד לא טען שיש לך טעות בנושא הGPS – קרא מה שאמרתי.

    ובניגוד לטענתך, אין שום ניסוי פשוט או עדות המאשרת את התקצרות האורך.

  168. ישראל שפירא
    אני לא רואה הרבה טעם להפנות אותך למאמרים, אתה ממילא לא מתאמץ להבין מה שאני כותב, ולא מתייחס כלל למאמרים שאני מקשר אליהם, למרות, ואולי בגלל, שהם מחזקים את טענותי.

    אבל בגלל אני בחור טוב – קרא בבקשה http://skepticsplay.blogspot.co.il/2011/03/electricity-magnetism-space-and-time.html

    אני מצטער אם נשמעתי יהיר. העניין הוא שהטענה שלי על מה זה פרדוקס הוא עקרוני בעיני. הפרדוקסים של זנון, כולל זה שתיארתי, אינם נפתרים בחדוו”א. גם חץ הזמן, שציין יובל חייקין, אינו נפתר בחדוו”א. לי יש בראש פתרון לעניין, אבל אני לא מתכוון לבזבז את זמנכם בעניין.

    ובנושא ה-GPS – אשמח שתפנה אותי למקור שיראה לי את טעותי. אני מדבר ברצינות.

  169. ניסים, ניסים, מה יהיה הסוף?

    אתה כל הזמן מטיף לכולם להיות צנועים וענווים, אבל משתמש בשפה מתנשאת ומתיימרת.

    החדו״א כן פותר את פרדוקס זנון המקורי. הבעיה הפילוסופית שהעלית פשוט לא מעניינת אותי.

    והתשובה שנתת בעניין המצלמות אינה נכונה. הראתי לך את זאת בכמה דוגמאות. זה שאינך מבין לא אומר שזה לא קיים.

    אינני אומר שאין התארכות זמנים, אני אומר רק ששני צופים החולפים זה על פני זה לא יראו את הזמן של הנגדי כמוקדם יותר. קיים פיתרון לפי היחסות, אבל הוא לא זה שאתה וב מתעקשים עליו, פיתרון שאיננו פיזיקלי.

    בוודאי שיש סתירה בכך שכל צופה רואה את השעון של השני מפגר אחרי זה של עצמו. אם אינך מסוגל לראות זאת, עדיף שתישאר בתחום הפילוסופיה.

    איני מבין מהו פרדוקס? קשוט עצמך תחילה. רוב מה שאתה אומר פה אינו נכון במושגים פיזיקליים. אם יצא לך להתעסק בלווייני GPS ועדיין לא הבנת זאת, אז המצב יותר חמור משחשבתי.

    ולסיום – התוכל לקשר אותנו לאותם ניסויים פשוטים המוכיחים את התקצרות האורך?

    יום טוב.

  170. ובקשר לרכבות…..
    ניסוי של שעונים במהירות גבוהה מתבצעת ברציפות כבר 30 שנה. הניסוי מוכיח את התיאוריה.
    תורת היחסות מנבאת גם התקצרותבציר האורך. גם זה מוכח בניסויים פשוטים.

    ושוב – ישראל, אתה לא מבין מה זה פרדוקס. אין כל סתירה שכל צופה רואה את השעון של מישהו אחר מפגר אחרי שעונו.
    אני באמת לא מבין על מה אתה מתווכח. תורות היחסות תואמת את הראיות בצורה יפה מאוד ואף עברו מתחום המדע לתחום ההנדסה. במקרה יצא לי להתעסק בלוויני GPS והשעונים שלהם …..

  171. ישראל שפירא ויובל חייקין

    תרגיל בחדוו”א לא פותר בעיה במטאפיסיקה. הצעתי לכם ללמוד מה זה נומינליזם ולא עשיתם את זה. אז אפנה אותכם למאמר פשוט שמסביר יותר טוב ממני:
    http://philsci-archive.pitt.edu/2304/1/zeno_maths_review_metaphysics_alba_papa_grimaldi.pdf

    בלי שתבינו את המאמר הזה, אין טעם להמשיך בשיחה על זנון.

  172. ישראל! היה פוקר קצר. זה טוב או רע?
    “התארכות הזמנים תופסת גם במהירויות איטיות, פשוט צריך לנסוע הרבה”, אז קדימה! תתחיל לנסוע ואני אסע לקראתך. מה זה 2500 מייל ביני ובינך?

  173. ישראל! אנא ממך, היה רציני. אנחנו מסתובבים סחור סחור מסביב לביצה שלא נולדה. להגיד “מה הבעיה לעשות?” בלי באמת לעשות, זהו בדיוק בזבוז הזמן המשווע שממנו נקעה נפשי. על חשיבותו של הניסוי המעשי ראה, לדוגמה, את ניסוי מייקלסון מורלי. בטרם זה בוצע לא ידע העולם על התופעה שזכתה לכל השמות, ההסברים והפרדוקסים. אם נצליח לבצע הלכה למעשה ניסוי בו ישתתפו שני כלי רכב הנוסעים במהירות חצי c (אני מוכן להסתפק גם במאית ואפילו באלפית), אולי תהיה בידנו תשובה, ואולי אף יתגלו תופעות שלא ידענו עליהן. אך כל עוד אין לנו ניסוי כזה אנחנו טוחנים זמן לריק.

    ניסים! על הפיתוח המבריק שלך לבעיית זנון אני מסיר בפניך את כובעי. האם אתה יכול להביא כזה גם לבעיות אחרות שלו? למשל, האם חץ הנורה מקשת נמצא בתנועה או שמא באוסף גדול של מצבים נייחים?

  174. ניסים, עוד הפעם סידרת חינוך?!

    כולנו פה ידועים בצניעותינו וענוותינו הרבה. שאלנו שאלה. אסור כבר לשאול שאלות באתר הזה??

    הפרדוקס שלך אינו פרדוקס זנון הישן והחביב, אלא פרדוקס משופר. הפרדוקס הישן נפתר באמצעות החדו״א.

    התשובה שלך על הרכבות אינה עומדת במבחן ההגיון. אם ברור שבמהירויות איטיות שני הצילומים יראו אותו הדבר, אז זה מה שיהיה גם במהירויות יחסותיות. אחרת היינו צריכים להציב לימיט עליון למהירות שבה הצילומים זהים, ומעליה הם מתחלפים.

    יתירה מזאת: התארכות הזמנים תופסת גם במהירויות איטיות, פשוט צריך לנסוע הרבה. לכן אם אתה אומר שבמהירות יחסית של 20 קמ״ש הצילומים משני הצדדים זהים, מי יערוב לך שהרכבת מחיפה לדוגמא, לא הגיעה ישירות מאנדרומדה על האוטוסטרדה החדשה שנבנתה זה מכבר, ובגלל משך הנסיעה הארוך שעוניה מפגרים יחסית לרכבת ת״א?

    חוץ מזה שזהו אינו ההסבר המקובל של היחסות. כשאומרים שכל מערכת רואה את השניה כנעה לאט יותר, אין הכוונה שצילומים משני כלי רכב החולפים זה על פני זה יראו מצב שונה בשעונים, אחרת היינו נתקלים בפרדוקסים יותר מוזרים מאילו של זנון. במקרה של תאומים לדוגמא, כאשר תאום ישיש בעל זקן ארוך ולבן חולף על פני אחיו הצעיר, התמונה שלו תהייה גם כן של תאום צעיר וחייכן, כי הזמן שלו כפי שנצפה מהרכב הנגדי מוקדם בהרבה, לא?

    אז איך בכל זאת מוסבר פרדוקס זה? איך זה שכל מערכת רואה את השניה כאיטית יותר?

  175. ועכשיו על פרדוקס זנון – ואני מבקש מכל מי שחושב שהוא מבין אך לא מבין – קצת צניעות!!
    למי שלא מכיר – אכילס וצב מחליטים לבצע ביניהם מרוץ. אכילס נותן פור לצב ויוצא אחריו. נניח פור של 100 מטר. אחרי שאכילס התקדם 100 מטר הצב התקדם 10 מטר. אכילס רץ עוד 10 מטר והצב מתקדם עוד מטר. אכילס מתקדם עוד מטר, אבל הצב התקדם ב-10 ס”מ.
    זה ברור לכולם שזה טור מתכנס וקל לחשב מתי אכילס יגיע לצב.

    הטור אינסופי – כלומר 100 ועוד 10 ועוד 1 ועוד 0.1 ועוד 0.01 – מסתכם ל 111.1111111
    מטר.

    אז איפה הבעיה? ובכן, אכילס אינו יישות מתמטית – והוא חייב להתקדם צעד אחרי צעד אחרי צעד. הוא לא יכול “לחשב” את הטור הגיאומטרי בנוסחא הידועה – הוא חייב לסכם איבר אחרי איבר.

    אסביר את הבעיה בצורה טיפה שונה. בואו נניח שאכילס מבצע כל צעד כזה בצעד אחד. צעד ראשון 100 מטר וכן הלאה. ובוא נניח שברגע שהוא מגיע לצב הוא מבצע צעד נוסף של 100 מטר כדי לסיים את המסלול שאורכו 211.1111111 מטר.
    השאלה שלי היא – באיזה רגל הוא יסיים את המרוץ?

    אנחנו יודעים שבמציאות אין בעיה. השאלה היא – איפה הטעות? באחת מהנחות היסוד או בדרך ההיסק?

    ובבקשה – קצת צניעות. יש מושג בפילוסופיה שנקרא “נומינליזם”. הרעיון שבעולם הפיסיקאלי אין מספרים, הרי הם רק מושג מופשט. זה לא נכון שאפשר לזרוק נוסחה ולחשוב שפתרנו את הבעיה.

    בהצלחה 🙂

  176. ניסים

    אם אפשר, פשוט ענה במילים פשוטות על השאלה ששאלתי במילים פשוטות: מה יראה הצילום מהרכבת הנגדית שנוסעת במהירות של 20 קמ״ש יחסית אליה.

    טוב, יום שלישי, הולכים לפוקר.

  177. להסביר שוב? 🙂 🙂

    במקרה הקלאסי כל 2 צילומים שיבוצעו באותו זמן יראו את אותו הדבר.

    במקרה היחסותי הזמן במערכת שבתנועה תפגר אחרי הזמן במערכת הנעה. זו תופעה פיסיקאלית, שניצפית בלוויני GPS. לדוגמא – אם קטר א’ יראה אצלו שעה 10:00 ואצל הרכבת החולפת יראה 9:50 אז גם קטר ב’ יראה שהם חולפים שאצלו השעה 10:00, כאשר בעוד שברכבת השנייה השעה היא 9:50. ואגיד שוב – זו תופעה נצפיית בניסויים ואין כאן שום פרדוקס.

    האם מישהו לא מסכים למשהו בדברי?

  178. יובל

    הוא לא טעה, הוא שקלל.

    מה הבעיה לבצע את הצילומים בכל מהירות שהיא אם ברשותינו מצלמה חדת רזולוציה?
    חוץ מזה שהתארכות זמנים של כמה שניות תקרה גם במהירויות נמוכות. פשוט צריך לנסוע הרבה.

    ב
    אינך עונה תשובה פשוטה על השאלה שלי, שנוסחה בפשטות ובבהירות. אני מניח שאין לך תשובה, או שהתשובה לא מתאימה לתיזה שלך.

    אין טעם להמשיך לפני שתענה לשאלה.

  179. ישראל:
    במכונית המרחק בין קצות המכונית הוא לכל היותר עשרה מטרים והיא נוסעת במהירות נמוכה.
    ברכבת שאורכה שעת אור והיא נוסעת במהירות גדולה מאוד המדידות שונות.

  180. בסדר. בוצע. כעת אנא הצע ניסוי בר ביצוע לרכבים הנעים במהירויות שלפי תורת היחסות אינן זניחות.
    ולמה זה תיטפל לשגיאות הקטנות שלי. איפה היית כשיאיר לפיד טעה בשבעה מיליארד שקלים?

  181. יובל
    זה המיסוי הקל ביותר לביצוע. סע עם האוטו של אמא ליד מכונית חונה שבה שעון גדול ושעון קטן מקולקלים המראים שעה ארבע. כוון את השעון הגדול והקטן בטרנטה של אמא שיראו שבע. כשתחלוף ליד האוטו החונה, צלם את השעון הקטן אצלך והגדול שבחונה. בקש מיוסי שיושב בחונה לצלם את השעון הקטן אצלו והגדול אצלך.

    האם יש לך ספק ששני הצילומים יראו: שעוני אמא שבע, שעוני יוסי ארבע?

    כי לטענתם של ב ומיסים, צילום אחד יראה: אמא ארבע, יוסי שבע.

  182. מזתומרת לא מצלמת? הקטר חולף על פני המאסף, כן או לא? המהירות היחסית ביניהם 20 קמ״ש, אז מה הבעיה שלהם לצלם זה את זה?

  183. אם המצלמות ממוקמות בקטרים והם מצלמות אותו דבר אז במערכת המדידה של קטר א המצלמה של קטר ב אינה מצלמת בו זמנית עם המצלמה העצמית.

  184. יובל
    מה שלא יקרה הוא שתמונה מרכבת א תראה את הזמן בשעון הנגדי כמפגר לשלה, ולהיפך.
    שתי התמונות תראינה את אותו המצב בשעונים: אם א מפגר, אז הוא יפגר בשני הצילומים.

    בקשר למיסים ומיפלאות – עדיין מחכה להסבר שלו על ״פרדוקס״ זנון.

    בקשר למיסוי מ-מ – מיפלאות דרכי השם.

    ב

    סימטריה שמימטריה – שאלתי שאלה פשוטה על רכבות שהמהירות היחסית ביניהן 20 קמ״ש. התוכל לענות תשובה פשוטה על השאלה הפשוטה שלי?

  185. יובל:
    אכן כן במערכות שונות הדברים שונים. האורך שונה והזמן שונה. אפשר בכל זאת “לתרגם” ממערכת למערכת.
    ישראל:
    במקרה ששתי המצלמות ממוקמות ברכבת אחת : אין סימטריה בין הרכבות.
    במקרה שהמצלמות ממוקמות בקטרי הרכבות: יש סימטריה בין המצלמות אבל הסימטריה היא אך ורק ביחס לנקודה מרכזית כלשהיא הנמצאת בדיוק באמצע בין המצלמות. (נקודה על הרציף). נקודה זו אינה נעה יחד עם המצלמה. נקודה זו היא מערכת יחוס שלישית.

  186. ישראל! בתור זה שיודע את האמת, אנא האר את עינינו. מה פשוט לא קורה?
    ועד אז אולי יותר לנו לבדוק אפשרויות נוספות, למשל פירושים לתוצאות מיסוי מייקלסון מורליי שטרם נדונו?

    ובקשר לניסים: העובדה שהשכלתו בפילוסופיה יוונית אינה מושלמת אינה מעידה בהכרח על השכלתו בתחומים אחרים. אך נראה לי כי בטחונו הרב בצדקת עצמו מצריך אמצעי זהירות מצדנו.

  187. יובל

    יש עוד אפשרות, שהיא במקרה גם האפשרות האמיתית: זה פשוט לא קורה.

    אין דבר כזה ״כל אחד רואה את השעון של השני נע לאט יותר״ בדוגמא שהבאתי. הרכבות שחולפות זו על פני זו הן רכבות רגילות ויומיומיות, ולא יכול להיות שכל אחת תראה תמונה הפוכה אצל השניה. מי יודע? אולי הן נסעו כבר שנים רבות באותה מהירות איטית ולכן על פי היחסות נוצר אותו פער זמנים שהבאתי. עדיין התמונות משני הצדדים תהיינה זהות.

    ניסים – מה הבעיה של אכילס והצב? חשבתי שהפרדוקס נפתר עם המצאת החדו״א והסדרות המתכנסות. האם אין הסדרה האינסופית של חצי פלוס רבע פלוס שמינית… מתכנסת לאחד?

  188. ניסים! זה לא עניין של אינטואיציה. זו המצאת מתמטיקה חדשה אשר בה יחס הסדר אינו תקף. אם אחד גדול משנים ושנים גדול מאחד הרי זו סתירה.

    ב! הדבר היחיד שאפשר להסיק מדבריך (סעיף 2) בלי להגיע לסתירה הוא שאי אפשר להשוות בין תהליכים הקורים בשתי מערכות שונות.

  189. יובל
    ב’ ענה לך נכון. כל אחד יראה את השעון של השני מפגר. נניח 2 הרכבות מסנכרנות זמן בעמידה ואז נוסעות אחת חכיוון השניה. ברגע החליפה כל אחד יראה את השעון השני מפגר.

    כמו שב’ אמר – אין כאן שום פרדוקס. זה כמובן לא מובן מאליו ונגד האינטואיציה. כשאומרים פרדוקס מתכוונים לטיעון שמביא לסתירה. הכוונה בטיעון זה הנחות יסוד וכללי היסק. בד”כ אנחנו מוצאים שגיאה בהנחות היסוד וכך נפתר הפרדוקס.
    יש פרדוקסים אמיתיים שאנחנו לא יודעים לפתור. לדוגמא הפרדוקס של זנון על אכילס והצב.

  190. ניסים

    לא ענית לשאלה שלי. תארתי מצב והצגתי שאלה. יהיה קשה מאוד להתקדם אם לא אקבל תשובה על השאלה.

    ב. הקטרים משתי הרכבות חולפים על פני המאספים משתי הרכבות. אחרת, אם לדוגמא המפגש הוא בחדרה, הרכבות לא תגענה לחיפה ותל אביב בהתאמה.

    איני מדבר כאן על המצב שבו מאסף א מול קטר ב ולהיפך. זאת מאוחר יותר. כרגע השאלה היא פשוטה: האם כאשר מצולמים שעונים משתי נקודות החולפות זו על פני זו, יראו הצילומים זמנים שונים בשעונים. ענה על השאלה שהצגתי בתגובה הקודמת ואוכיח לך אני מאמין שהם תמיד מצלמים את אותו המצב.

    יובל

    על פי היחסות אין פרדוקס תאומים וניסים הסבר זאת יפה בתגובה הקודמת. אנו מנסים לראות אם ניתקל בסתירה בדוגמא זו של מערכות אינרציאליות.

  191. יובל:
    1)השעון אינו מקדים. רק מרווחי הזמן שונים. אפשר לתאם שעה שתהיה שעת אפס בשתי המערכות.
    2)כל אחד רואה את השעון שלו במערכת המדידה העצמית שלו ואת שעונו של האחר במערכת שנעה יחסית למערכת העצמית. לפיכך אין פרדוקס.

  192. ניסים
    הסברת מה קורה במצב היחסותי, להבדיל מן הקלאסי: כל אחד יראה את שעונו מקדים את השעון של השני.
    ואחרי ההסבר הזה, האם אתה עדיין בדעה שתורת היחסות אינה יוצרת פרדוקס?

  193. אם שתי המצלמות נמצאות ברכבת א הקטר של א יצלם את עצמו ואת המאסף של ב אבל המאסף של א לא יצלם את עצמו ואת הקטר של ב. הקטר של ב לא יגיע בזמן לצילום . אם המצלמות נמצאות בשני הקטרים אז מנקודת המבט של קטר א מצלמה ב אינה מצלמת בו זמנית עם המצלמה שלו.

  194. אנחנו כולנו יודעים מה יקרה במכניקה קלאסית.
    והסברתי מה קורה במצב היחסותי: המצב סימטרי וכל אחד יראה את שעונו מקדים את השעון של השני.

  195. אנונימי וב

    בואו ניקח את דוגמת הרכבת לחיפה שחולפת על פני הרכבת לתל אביב. המהירות היחסית ביניהן – 20 קמ״ש. ברגע שהקטר חולף על פני המאסף הזמן בשעוני הרכבת לחיפה – 1442. בשעוני הרכבת לתל אביב – 1753.

    הזמן מוצג על חלון הקרון והקטר כתצוגת לד דיגיטלית, כך שאין בעיה לצלם את הזמן בשתי הרכבות ביחד ממצלמה אחת מכל רכבת.

    שאלה:

    האם גם במקרה זה, שיכול לקרות כל יום ואין לו קשר ליחסות, יראו הצילומים זמנים שונים בשעונים? האם הצילום מרכבת חיפה יראה: חיפה 1442 תל אביב 1753, ואילו הצילום מרכבת תל אביב יראה: חיפה 1753 תל אביב 1442?

    במידה ותשובתכם היא חיובית, ראו הוזהרתם: אין כל בעיה לבצע ניסוי פשוט כזה עם מכוניות, שעונים ומצלמות. אני מוכן להתערב עם כל מי שמעוניין על כל סכום שירצה שהצילומים יראו את אותם הזמנים בשעונים, לא משנה מאיפה צולמה התמונה.

    מישהו רוצה להתערב?

  196. אני המשתמש האנונימי מקודם.
    הבנת את נושא הצילום?? הכל סימטרי ולכן התמונות “זהות” – השעה אצל המצלם הוא 1000 ואצל המצולם הוא 800. זה אפילו הגיוני…

  197. ב
    עצם קיום התופעה הנקראת “פרדקוס התאומים” מאומתת בניסויים רבים.
    ובפעם המיליון – אין כאן סימטריה!!!! ג’ אינו משנה את מערכת היחוס שלו בעוד שא’ וב’ כן.
    מניין שלפת שהגיל ש ג’ הוא בין הגיל של א’ לגיל של ב’??

  198. לגבי “פרדוקס התאומים” :
    נניח א ב ג הם שלישיה. הם נמצאים בנקודה משותפת בחלל.
    א ו ב נפרדים מ ג ומתחילים להקיף אותו במהירות גבוהה.
    בגלל הסימטריה :
    א ו ב מזדקנים באותו קצב ביחס ל ג שנשאר במקומו.
    הגיל של ג הוא תמיד בין הגיל של א לגיל של ב.
    אם הם חוזרים ונפגשים עם ג בנקודת המוצא הם חייבים להיות כולם בני אותו גיל.
    וזה רק בגלל שיקולי סימטריה.
    אם נתעלם לרגע לרגע מכך שהם שלישיה ונסתכל רק על תאום א שיצא למסע לעומת תאום ג שנשאר במקומו נראה שלאחר השיבה לנקודת המוצא תאום א ותאום ג הם באותו גיל. לכן פרדוקס התאומים אינו קיים.

  199. מאיזו נקודה מתבצע הצילום? מאיזו נקודה מתבצעת המדידה?
    לגבי צמד מצלמות הממוקמות בקטר של א ובמאסף של ב. במדידה מתוך הקטר של א המצלמות יפעלו בו זמנית.
    לגבי צמד מצלמות הממוקמות במאסף של א ובקטר של ב. במדידה מתוך הקטר של א המצלמות לא יפעלו בו זמנית. צילום א יהיה בזמן אחר מצילום ב.
    מערכת צירים המתיחסת לקטר של הרכבת כראשית הצירים אינה יכולה להתיחס למאסף של הרכבת כראשית הצירים.
    מדובר בשתי מערכות יחוס נפרדות אשר ביניהם מפריד מרחק. דבר זה מתבטא בכך שבקצה אחד של הרכבת מדידת זמן הצילום של מצלמה הממוקמת בקטר א מניבה אותו זמן צילום כמו מצלמה הממוקמת במאסף ב. אבל מדידה שמתבצעת מאותו קצה של הרכבת לגבי הקצה השני של הרכבת של זמן צילום של א מניבה תוצאה שונה מזמן צילום של ב.
    אם המצלמות ממוקמות על הרציף אז מדובר על מערכת ייחוס שלישית היא מערכת הרציף ושם אמנם הצילום הוא בו זמני.
    בכל מקרה בגלל הסימטריה בין הרכבות התמונות יהיו סימטריות. כלומר המצלמה של קטר א תצלם את מאסף ב והמצלמה של מאסף ב תצלם את קטר א. וכן גם המצלמה של קטר ב תצלם את מאסף א והמצלמה של מאסף א תצלם את קטר ב.
    שים לב : זוג המצלמות קטר א ומאסף ב ממוקם במערכת יחוס אחת בעוד זו המצלמות קטר ב ומאסף א ממוקם במערכת יחוס אחרת. המרחק בין מערכות היחוס גורם למדידת הפרשי הזמנים בין המצלמות המרוחקות מנקודת המדידה היא נקודת ראשית הצירים.

  200. ישראל שפירא
    ב צודק. אם אתה מצלם מרכבות שונות אז אתה מצלם ממערכות ייחוס שונות. כמו שכתבתי כבר הרבה – בכל צילום הצלם יראה את השעון ברכבת השנייה מפגרת אחרי שלו.

    זה לא משהו תיאורטי. בלוויני GPS יש שעונים אטומיים ויש בנוסף שעונים אטומיים במקטע הקרקעי – אותם תחנות שאומרות ללויינים מה מיקומים. החישובים שם מתבססים על זה שהשעון בצד השני מפגר. האמת – זה יותר מסובך מזה בגלל 2 תופעות נוספות. הראשונה היא תופעת סאניאק והשנייה היא תוצאה ישירה של תורת היחסות הכללית.

    שוב – המצב סימטרי לגמרי – השעון אצל המצלם יראה 1000 והשעון ברכבת השנייה יראה 800. אין כאן כל סתירה.

  201. ב

    אם רכבת נוסעת דרומה לתל אביב חולפת על פני רכבת הנוסעת צפונה לחיפה, מה הבעיה לצלם בעדשה רחבה את השעונים שעל שתי הרכבות, ואין זה משנה מאיזו רכבת אתה מצלם?

    נסיעה טובה.

  202. ישראל שפירא:
    בתיאור הרכבות שלך כאשר השעונים מאופסים והקטרים והמאספים נמצאים זה מול זה.
    אתה מתאר מדידה שאינה מתבצעת מתוך אף אחת מן הרכבות.
    תוצאות מדידה כאלה יתכנו אך ורק מתוך מערכת שלישית הנמצאת בדיוק באמצע בין שני הרכבות.

  203. ניסים

    המצלמות חולפות זו על פני זו, ובאותו הרגע שבהן הן ״נוגעות״ זו בזו, כל מצלמה מצלמת את שני השעונים: בקטר ובמאסף.

    נאמר שהצילום מהקטר מראה: שעון קטר 1000 שניות, שעון מאסף 800 שניות.
    מה יראה הצילום מהמאסף?
    לגבי השאלה שלך – על פי היחסות אין לך כל טעות, וזהו אכן פתרון פרדוקס התאומים במסגרת היחסות הפרטית.
    אולם גם לפי דבריך, אין פה סימטריה. בדוגמא שלי הסימטריה היא מוחלטת(?) אז איך זה שקיבלנו ששעוני רכבת א או ב יפגרו? איפה הסימטריה?

  204. ישראל שפירא
    נניח שהחללית טסה לכוכב במרחק 4 שנות אור ובמהירות 0.8c. לגבי התאום הקרקעי – זמן המסע הינו 5 שנים לכיוון, סה”כ 10 שנים. עבור התאום הטס – המרחק מתקצר ל-2.4 שנות אור (קל לחשב). לכן – 3 שנים לכיוון ו-6 שנים עד שהתאומים ניפגשים. כלומר התאום שטס צעיר יותר מאחיו ב-4 שנים.

    עכשיו שים לב – לגבי הצופה הקרקעי – שעון החללית רץ לאט יותר, ולכן, אין כאן כל סתירה.
    אין כאן כל צורך בתורת היחסות הכללית – הפרטית בהחלט מספיקה.

    אין כאן סימטריה – התאום הקרקעי נמצא כל הזמן במערכת צירים אינרציאלית קבועה – אבל התאום הטס משנה את מערכת הצירים האינרציאלית שלו.

    האם ואיפה אני טועה?

  205. ישראל
    מצד רכבת א’ – כשהקטר של א’ חולף את המאסף של ב’, הקטר של ב’ עדיין לא יחלוף את המאסף של א’.
    התמונות תהיינה זהות – השעון של המצלם יקדים את השעון של המצולם.

  206. ניסים
    אז איפה נמצא מאסף א – לפני או אחרי קטר ב?
    לפי דבריך התמונות אינן זהות אלא הפוכות. האם אתה מתכוון שהמצלמות החולפות זו על פני זו תצלמנה תמונות שבהן שעון מסויים מראה זמנים שונים?
    יש איזה חמש תגובות שלך על מאדים – התוכל להעתיק לי את התגובה הספציפית עליה תרצה תשובה?
    תודה.

  207. ישראל שפירא
    אני מתכוון שכשקטר א’ נמצא ליד מאסף ב’ (מנקודת מבט קטר א’) אז מאסף א’ אינו נמצא ליד קטר ב’ (מאותו מערכת יחוס).
    התמונות “זהות” – בזה שכל צלם רואה את שעונו מקדים את השעון של הקטר השני.

    אני נתתי דוגמא של טיסה למאדים – אולי תקרא את מה שכתבתי ???

  208. ניסים

    כשאתה אומר ״אינן מתמזגות״ למה אתה מתכוון? האם יש לך ספק שקטר א יעבור מול מאסף ב ולהפך?
    אם התמונות זהות, אז איך הן מראות דברים שונים, דהיינו זמנים שונים בשעונים?
    ומהי השאלה שלך?

  209. ישראל שפירא
    הרכבות אינן “מתמזגות” – כל רכבת רואה את השנייה כקצרה יותר.
    כל רכבת רואה את השעון של הרכבת השנייה מפגרת אחרי הזמן שלה. זה קורה בכל זמן התנועה.

    1 – התמונות זהות
    2 – כל רכבת תראה שהשעון של הרכבת השניה מפגרת.

    עכשיו – לא ענית על השאלה שלי…..

  210. ניסים

    אנסה הפעם עם רכבות, כפי שאתה ואיינשטיין אוהבים.

    רכבת א חולפת על פני רכבת ב. אורך כל רכבת במערכת הייחוס שלה – שעת אור. בכל רכבת קטר וקרון מאסף, ששעוניהם מסונכרנים בינהם.

    כאשר מתמזגים קצותיהן, קטר א נמצא מול מאסף ב וקטר ב נמצא מול מאסף א. מצלמות חדות רזולוציה משתי הרכבות מנציחות את הארוע. הזמנים בצילומים: 0 בכל השעונים.

    כאשר מגיע מאסף א מול קטר ב מצולם המצב שנית מהמצלמות בקטר ובמאסף.

    שאלות:
    1. האם התמונות ממאסף א וקטר ב תהיינה זהות? או שכל תמונה תראה זמן שונה בשעונים.
    2. האם הזמן בשעונים יהיה זהה? אם לא, איזה שעון יקדים?

    בהצלחה.

  211. ישראל שפירא
    קראתי שוב את מה שכתבת למעלה – זה ממש לא ברור. אנ ינתתי דוגמא פשוטה יותר – ולא אמרת לי את דעתך עליה.

  212. ניסים
    נראה לי שמגיעים לך עוד כמה פרסים, שגם הם משום מה מסתיימים ב ות.
    מכיוון שאף אחד לא דיבר פה על הרכבת של איינשטיין, ומכיוון שאתה מתעקש לא לענות על מה ששואלים אותך אלא לענות בכלליות על נושאים אחרים, אני נאלץ להסיק בצער שאו שאינך מבין את שאלתי, או שאינך יודע את התשובה.

    אז אנסה בפעם האחרונה: האם אתה מתכוון לענות על מה ששאלתי ואך ורק על זה? אין לי שום חשק לעבור סדרת חינוך או לשמוע על כל מיני דוגמאות שאינן קשורות לעניין. אם דרושות לך הבהרות בקשר לשאלתי, בקש ואבהיר. אבל אנא אל תצא מההנחה שאתה בהכרח צודק, אלא עבור על השאלה בפרטים.

    יום טוב.

  213. ישראל שפירא
    מגיע לי פרס על סבלנות…….
    נתתי דוגמא פשוטה ב-2 נקודות. אם 2 הנקודות שלי נכונות אז אני צודק. אם אני לא צודק אז לפחות נקודה אחת שגוייה.
    תראה לי בדוגמא שלי איפה אני טועה.

    הדוגמא שלך היא, כמו שאמרתי כבר, דוגמא של הרכבת של איינשטיין. במקרה הזה – כל עוד התנועה קצובה אז כל אחד רואה את השעון של השני רץ לאט יותר. הוא גם רואה את השני מקווץ בציר האורכי (2 הדברים שקולים).
    בדוגמא שאני מדבר אליו המצב שונה!!! (וזה בדיוק פרדוקס התאומים). מרחק התנועה ב-2 מערכות היחוס שונות. ולכן – המצב אינו סימטרי.

    שמע – זה ניסוי שעשו כבר והתוצאות מאשרות את מה שאני אומר.

    בבקשה – באיזו נקודה אתה חושב שאני טועה?

  214. ניסים
    אינך מבין את ״הטעות״ שלי – אולי מהסיבה הפשוטה שאין פה כל טעות.
    יקל עליך לראות זאת אם במקום הארץ ומאדים, נציב חלליות ה׳ ומ׳.

    עכשיו: מבנה חלליות א׳ וב׳ חולף על פני מבנה חלליות ה׳ ומ׳.
    המרחק בין חלליות א׳ ב׳ – שעת אור.
    המרחק בין חלליות ה׳ מ׳ – שעת אור.

    שעוני חלליות א׳ וב׳ מסונכרנים ביניהם ושעוני חלליות ה׳ ומ׳ מסונכרנים ביניהם.

    ברגע שא׳ חולפת על פני ה׳ הזמן בשעוניהם 0.

    בעבר השני, ברגע שב׳ חולפת על פני מ׳ הזמן בשעוניהם 0.

    המצב עכשיו זהה למצב שהצגנו קודם, רק במקום מאדים יש חללית מ׳, במקום הארץ יש חללית ה׳, וישנה חללית נוספת ב׳ שאינה משנה דבר.

    על פי דבריך – כשא׳ מגיעה למ׳ השעונים מראים: א׳ 24, ב׳ 40.

    אבל שים לב: אילו חלליות, לא פלנטות. אנו יכולים לאמר שא׳ היא בעצם מאדים ומ׳ הוא בעצם חללית. ואז יצא שהשעונים מראים: מאדים 24, חללית 40.

  215. ישראל שפירא
    אז אני מבין מאיפה הטעות שלך:
    1. החללית יוצאת מכדור הארץ לכיוון מאדים ורואה את מאדים במחרק 24 דקות אור.
    2. אבל – ממאדים התמונה היא שהחללית יוצאת מכדור הארץ, שזו נקודה עומדת במרחק 40 דקות אור.

    אתה מבין את 2 הנקודות שלי? המהירות היחסית בשני המיקרים הוא באמת אותו דבר – 0.8c, אבל המרחקים שונים.

  216. יובל יקיר.

    מה שאתה אומר זה בערך כך (אם הבנתי נכון):
    גם בחללית וגם במאדים מותקנות מצלמות שמצלמות את שני השעונים, בחללית ובמאדים, ברגע שהחללית חולפת על פני מאדים. מכיוון שיש פער של נאמר 20 דקות בין השעונים, הצילום ממאדים יראה ההיפך מהצילום מהחללית. מאדים לוחש במתיקות לחללית: את שוב מפגרת יקירה. היא סונטת חזרה: אתה בעצמך מפגר. אכן, יחסות במיטבה!

    פרדוקס מארץ הפרדוקסים.

    אך לא זה המצב. בפועל, שני התצלומים יראו אותו הדבר.

    ניסים.

    גם אני אנסה שוב, בפעם השביעית או השבעים. ;(

    ראשית, אני מאמין שאתה טועה בתשובתך ליובל. יובל טוען שכל מצלמה מראה הפוך מרעותה. לא כך המצב. שתיהן מראות אותו הדבר: החללית מפגרת יחסית למאדים.

    אולם שים לב לבעייתיות של המצביות: מאדים נע בכיוון החללית בדיוק כפי שהחללית נעה בכיוונו. אתה יכול להחליף ביניהם, ואז יוצא שהמרחק ממאדים הנע בכיוון החללית מתקצר, ומאדים יגיע לחללית בזמן מוקדם יותר.

    אם אינך רואה את הסימטריה, חשוב על חללית נוספת שחולפת על פני מאדים בזמן 0, ומהירותה וכיוונה כמהירות החללית הראשונה. עכשיו קיבלנו סימטריה מוחלטת: שתי החלליות שקולות לחלוטין למערכת ארץ – מאדים. כשהן חולפות על פני הפלנטות, המצלמות מצלמות 0 בכל ארבעת השעונים: בשתי החלליות, בארץ ובמאדים.

    אם לטענתך כשתגיע חללית א׳ למאדים יראה הצילום ששעונה מפגר יחסית אליו, אז בגלל הסימטריה כאשר יגיע מאדים לחללית א׳ יראה הצילום ששעונו מפגר יחסית אליה. אך זהו אותו צילום בדיוק. אז איך הוא יראה דברים הפוכים?

    אם יש לך הסבר פרטני, תבורך. רק אם אפשר, בלי יותר מדי פדגוגיה. פשוט תסביר מה קורה.

  217. ישראל שפירא
    אסביר שוב, בפעם השביעית אני חושב 🙂
    נניח שהמרחק למאדים הינו 40 דקות אור ומהירות החללית 0.8c
    לצופה קרקעי וגם לצופה על מאדים – זמן הטיסה הינו 50 דקות.
    לצופה בחללית המרחק הוא רק 24 דקות אור (צריך להסביר למה?). לכן זמן הטיסה הינו 30 דקות.
    המצב אינו סימטרי. מאדים נחה לגבי צופה ארצי אך נעה לגבי צופה בחללית.
    זה הכל. תורת היחסות הפרטית פותרת את זה יפה.

    מה שיובל חייקין מתאר בצורה מסובכת זה ניסוי מחשבה של איינשטין מלפני 100 שנה – ניסוי הרכבת. התוצאה היא כמו שיובל אומר – וזה מאוד לא אינטואיטיבי. אבל זה לא פרדוקס.

  218. ישראל יקירי, בטרם נחבוט ונתחבט לשוא בוא בבקשה ונפשט את הבעיה.
    היות ששתי הפלנטות נייחות זל”ז, הרי מהירותה של החללית יחסית לאחת זהה למהירותה יחסית לשניה. יתירה מזו, נוכל אפילו להתייחס אל שתי הפלנטות כאל גוף אחד ועל כן דין שתי מצלמותיהם כדין מצלמה אחת (או, אם תרצה, שתי מצלמות שממסר-על מקשר ביניהן). לפיכך, תמונת מחוגי שעון החללית תהיה זהה בכל אחת מן המצלמות הפלנטריות והתמונה שתפיק מצלמת החללית ממחוגי שעון הפלנטה האחת, תהיה זהה לתמונה שהיא תפיק ממחוגי שעון הפלנטה השניה.
    לצורך השלמת הניסוי נתקין בצד כל מצלמה עוד מצלמה שתצלם את שעון עצמה בדיוק כאשר הראשונה מצלמת את השעון במערכת האחרת. וראה זה פלא: בעוד שמצלמות החללית יראו כי שעוני הפלנטות מפגרים ביחס לשעון החללית הרי התמונה שתתקבל במצלמות הפלנטריות תראה כי שעון החללית מפגר ביחס לשעוני הפלנטות. ואם זה לא פרדוקס, אז מה זה?

  219. נפלט לפני הזמן. אייפון, המשך: על פניה, הזמן בחללית מאוחר יותר. אז איפה הסימטריה? למה שבמאדים לא יהיה זמן מאוחר יותר?

  220. ניסים
    מצטער שלא הבנת את השאלה. שים לב: יש לך פה שתי מערכות אינרציאליות – חללית ומאדים – שכל אחת מביניהן מבחינתה נמצאת במנוחה, והאחרת היא זו שנעה. ולמרות זאת, לפי דבריך, כאשר החללית חולפת על פני מאדים – או כשהוא חולף על פני. –

  221. ישראל שפירא
    לא הבנתי את השאלה שלך. בכל מקרה – אין כאו סימטריה. החללית משנה את מערכת היחוס שלה, בעוד שהצופה הנייח נשאר באותה מערכת יחוס.

    אני באמת לא מבין למה אתם לא קולטים את מה שאני אומר. הסתכלו שוב על הדוגמא שהבאתי – ותראו לי איפה אתם חושבים שאני טועה. תפסיקו להשתמש בסיסמאות והתייחסו למה שכתבתי. נתתי דוגמא מספרית ולא מצאתם בה טעות.
    תהיו יותר צנועים ותנסו ללמוד משהו, בסדר?

  222. בינתיים אף אחד עוד לא ענה לשאלה שלי (התגובה האחרונה שלי מלפני שבועיים).
    ב – זה במיוחד בשבילך. תנועה לא מואצת. ניסים – בעצם גם בשבילך. שבירת הסימטריה. יובל – על אחת כמה וכמה בשבילך. אתה העלית מחדש את הנושא החבוט, תשלם.

  223. יובל חייקין
    אכן תודה. אתה כמובן צודק. אבל מה האינטרס שלי בזה. גורנישט.

  224. יובל חייקין
    המהירות היחסית שלהם לא שייכת לעניין. הסברתי מספר פעמים ואסביר שוב. ובבקשה – תקשיב 🙂
    נניח שהחללית טסה לכוכב במרחק 4 שנות אור ובמהירות 0.8c. לגבי תאום הקרקעי – זמן המסע הינו 5 שנים לכיוון, סה”כ 10 שנים. עבור התאום הטס – המרחק מתקצר ל-2.4 שנות אור (קל לחשב). לכן – 3 שנים לכיוון ו-6 שנים עד שהתאומים ניפגשים.
    עכשיו שים לב – לגבי הצופה הקרקעי – שעון החללית רץ לאט יותר, ולכן, אין כאן כל סתירה.

    אין כאן סימטריה – התאום הקרקעי נמצא כל הזמן במערכת צירים אינרציאלית קבועה – אבל התאום הטס משנה את מערכת הצירים האינרציאלית שלו.

    לא צריך את תורת היחסות הכללית. לא צריך להתעצבן. התופעה נצפתה במספר גדול של ניסויים. הבנתם???

  225. שמעון ופלאפל: אתם צודקים, אך תגובותיכם קנטרניות ואינן מועילות. אנא תרמו לנו מן הידע שלכם ועזרו לניסים להבין היכן הוא שוגה

  226. ניסים, אתה לא רציני או שפשוט אינך מבין את הטענה. כאשר תאום אחד נשאר על הארץ והשני טס בחללית, שניהם נעים באותה מהירות זה יחסית לזה. לכן, מה שחל על התאום הראשון יחסית לשני הוא בדיוק מה שחל על השני ביחס לראשון.
    ומה שהוכח בניסויים אלה טענות של תורת היחסות הכללית, לא הפרטית, ולא על אלה מדובר כאן.

  227. רוב הדברים שאתה כותב אינם נכונים ואתה גם עושה סלט מהמושגים.
    אם היית יותר לומד ופחות מקשקש היית מצליח להבין לבד למה . בהצלחה

  228. ב
    יש תאוצות בתורת היחסות הפרטית. זה יותר מסובך, אבל זה קיים. עקרונית, אין לפרדוקס התאומים קשר לתורת היחסות הכללית. יתרה מכך, ההסבר לפרדוקס לא קשור להאצות/האטות.
    שמעון מגבת – מה לא נכון במה שאני אומר?

  229. שניכם מתבלבלים לסירוגין. זנ נובע מהעובדה שחסר לכם ידע בנושא שעליו אתם מקשקשים. מספיק ודי.

  230. חללית א וחללית ב יוצאות מנקודה מספר אחת לנקודה מספר שתיים.
    אם חללית א מהירה יותר היא תגיע לנקודה מספר שתים לפני חללית ב.
    לכן אם החלליות נעות בתנועה קצובה לא יתכן שהן יפגשו מחדש בנקודה מספר שתיים.

  231. מה שאני מנסה להגיד הוא שתורת היחסות הפרטית מדברת על תנועה קצובה.
    אם אין תנועה קצובה אז הכל שונה.
    לגבי פרדוקס התאומים:
    תאומים שנפרדו ונעים בתנועה קצובה זה ביחס לזה אינם יכולים לשוב ולהפגש עוד !
    לכן פרדוקס התאומים לא קיים!

  232. ב
    מה אתה מנסה להגיד? אתה כל פעם ממציא סיפור חדש. אין לך מושג על מה אתה מדבר. מה שאמרתי הוא הסבר נכון תיאורטית ומוכח ניסויית ל”פרדוקס התאומים”. מה שאתה אמרת זה גיבוב עובדות, חלקן נכונות, שאין להם שום קשר לבעיה שדיברנו עליו.
    אני באמת לא רואה טעם לנסות להסביר לך שוב. חבל על הכוח….

  233. אתה שוגה וחבל שאתה מתבטא כך.
    הסימטריה בין שתי החלליות אינה מתבטאת במרחק שכל אחת עברה.
    הסימטריה היא :
    אם א רואה את ב נע במהירות מסויימת אז ב רואה את א נע באותה מהירות בכיוון ההפוך.
    לכן:
    אם א טוען שהזמן אצל ב מתארך אז ב טוען שהזמן אצל א מתארך.
    לאורך כל הדרך מן הפרידה ועד הפגישה המשותפת כל אחד מודד אצל השני את אות התארכות של הזמן.
    אבל שים לב:
    זה קורה אך ורק אם שתי המערכות הן בתנועה קצובה. אבל שתי מערכות שנמצאות בתנועה קצובה אינן יכולות להפגש פעמיים . הן יכולות להפגש בנקודת זמן ומרחב משותפת פעם אחת ויחידה. על מנת להפגש פעם נוספת הן צריכות לשנות את הכיוון ולכן זה לא נקרא תנועה קצובה.

  234. ב
    הדוגמא שלך פשוט שגויה ואתה מסיק מסקנה שגוייה. המצב אינו סימטרי בין החלליות. שתיהן טסות לאותו נקודה בחלל במהירויות שונות ולכן הם ירגישו זמן שונה, מסיבה שתיארתי (החללית המהירה תטוס מרחק קצר יותר).

    צר לי, אבל אין לך מושג בתורת היחסות. תורת היחסות הפרטית מתמודדת יפה מאוד עם מערכות מואצות. ההסבר שנתתי הוא ההסבר המדוייק וכל עוד אתה לא מבין אותה אל תבזבז זמן בכתיבה כאן.

  235. 1)התאומים יהיו באותו גיל כי יש סימטריה לגבי חלליות שנעות זו ביחס לזו.
    2)השגיאה בתיאור שלך היא שאתה מנסה לנתח מצב שאינו שייך לתורת היחסות הפרטית.
    תורת היחסות הפרטית נכונה אך ורק לגבי מערכות שנעות זו ביחס לזו בתנועה קצובה (מערכות אינרטיות).
    זה אינו נכון לגבי חללית שיוצאת מנקודה מסויימת וחוזרת לנקודה זו. אם היא חוזרת לנקודת היציאה משמעות הדבר שתנועתה אינה תנועה קצובה.

  236. ב
    הנה המקרה הפשוט:
    נניח שחללית יוצאת בקו ישר מכדור הארץ לכוכב הקרוב וחזרה במהירות גבוהה מאוד אך קבועה.
    לגבינו – המרחק שהחללית עוברת היא לשמש וחזרה וזמן שעבר הוא המרחק הזה מחולק במהירות.
    לגבי החללית המצב שונה – בגלל המהירות, המרחק לשמש קטנה יותר ולכן יעבור פחות זמן, וכנ”ל לגבי החזרה.

    שים לב ששני התאומים מייחסים אותה המהירות לחללית!!! כלומר המרחק השתנה אבל המהירות קבועה, ןלכן השוני בזמנים.

    עכשיו – בלי דוגמאות מסובכות – תגיד לי מה לא נכון בתיאור שלי. אם תרצה, אפשר גם דוגמא מספרית פשוטה.

  237. ב
    זה שתכתוב שטויות הרבה פעמים לא הופך אותן לנכונות 🙂
    לגבי התגובה הראשונה שלך:
    למה אתה מניח שהתאומים יהיו באותו גיל?

    לגבי התגובה השנייה:
    אתה יוצא מההנחה השגויה שלך מהתגובה הראשונה, ולכן המסקנה גם שגויה

    לגבי התגובה השלישית:
    תנסה להבין את המקרה הפשוט לפני שאתה מסבך את עצמך במקרים מורכבים יותר.

  238. לצורך המחשה:
    נניח שחללית מתרחקת במשך שנה מרחק שעת אור ואחר כך חוזרת חצי שעת אור. וחוזר חלילה.
    מכדור הארץ מודדים את מיקום החללית ומסיקים על מהירותה.
    לאחר שנה וחצי החללית נמצאת במרחק של חצי שעת אור מכדור הארץ. לאחר שלוש שנים החללית נמצאת במרחק של שעת אור מכדור הארץ. כלומר מהירות התרחקותה מכדור הארץ היא חצי שעת אור לשנה וחצי. וזאת כאשר מהירות החללית בפועל היא שעת אור לשנה.
    במקרה כזה מהירות החללית ה ר ג ע י ת היא שעת אור לשנה אבל מהירות החללית הממוצעת היא חצי שעת אור לשנה וחצי.
    בכדור הארץ לא יודעים שום דבר על המהירות הרגעית של החללית . המדידה היא רק של המהירות הממוצעת. הפרשי הזמנים בשעונים הם אך ורק לפי המהירות הממוצעת.

  239. המסקנה המתחייבת היא שלאחר שובם של היוצאים לנקודה ממנה יצאו מתבטלים כל הבדלי הזמנים.
    חזרה לנקודת המוצא מבטלת את הבדלי הזמנים במערכות מדידה שונות.
    איך זה קורה?
    זה קורה בגלל שינוי המהירות. (תאוצה).
    אם במשוואות של תורת היחסות הפרטית נחליף את מושג “המהירות” במושג “המהירות הממוצעת” נראה שהמהירות הממוצעת של החללית היא אפס משום שהיא חזרה לנקודת המוצא כלומר עברה כברת דרך של אפס מטרים בפרק הזמן הנתון.

  240. נניח שביחס לכדור הארץאחת מהחלליות טסה במהירות גדולה מאוד והשניה במהירות קטנה מאוד.
    בכל זאת שני האחים בחלליות יהיו בני אותו גיל בשובם יחדיו לכדור הארץ.
    פירוש הדבר שהגיל אינו תלוי במהירות ביחס לכדור הארץ.

  241. ב
    כלומר – אתה אומר שהם מגיעים בו זמנית לנקודה בחלל ושוב ניפגשים בו זמנית בכדור הארץ. אם 2 הנוסעים באותו גיל בנקודת המפגש בחלל אז, בגלל הסימטריה, הם בם בני אותו גיל בנקודת המפגש על כדור הארץ.

    שוב – אני לא מבין מה הבעיה. הם בני אותו גיל, אבל צעירים יותר מאחיהם שנשאר על כדור הארץ. איפה הפרדוקס?

  242. כאשר שתי חלליות נעות זו ביחס לזו אין האחת מהירה מן השניה. יש תנועה הדדית ותו לא.
    מהירה מהשניה מתיחס אך ורק ביחס לנקודה שלישית כלשהיא.

  243. לגבי השלישיה ….
    בנקודת המפגש הם אינם בני אותו גיל.
    אני לא מבין איך קבעת שהמצב סימטרי – חללית אחת מהירה יותר מהשנייה.

  244. יובל חייקין
    מה שאני (וכל מי שמבים בעניין) אומר זה שניסוי של התאומים אינו פרדוקס – יש לנו הסבר למה שקורה שם, הסבר פשוט המסתמך על תורת היחסות הפרטית. אין כאן כל סתירה לוגית – התאום שנסע חוזר צעיר יותר. זה נכון בתיאוריה וזה מוכח בניסויים.
    אם אתה לא מבין את ההסבר אשמח להסביר אותו שוב…..

  245. תאומים ושלישיה:
    נניח שאחד מן השלישיה נשאר בכדור הארץ. שני תאומים ממריאים מכדור הארץ לחלל. הם טסים במהירויות שונות זה מזה . לאחר סיבוב בחלל הם נפגשים וחוזרים יחד לכדור הארץ.
    בנקודת הפגישה שלהם בחלל הם בני אותו גיל בגלל הסימטריה ביניהם. (בין השניים שיצאו לחלל אין עדיפות של חללית אחת על פני האחרת). הם חוזרים לכדור הארץ ביחד.
    מה יהיה גילו של כל אחד מהם ביחס למי שנשאר בכדור הארץ? האם יתכן שבכדור הארץ ימדדו הבדל בין הגילים שלהם כאשר הם עצמם יעידו שהם בני אותו גיל?

  246. ניסים (וכל מי שאומר שפרדוקס התאומים לא קיים):
    אם צריך פירוש רש”י אז לא בפיסיקה עסקינן כי אם בכתבי קודש.
    כשתאום א מזדקן יחסית לתאום ב ותאום ב מזדקן יחסית לתאום א, קורין זאת פרדוקס. לא יעזור בית דין ושום בר דעת לא יפסוק אחרת.

  247. ניסים, לפי טענתך, הזמן במערכת החללית נע לאט יותר מבמערכת ארץ – מאדים.

    אבל מה קורה אם מסתכלים מכיוון המערכת הנגדית? הרי מבחינתה הזמן במערכת מאדים נע לאט יותר, אז מדוע הקביעה החד משמעית (שמונצחת בתמונות, גם מהחללית וגם ממאדים) שדווקא שעון החללית נע לאט יותר? איפה הסימטריה?

    זה איננו פרדוקס התאומים שבו נשברת הסימטריה בגלל שהתאום הצעיר מסתובב וחוזר. כאן שתי המערכות אינרציאליות, במהירות קבועה, ולמעשה שתיהן מבחינתן כל הזמן במנוחה. אז למה אפליה?

  248. ישראל שפירא
    זמן הטיסה למאדים, במערכת צירי הכוכבים הינו שעתיים. במערכת החללית זמן הטיסה הינו 0.87 שעות. לכן השעון בחללית יראה 0.87 שעות והשעון על מאדים יראה שעה.
    זה נשמע מוזר, אבל שים לב: מצד החללית – הטווח הינו 0.87 שעות אור אבל מצד צופה במאדים המרחק הינו שעת אור.
    זה בדיוק ההסבר לפרדוקס התאומים…..

  249. ניסים.

    הנה שנית החידה בפירוט:

    נאמר שהמרחק בין מאדים והארץ הוא שעת אור בדיוק, ואין תנועה יחסית ביניהם. סינכרנו את השעונים ביניהם בדרך שמציע איינשטיין, דהיינו באמצעות קרני אור או גלי רדיו.

    זמן רב אחרי שבוצע הסינכרון, חולפת חללית על פני הארץ בדרכה למאדים. מהירות החללית ביחס למערכת ארץ – מאדים, חצי C, והיא נשארת קבועה כל הדרך למאדים.

    כאשר חולפת החללית על פני הארץ, מצלמת מצלמה חדת רזולוציה בארץ את שני השעונים, בארץ ובחללית. מצלמה זהה בחללית מצלמת גם היא את שני השעונים.

    הצילומים משתי המצלמות מראים: 0 בשעון הארץ ו0 בשעון החללית.

    השאלה:

    כאשר תחלוף החללית על פני מאדים, מה יראו הצילומים של שני השעונים במצלמות ממאדים ומהחללית?

    אין צורך בפרטים מדוייקים. רק מוקדם, מאוחר, או אותו הדבר.

    הדילמה:

    על פי היחסות, כל מערכת רואה את שעוניה של האחרת כנעה לאט יותר. לכן האפשרויות שלפנינו הן:

    1. כל הצילומים משתי המצלמות יראו את אותו הזמן בארץ ובחללית.
    2. כל הצילומים משתי המצלמות יראו את החללית מפגרת יחסית למאדים.
    3. כל הצילומים משתי המצלמות יראו את מאדים מפגר יחסית לארץ.
    4. הצילום מהחללית יראה אותה מפגרת ,והתצלום ממאדים יראה אותו מפגר.
    5. להיפך.
    6. וריאציות נוספות.

    מה התשובה?

    יובל, השונא בזבוז זמן משווע: אין פה ברכה לבטלה. ניסים ואני באמת מתכננים קפיצה למאדים, ורוצים לוודא שלא נגיע בטעות בדיוק באידיו של מרס, כשמאדים בצד מאדים, עם הקרניים שהצמיחה לו ונוס. מכאן נושא הסינכרון.

  250. ישראל שפירא
    לא הבנתי עד הסוף א החידה שלך, כי אני לא מבין מתי ואיך סונכרנו השעונים.
    אבל בוא נסתכל על זה בעזרת “פרדוקס” התאומים (אין כאו באמת פרדוקס כמובן). נניח שיש חללית על כדור הארץ. אנחנו מסכנרים איתה שעון ומשגרים אותה למאדים וחזרה. נניח שהיא נעה קרוב למהירות האור ואנחנו משגרים אותה למאדים במרחק דקת אור. בשעון שלנו, יעברו כמעט 2 דקות עד לחזרתה לכדור הארץ. אבל – החללית רואה את מאדים יותר קרוב, וכשהיא מסתובבת חזרה גם כדור הארץ נראה יותר קרוב. לכן – הזמן שיעבור, בחללית, תהיה קצרה בהרבה משתי דקות. השעון (של החללית) בנחיתה יראה, נניח, שעברו רק 40 שניות.
    זו תוצאה ישירה של תורת היחסות הפרטית ואינה קשורה לתאוצות.
    I rest my case …

  251. יובל

    איזה פרדוקס? למה עקר? שאול שאלתי שאלה. אסור כבר לשאול שאלות באתר הזה?
    ואין דרך טובה יותר להבנת נושא מאשר התדיינות.

    אז מה התשובה לשאלתי? האם החללית תמהר/תפגר/תשאר יחסית למאדים?

  252. ישראל! לא באתי להוסיף להג לשמו. כל חפצי בהתערבות האחרונה היה רק להדגיש עד כמה הדיון הזה עקר. אין לנו תצפיות שיאששו אף אחת מן הטענות שאתם ברוב חכמתכם מעלים כאן. מה שאתם עושים זה הפרחת השערות, ובלי לחכות לאישור נכונותותן אתם בונים עליהן השערות נוספות וחוזר חלילה. לכל היותר אתם מגיעים שוב ושוב למסקנה שמדובר בפרדוקס. לא חבל על הזמן?

  253. יובל.

    שאלת הבונוס (אם אליה התייחסת) מדברת רק על הזמן בשעון בו ניתן לצפות במישרין, ולא על זמן מחושב.

    הסיבה: כל אחד וחישוביו עימו. אך עם צפייה – ורצוי שתהייה מגובה באיזו תמונה חדה וברורה – אי אפשר להתווכח.

    אז מה יראו שעוני החללית ומאדים בצילום על פי היחסות?

    אנונימי (שוב אתה, ניסים?)

    מסכים איתך בכל חוץ מ3. הסיבה: אתה יכול אמנם לסנכרן את שעוני שתי החלליות כמו שאמרת, אבל אז לא תוכל לסנכרן את המרחק בין 2 החלליות במערכת החלליות כך שהוא יהיה זהה למרחק בין הארץ למאדים.

    מה שיצא הוא שאם מבנה של שתי חלליות שהמרחק ביניהן הוא שעת אור ושעוניהן מסונכרנים חולף על פני המבנה הזהה, ארץ – מאדים, שגם שעוניהם מסונכרנים ושעת אור מבדילה ביניהם, אז אם צילום משותף של חללית א’ וכדה”א יראה זמן 0 בשניהם, אז בצד השני במאדים יראה הצילום זמן 0 שעון מאדים וחלל ריק. חללית ב’ עדיין לא הגיעה אל מאדים.

    הסיבה היא שבגלל התקצרות לורנץ, המרחק ביניהן התכווץ במערכת ארץ – מאדים.

    או כך לפחות הבנתי את הנושא.

    אבל אם אינך מקבל את הטענה, ואתה טוען שבזמן 0 מאדים תחלוף לידו חללית ב’ ושגם בה יראה השעון 0 וזה מה שיראה צילום משותף – אוכל אני מאמין להוכיח שיש בעיה בכל הנחת היסוד של איינשטיין כפי שהיא מופיעה במאמריו ובספריו.

    אז לא עדיף לקבל את הטענה ולסגור עניין?

  254. ישראל שפירא:
    1)גם אם שדר מן החללית היה מגיע למאדים תוך דקה על פ שעון החללית. עדיין לפי שעון כדור הארץ היתה נמדדת שעה. לפיכך אין אפשרות לזרז את השדר על ידי שליחתו לחללית ומן החללית למאדים.
    2)במערכת שבה מתיחסים לחללית ולמאדים אין כל צורך להתיחס לכדור הארץ. חוסר הסימטריה בין הקרן המשוגרת מן החללית למאדים ובין הקרן החוזרת מן המאדים לחללית אינו תלוי כלל בקיומו של כדור הארץ.
    לצורך המחשה: נניח שלאחר שיגור הקרן מן החללית לכוון המאדים פתאום כדור הארץ מקבל תאוצה אדירה ומתרחק ממקומו במהירות העולה על חצי מהירות האור. זה לא ישפיע על מהירות הקרן הנשלחת מן החללית למאדים ועל מהירות הקרן החוזרת מן המאדים לחללית.
    3)אם נניח שבמקום המאדים תהיה שם חללית שנעה באותה מהירות ובאותו כיוון כמו החללית שחולפת על פני כדור הארץ בכל זאת נוכל לאפס את כל השעונים כך שהם יראו שעת אפס ברגע שחללית אחת נמצאת במרחק שעת אור מכדור הארץ והשניה נמצאת ממש בצמוד לכדור הארץ. כיצד נעשה זאת? נסנכרן מבעוד מועד את שעוני שתי החלליות ואחר כך כאשר אחת מהן תעבור ליד כדור הארץ נסנכרן את שעון כדור הארץ עם שעונה. התרגיל הזה זהה לתרגיל שבו מסנכרנים קודם את שעון כדור הארץ עם שעון המאדים ואחר כך כאשר החללית עוברת ליד כדור הארץ מסנכרנים את שעון כדור הארץ עם שעון החללית.

  255. יובל חייקין
    מה שאתה אומר ממש אינו נכון.דבר ראשון – אף צופה לא רואה אור מתקרב אליו. עבור החללית שמתקרבת למאדים – המרחק למאדים קצר יותר ולכן הזמן משידור האות לקליטה בחללית יהיה קצר יותר מהזמן משידור לקליטה על כדור הארץ. מצד הצופה על כדור הארץ, האור מגיע לחללית מוקדם יותר בגלל התקרבות החללית למאדים.

    נקודה שנייה – בו זמניות מוגדרת היטב עבור 2 ארועים המתרחשים באותו נקודה. ארועים אלה יראו בו0זמנית מכל מערכת תצפית.
    אין כאן שום פרדוקס!! אולי אתה מדבר על פרדוקס התאומים, אבל גם זה לא פרדוקס כי אנחנו יודעים להסביר אותה בפשטות. וזה בכלל לא הסבר מסובך. פרדוקס הוא דבר מאד מסויים – “פרדוקס התאומים” לא מתאים כלל להגדרה זו.

  256. צופה מהארץ רואה את החללית וגל האור מתקדמים זו לקראת זה במהירות שיא (אחת וחצי סי). אך הצופה בחללית רואה את גל האור מתקרב אליו במהירות סי בלבד, נאום איינשטיין. בתנאים כאלה סינכרון שעונים אינו אפשרי, וגם דוד ישראל טוב הלב לא יצליח לרצף את הנתיב האמור במדי-עת.
    הצופה מן הארץ רואה את האור פוגש בחללית אחרי 40 דקות (לאחר חישובי התאמה). נהג החללית רואה את האור רק מקץ שעה. הוה אומר שארבעים דקות ארציות שוות שישים דקות חלליתיות.
    מאידך, החלליתן צופה גם על הארץ והוא רואה את גל האור מתנגש בה אחרי שעה. ז”א הוא רואה את האור מתנגש בו ובארץ בוזמנית (“מה זו בוזמניות?” שאלה טובה). כלומר, בעיניו שישים דקות שלו הן שישים דקות ארציות. לזה כיוונו הגדולים כאשר אמרו “פרדוקס”. ואם פרדוקס, אז מתבקש פתרון. אך את זה יש לחפש לא בתוך המערכת בה הוא נוצר – לא מתוך החללית ולא מתוך הארץ, ובודאי שלא מתוך תורת היחסות.

  257. שניכם טועים.

    אילו הייתם צודקים, אז כשהחללית חולפת על פני הארץ, היה ניתן לשגר לה שדר שהיא בתורה הייתה משדרת למאדים. אם כפי שטענתם היה לוקח לשדר פחות מדקה להגיע למאדים, אז הצלחנו לשגר אינפורמציה למרחק של דקת אור בזמן של פחות מדקה, ולכן מהר מהאור. מה שלא אפשרי.

    ניסים – אכזוב אכזבתני אכזב. כשהתגאתי בך חשבתי שהתכוונת למשהו אחר:

    הנחת היסוד שלנו שאם חללית א’ שולחת פולס למאדים והפולס יגיע אליו תוך שעה (תשובתו של עופר), אז אם מאדים שולח פולס לחללית הוא יגיע אליה גם כן בתוך שעה בגלל הסימטריה, אינה בהכרח נכונה לפי היחסות הקלסית. המערכות אינן סימטריות. החללית חולפת במערכת כדה”א – מאדים הנמצאים במנוחה יחסית זה לזה, ושעוניהם מסונכרנים. החללית לעומת זאת אינה מסונכרנת איתם. אילו היתה חללית נוספת שטסה באותה מהירות וכיוון ומרחקה מהחללית המקורית שעת אור ושעוניה מסונכרנים איתה, אז היינו יכולים לדבר על סימטריה. הצרה היא שלפי היחסות, המרחק משתנה עם המהירות, ולכן לא נוכל לאמר שכאשר הזמן הוא 0 בשעוני שתי החלליות, הן בהכרח נמצאות מול הארץ ומאדים בהתאמה. לכן כאשר שולח מאדים פולס בזמן 0 לפי שעונו, אין זה זמן 0 במערכת החלליות והפולס לא יגיע לחללית בזמן של שעה עגולה.

    ועכשיו שאלת בונוס:

    על פי היחסות כל מערכת רואה את השניה כנעה לאט יותר. אז אם חלליתינו מגיעה למאדים, האם שעונה יפגר/ימהר/ישאראותודבר יחסית אליו?

  258. מובן.
    רציתי לחשוף את השגיאה של ישראל. הוא טוען שיש סימטריה בשעה שאין סימטריה.
    אין סימטריה כי צד אחד משדר וקולט את ההחזר בעוד הצד השני מקבל שדר ומחזיר אות.

  259. ב
    בוא נניח שכדור הארץ ומאדים אינם נעים והמרחק ביניהם דקת אור. נניח שהחללית משגרת אות בחליפת כדור הארץ. ממאדים – זה נראה במרחק דקת אור ולכן האות יגיע כעבור דקה. אבל – מהחללית – מאדים נראית יותר קרובה – ולכן, לפי שעון החללית האות יגיע יותר מהר.
    עכשיו זה מובן?

  260. היכן אבדה הסימטריה?
    לפי מערכת המדידה של החללית:
    החללית עומדת והמאדים נע.
    הקרן יצאה וחזרה. המרחק הלוך שווה למרחק חזור. וגם הזמן הלוך שווה לזמן חזור.כלומר הדרך הלוך והדרך חזור הן סימטריות.
    לפי מערכת המדידה של מאדים:
    המאדים עומד והחללית נעה.
    הדרך שהקרן עושה עד למאדים היא מנקודת יציאת הפולס עד למאדים. הדרך שהקרן עושה חזרה עד לספינה היא ארוכה יותר כי הספינה התרחקה בינתיים. אין סימטריה בין האורך הלוך והאורך חזור וגם לא בזמן הלוך ובזמן חזור.
    לפיכך:
    למרות שמבחינת התנועה ההדדית החללית והמאדים הם סימטריים. בכל זאת בגלל שהחללית היא שמשגרת את הקרן ומקבלת אותה בחזרה. אין סימטריה.
    הזמן הלוך לפי שעון המאדים שונה מהזמן חזור לפי שעון החללית !!!

  261. ישראל שפירא
    אם הבנתי את כוונתך – הצופה במאדים יראה את האותות יחד אחרי שעה – אז כן.
    סניאק (Sagnac) המציא לפני 100 שנה סוג של ג’ירו שמשמש למערכות ניווט אינרציאליות במטוסי קרב בשנים האחרונות. עד לפני 20 שנה הג’ירו-ים היו מכאנים, כמו סביבון של ילדים. היום זה בנוי מלייזר שמסתובב בין מראות בשני כיוונים. הפרש הדרכים שנוצר מסיבוב גורם להפרש פאזה, שנמדד לצורך חישובי מצב המטוס בשמיים.
    אז סניאק דווקא בחור חיובי 🙂

  262. ניסים.
    האמת היא שדי שכנעת אותי, אם כי אינני בטוח לגבי ה30 דקות בדיוק. אני גאה בך. ליתר ביטחון, אציג את השאלה לעופר.

    אבל אנחנו מסכימים שהאותות מהארץ והחללית למאדים מגיעים ביחד בתוך שעה, נכון?

    והסניאק הזה נשמע כמו מניאק לא קטן בכלל.

  263. יש עוד אפקט ב-GPS שנקרא תופעת סניאק והוא אכן נובע מהתנועה הסיבובית – אבל הוא קטן (פחות ממיקרו שניה) לעומת 2 האחרים.

  264. בודאי. מאדים נראית קרובה יותר ומהירות האור קבועה ולכן הזמן מתקצר.

    ולגבי GPS – ההשפעה של הסיבוב זניחה. ההשפעה הגדולה היא של שינוי בגרביטציה, ההשפעה השנייה היא של מהירות התנועה. יש השפעות נוספות אבל הן קטנות משמעותית.

  265. ואותן התמרות בדיוק, מאריכות את הזמן פי שתיים, כך שהכל מתקזז.

    האם אתה טוען שאם החללית נעה במהירות שאמרת, הסיגנל יגיע אליה בזמן 30 דקות לפי שעונה?

  266. ישראל שפירא
    בוא נניח שהחללית נעה במהירות כזו שהתמרת לורנץ מקווצת אורכים בחצי (אני מניח שהמהירות היא 0.8-0.9c). אז הזמן גם יתקצר בחצי.

  267. ישראל
    אני חושב שאנחנו לא מבינים אחד את השני.
    בוא נסתכל על הארוע שהחללית חולפת ליד כדור הארץ.

    ממאדים, גם כדור הארץ וגם החללית באותו מרחק – המרחק הסטטי לכדוד הארץ. במצב זה, אות שיוצא מכדור הארץ ואות שיוצא החללית יגיעו יחד למאדים.

    מנקודת הראיה של צופה על כדור הארץ וצופה בחללית, המצב שונה. ברגע החליפה – החללית תראה את מאדים במרחק קטן יותר מאשר צופה מכדור הארץ.

    האם אתה מסכים?

  268. ב
    אתה צודק לגבי שעון כדה”א – אבל הוא די רחוק. החללית יכולה לדעת את זמן פגיעת הפולס רק לפי השעון שלה.
    אבל אל פחד – דוד ישראל דאג לכל. לכל אורך מסלולה של החללית פזורים שעונים המסונכרנים עם הארץ ונייחים יחסית אליו. לכן בזמן פגיעת הפולס, אפשר להציץ החוצה ולראות את זמן כדה”א כפי שהוא מופיע בשעון חולף.
    זמן זה יהיה שונה לפי היחסות מזמן החללית.

    ולגבי סעיף ב’ – בסדר, 2 פוטונים. אחד לכדה”א והשני לחללית.

    ניסים.

    איפה חוסר הסימטריה? לגבי החללית היא זו שבמנוחה ומאדים הוא זה שבתנועה. לכן אם אמרנו (ואמרנו!) שהקרן מגיעה מהחללית למאדים בשעה לפי שעונו, אז זה גם הזמן שיקח לקרן ממאדים לחללית לפי שעונה.

    לא?

    ולגבי לוויני GPS – הכל נכון, אבל לא לשכוח שזו תנועה מואצת (סיבובית). לא שזה אומר משהו למי שלא מכיר את הנושא החבוט.

  269. ישראל שפירא
    המצב כלל אינו סימטרי!
    ממאדים – האות משודר מאותו מקום ומאותו זמן. קרי – זה אותו ארוע. כלומר ממאדים יראו את 2 האותות באותו זמן. זה נכון, דרך-אגב, לכל צורת העברת מידע. מידע אינו יכול לנוע יותר מהמהירות c, וגם האור נע במהירות ב.

    בכיוון השני – ברגע שהחללית חולפת ליד כדור הארץ – החללית רואה את מאדים יותר קרוב מאשר הצופה על כדור הארץ. לכן, האות ממאדים יגיע לחללית (בזמן החללית) בזמן קצר יותר מאשר שיגיע לכדור הארץ. בכל מקרה – זה הניבוי של תורת היחסות הפרטית, וגם מתאים לתצפיות.

    לגבי צופה על כדור הארץ, השעון בחללית רץ לאט יותר. את זה רואים בקלות בלוויני ה-GPS. עם נתעלם מכוח הגרביטציה השעונים שם רצים לאט בכ-7 מיקרושנייה כל יום. חוסר הכבידה גורם לזמן לרוץ מהר יותר בלווינים – והמיהור הוא כ-45 מיקרושנייה ביום. את ההפרש של 38 מיקרושנייה ליום מתקנים – אחרת נקבל סטייה של 10 ק”מ ביום!!!

  270. לא הבנתי את הקשר בין השאלה לבין עובדת היות הפוטון חלקיקי או גלי.
    בכל מקרה לא מדובר בפוטון אחד בלבד.
    אילו היה זה פוטון אחד בלבד הוא היה נבלע באחת ממערכות המדידה ולא נודע במערכת המדידה האחרת !
    ברור שעל מנת לבצע מדידות כאלו צריך להשתמש בקרן עם כמות גדולה של פוטונים אשר רק חלק מהם יבלעו במכשירי המדידה.

  271. אם החללית מתרחקת מן המאדים:
    לפי שעון כדור הארץ:
    הקרניים מגיעות למאדים בתוך שעה.
    הקרניים חוזרות מן המאדים לכדור הארץ בתוך שעה.
    עד כאן עברו שעתיים.
    בינתיים החללית התרחקה מכדור הארץ מרחק של שעת אור אחת.
    מכאן והלאה:
    ניתן להתיחס כאילו קרן אור יוצאת מכדור הארץ לכיוון חללית שנמצאת במרחק שעת אור אחת מכדור הארץ ומתרחקת מכדור הארץ במהירות השווה למחצית מהירות האור.
    לאחר שעתיים שהקרן תנוע בכיוון החללית היא תעבור מרחק של שתי שעות אור.
    בינתיים החללית תעבור מרחק של שעת אור אחת בנוסף למרחק הראשוני של שעת אור אחת והיא תהיה במרחק של שתי שעות אור מכדור הארץ.
    כלומר הקרן תפגוש בחללית לאחר שעתיים.
    אם נוסיף את הזמן הלוך ושוב למאדים שהוא שעתיים נקבל סך הכל ארבע שעות.
    כל זה לפי שעון כדור הארץ כמובן !

  272. ניסים.

    מובן – אבל לא מוסכם.

    למאדים יגיעו האותות מן הארץ והחללית באותו הרגע: שעה לפי שעון מאדים.
    המצב סימטרי לחלוטין לגבי אור שנע ממאדים כלפי הארץ והחללית. אם הוא יצא בזמן 0 לפי שעון מאדים המסונכרן עם שעון הארץ, הוא יגיע בזמן 1.00 גם לארץ (שעון ארץ), וגם לחללית (שעון חללית). וזאת למרות שהחללית התקרבה מרחק ניכר לארץ. לו ברגע הגעת הפולס לחללית היא הייתה חולפת על שעון נייח במערכת ארץ – מאדים ומסונכרן עימם, ומצלמה חדת רזולוציה הייתה מצלמת בו זמנית את שעון החללית והשעון המסונכרן, היה הצילום מראה: 1.00 בשעון החללית ו52.2 דקות בשעון המסונכרן.

    הסיבה – התארכות הזמנין, התקצרות האורכין, והתמתחות הזברבירין.

    או כך לפחות טוענת היחסות, אני מאמין.

    יובל – מה אתה מתפרץ, הדלת פתוחה לרווחה. שאלתי כדי לקבל אישור מוסמך למה שכבר ידעתי, אבל עורער עיי גורם נוסף. האישור כבר ניתן עיי עופר מגד, ראה תגובה קודמת. אתה יכול גם לדבר איתו ישירות בבלוג שלו.

    ומי מתיחס לפוטון באופן שגוי? פוטון היה ונשאר – גלקיק. חציו גל וחציו חלקיק. וברור שהוא נמצא בשני מקומות בו זמנית – השאלה היא מה ההגדרה של “בו זמנית”.

    תכליתיות – אני.

    חוץ מזה שהסמיילים שלך נעשים עצובים יותר ויותר באופן ליניארי.

  273. יובל חייקין.
    זה לא הסבר נכון לתופעה – כי לא זו התופעה.
    תקשיבו – צופה במאדים יראה את האיתות של חללית מהירה שחולפת ליד כדור-הארץ נייח באותו רגע. הסברתי למה.
    הצופים בחללית ובכדור הארץ יראו איתות שיוצא ממאדים בזמן שונה, למרות שהאיתות שודר ברגע החליפהשלהם. גם זה הסברתי למה.
    מה לא מובן?

  274. ובכל זאת, הקפצת לי פיוז: האופן בו אתה מתייחס לפוטון הוא שגוי. אתה רואה פוטון כחלקיק ומתעלם מן המופע שלו כגל. יואב ליטבק המשיל את הדואליות הזאת לגל בים אשר כל עוד הוא רחוק מן האדמה הוא גל אך בהתנפצו אל החוף הוא מבטא את אופיו החלקיקי. המשל הזה אינו מסביר את כל התופעות הידועות. אני סבור שפוטון (וגם אלקטרון) הוא חלקיק וגל בוזמנית בכל רגע נתון. יש לי גם מודל הממחיש כיצד זה מתאפשר. לכן התשובה לשאלתך “אז אם הקרן היא בעצם פוטון בודד – אז אותו פוטון נמצא בו זמנית בשני מקומות, לא?” היא “כן!” גורף ומוחלט: פוטון בודד, מעצם היותו גל, נמצא בוזמנית במספר רב של נקודות.

  275. ישראל! אתה מחפש תשובה מוסמכת לסוגייה מדעית באמצעות סקר בקרב שניים וחצי חובבים מזדמנים?! תמהני 😛
    🙂 אני שמח שסוף סוף נמצא לי מישהו המבין את ההגיון שבדברי, גם אם לא מן הסיבה הנכונה, אך אני עדיין איתן בכוונתי לא להשתתף יותר בדיונים עקרים 🙁

  276. ישראל שפירא
    אני חושב שבדיוק בגלל זה אתה טועה. בגלל המהירות היחסית, כשהחללית ליד כדור הארץ מאדים יראה (מהחללית) קרוב יותר, ולכן האור יגיע מוקדם יותר. המצב אינו סימטרי

  277. חברה, אין צורך לשכנע אותי. השאלה הייתה בשביל שאוכל לקבל תשובה מוסמכת בדיון שיש לי עם אדם נוסף.
    אבל שימו לב לשאלתו הלגיטימית: אם באותו רגע יצאו קרניים ממאדים לארץ והחללית, הם יגיעו אליהם באותו הזמן – למרות שהחללית כבר מרוחקת מאוד מהארץ.

    אז אם הקרן היא בעצם פוטון בודד – אז אותו פוטון נמצא בו זמנית בשני מקומות, לא?

    אם המחיר הוא לעבור לאקספלורר, נישאר כנראה כחולים.

  278. ישראל שפירא
    שוב – תחשוב שהחללית נוגעת בכדור-הארץ וברגע משוגר אות. המביט ממאדים חייב לראות את האותות יחד. אחרת – אפשר להגיע לסתירה. אני מקווה שברור למה יש סתירה….

  279. מבחינת הצופה על פני כדור הארץ:
    עד שקרן האור הגיע למאדים החללית בינתיים התקדמה מחצית הדרך למאדים. הזמן שעבר הוא שעה.
    עד שהקרן החוזרת מן המאדים פגע בחללית החללית התקדמה בינתיים שליש מן המחצית הנותרת של המרחק למאדים. הזמן שעבר הוא שליש השעה.
    סך הכל על פני כדור הארך הזמן שעבר עד לפגיעת הקרן בחללית הוא שעה ושליש. כלומר שעה ועשרים דקות.

  280. מבחינת החללית אין סימטריה בין הקרן היוצאת לקרן החוזרת .
    לפי שעון כדור הארץ ושעון המאדים : הקרן היוצאת מכדור הארך לכיוון המאדים וגם הקרן היוצאת מן החללית לכוון המאדים יגיעו באותו זמן למאדים.
    כמה זמן יקח לקניים להגיע למאדים: אם המאדים מרוחק שעת אור מנקודת יציאת הקרניים אז זה יקח להם שעה להגיע למאדים.
    הקרן החוזרת לכדור הארץ תחזור בתוך שעתיים. שעה הלוך ושעה חזור.
    לגבי הזמן שיקח לקרן להגיע חזרה לחללית: שעון החללית אינו מסונכרן עם שעון המאדים ושעון כדור הארץ !
    זאת משום שהחללית נמצאת בתנועה לגבי כדור הארץ וגם לגבי המאדים. לחישוב הזמן המדויק צריך להשתמש בטרנספורמצית לורנץ.

  281. ניסים

    למה ברור? מנקודת מבטה של החללית היא במנוחה ומאדים מתקרב אליה, אז אם ישוגר אות ממאדים לחללית, מטעמי סימטריה יקח לו אותו הזמן להגיע לחללית כמו מהחללית אליו. אבל בינתיים החללית התקדמה וכדה”א לא.

    עזוב שטויות עכשיו, יחסות שמחסות דמי חסות. איך אתה מצליח לשגר תגובות בלי לינק? למה כולנו כחולים ואתה שחור?

  282. ישראל שפירא
    הרשה לי לנסח את השאלה שלך בצורה אחרת. (הניסוח עצמו הוא של מריו ליוויו.)
    אתה עומד בסוף מסלול נחיתה. מטוס בא לנחיתה מולך ובאותו זמו רכב חוצה את המסלול. נניח ששני הכלים מפעילים הבזק אור ברגע ההתנגשות. האם נראה את שני ההבזקים ביחד?
    ברור שכן – ההבזקים הופעלו במאורע ההתנגשות – מאורע זה הוא נקודה אחת בחלל-זמן. לכן – כל צופה, ללא תלות במהירותו או במיקומו יראו את 2 ההבזקים יחד.
    QED

  283. אני רואה את ההגיון שבדבריך – אך לא מהסיבה לה ציפית.

    עיקרון מאך – הקשר בין מצב מנוחה מוחלט, או סיבוב מוחלט, וכוכבי השבת – תופס רק למערכת המנוחה של כוכבים אילו בגלקסיה שלנו, שביל החלב. יחסית לגלקסיות אחרות הסיבוב המוחלט יהיה כנראה שונה (כך הבנתי מצבי).

    אז אם יש גורם נסתר שקובע על איזו צלחת מסתובבת יפעל כוח צנטריפוגלי ועל איזה לא, והוא שונה בגודלו מגלקסיה לגלקסיה, אז יש להניח שבתווך שבין הגלקסיות פועלים חוקים שונים מאשר בתוך הגלקסיות. אחרת איך ידע הכוח הצנטריפוגלי לפי איזו גלקסיה עליו להתיישר?

    ואם חוקי הטבע משנים את קבועיהן ביחס לגורם אחד (כוח הצנטריפוגלי), מה ימנע מהם לשנות את גודלו של קבוע אחר? (מהירות האור בדוגמה שלך).

  284. בהתנצלות על זה שאני ממחזר תגובה עתיקה:
    הבעיה עם הדיון על התארכות הזמנים (או התכווצות לורנץ או כל השמות והמינוחים הקשורים) היא שהניסוי עליו הוא מתבסס מבוצע במיקרו ואין לייחס אותו כמובן מאליו למאקרו. צריך להבין את המכניזם הגורם לתופעה הנצפית במרחקים הקצרים ולבדוק אם נכון להשליך אותו על מרחקים אסטרונומיים.
    הפרדוקס לא מסתיים בשעונים (או התאומים) אלא ממשיך בתופעות אחרות אשר ניתוחן תחת ההנחה שמהירות האור קבועה בכל מערכת ייחוס מוליך לתעלומות גדולות יותר. הדוגמה הבולטת מן העשור האחרון היא התופעה שזכתה לשם “אנרגיה אפלה”. אם נקבל שמהירות האור קבועה רק במרחב מוגבל, למשל בתחום מערכת השמש או אפילו שביל החלב, אך במקומות אחרים היא גבוהה יותר, האנרגיה הזאת כבר לא תהיה אפלה. אמנם איננו יכולים למדוד ישירות את מהירות האור במרחב הבין-גלקטי, אך תופעה אחרת, הגורמת לגלקסיות להתנהג כעדשות (“החומר האפל”, “עדשות כבידה”) יש בה כדי להצביע בסבירות גדולה על כך שמהירות האור בתוך הגלקסיות נמוכה מזו שמחוץ להן.
    לכן אני מציע שבטרם נבנה מגדלים על יסוד ההנחה הזאת ננסה, ראשית, להתמקד בניסוי המקורי ונחפש הנחות אפשריות נוספות – כאלה שיתאימו גם לתופעות החדשות שנצפו לאחרונה.

  285. אנא ערף, כתבת שעת אור והשאלה שלי היתה על זמנים.

    אבל אל פחד. בינתיים רב שועל עופר מגד חרץ את דינה של שאלתי – לחסד. שעה עגולה לשתי האותות (או שני הפולסים, תלוי במכפלת המינים). ראה

    http://ofer-megged-phys-notes.blogspot.com/2011/04/blog-post.html

    שמת לב לכך שהרבה פיזיקאים הם צבאים ועופרים?

    ומה הקשר ל מ-מ? מ-מ בא לגלות מהי מערכת המנוחה של האתר. השאלה שלנו עוסקת יותר בהאם מהירות האור מצטרפת למהירות מקור האור, מה שהחברה ברפת מכנים: EMISSION THEORY.

    כי שים לב: אם האור מגיע מהחללית למאדים בתוך שעה, אז מטעמי סימטריה גם אור ממאדים לחללית מגיע בשעה. וגם לכדה”א בשעה. אבל החללית בינתיים התקרבה כברת דרך למאדים, אז איך זה שהאור ממאדים מגיע גם לארץ וגם לחללית באותו זמן?

    על כך בפרק: התארכות הזמנים.

  286. שעת אור זה מרחק. מישהו אמר אחרת?
    אם השרשור של שאלותיך החל במקום אחר, אנא שלח קישור. על פניו נראה שהתרחיש בשאלותיך מחקה בקירוב את ניסוי מ”מ.

  287. חיים מצטער, הקללות היחידות שאני מכיר הן בערבית. יש בכלל דבר כזה קללות בעברית? או אוכל ישראלי? זה לא קצת בושה שכל מה שיש בו קצת טעם לקחנו מההורים או השכנים?

    חוץ מזה שבקיבוץ גדלנו באווירה של סלידה מכל מה שהוא גרמני, והערצה לכל מה שהוא רוסי.

    היטלר קאפוט! סטלין גוט!

    אלי, עשה כמוני, הוסף את הלינק בכתובת למעלה. זה מרגיע את מנגנון התגובות שקצת מתפרע לאחרונה.

    שלמה, השאלה הייתה על זמן. שעת אור זה מרחק.

    ואולי יכול מישהו לנסות לענות ברצינות על השאלות שלי? זה חשוב ביותר לקיום תקין של היקום.

  288. תודה ספקני, נשמע מעניין.

    דרך אגב אצלי (אינטרנט אקפלורר 10, חלונות 7) כשאני מנסה לשלוח תגובה בלי למלא את שדה הכתובת (עם אייקון הבית לידו) אז השדה מודגש לי מיד באדום והסמן קופץ לשם, וההודעה לא נשלחת עד שאני ממלא שם איזו כתובת פיקטיבית.

  289. אלי

    אתה לא צריך למלא את כל השדות בטופס התגובה. רצוי (או חובה) למלא רק את השדה של הניק (שם מזהה שלך באתר).

    לגבי העדר הסבר על קיצר הטיסה. כנראה שאין הסבר גם במאמר ממנו הופק המאמר כאן, או שההסבר שם מסובך מדי. חיים מזר כאן העיר איזו שהיא הערה בעניין ורצוי שיתן הסבר מפורט מדי.

    אני משער מדבריו של מזר שיש לסוייוז כמות מוגבלת של דלק לצורך השיוט של החללית, כתוצאה מכך יש ירידה בכושר החללית לבצע תמרונים בתנועה שלה. אני מנחש (שוב על פי דבריו של מזר) שבמסלול הקודם, שהיה בצורה של ספירלה של סיבובים רבים יותר סביב כדור הארץ (סיבובים תוך הגדלת המרחק מכדור הארץ) — החללית סויוז מתקרבת במסלול “כמעט משיקי” למסלול של תחנת החלל. זאת בהשוואה למסלול החדש שגם הוא ספירלי אבל עם פחות סיבובי ספירלה. כתוצאה מהקטנת מספר סיבובי הספירלה ההתקרבות של הסויוז לתחנת החלל היום היא פחות משיקית למסלול תחנת החלל. עקב הקטנת המשיקיות בין שני המסלולים הנ”ל (במסלול החדש) צריכה הסויוז לבצע תפנית חדה יותר כדי להכנס למסלולים זהים בזמן המיפגש. תפנית חדה יותר דורשת יותר דלק (וכאמור הסויוז מוגבלת בכמות הדלק לשיוט ולתמרון). קשה לי להסביר יותר למה כוונתי בלי שרטוט וגם אין לי זמן לשרטט. מקווה שאיני טועה ומטעה, בכל מקרה כדאי שחיים מזר יתן הסבר מדוייק (אם יש לו).

  290. ומדוע בזמן כתיבת תגובה מחייבים אותי למלא את השדה של כתובת אינטרנט ? מה בדיוק אני אמור לכתוב שם ?

  291. אבי מדוע אין שום הסבר לגבי מה איפשר את קיצור הזמן ? כשקראתי את הידיעה בווינט הכעיס אותי שאין שום פירוט אבל ידעתי שאין יותר מדי מה לצפות מהם, אבל באתר הידען ? לא הגיוני לפרט קצת מה מיוחד כל כך במסלול החדש ומדוע לא השתמשו בו עד היום ? בידיעה בווינט הוזכר גם כי המסלול החדש דורש תמרונים מאד מדוייקים (יחסית למסלול הקודם) מדוע ?

    הייתי מצפה לאיזה הסבר מינימאלי.

  292. אױ װעי! בתור זמיר אתה לא רציני!
    שעת אור בכיוון הלוך ואותו מרחק בכיװן חזור, כי אילצת את הפלנטות להתנײח זל”ז. אתה לא יודע לעשות אחד ועוד אחת?
    ואגב כך (ולרגל היום הקדוש הזה), שאלה מגדרית: כשמכפילים זכר בנקבה מה מין המכפלה? למשל, אחד כפול אחת זה שניים או שתיים?

  293. לישראל שפירא אהבתי את התוספת שלך. בדיחה כזו יכול הבין רק מי שיודע יידיש ואני מניח שאתה שולט ביידיש על בורייה. כנס לאתר יקום תרבות ושם ברשימת הכותבים מצד שמאל כנס לשם שלי, תתחיל לדפדף עד שתגיע לנושא שהוא קללות ביידיש כולל הלינקים. כל הקללות שמופיעות שם הן פרי המצאתי.

    לגופו של עניין מה שקרה הוא שבמהלך השנים שהמיר פעלה חלו בה ויותר ויותר תקלות עד שלבסוף הוציאו אותה ממסלולה הכניסו לאטמוספירה ומשם לאוקיינוס השקט . הסיבה לכך שהרוסים החזיקו את המיר זמן כה רב בחלל באה מתוך מטרה ללמוד איך מתמודדים עם תקלות בתנאי חלל. יש להניח שכאשר תשוגר חללית מאוישת למאדים, בטכנולוגיה שקיימת היום טיסה כזאת למאדים תימשך מספר חודשים, שהייה של עד חודש וחצי על הקרקע וחזרה ארצה שוב מספר חודשים. טיסה שבין שנה לשנה וחצי. סביר שתהיינה תקלות ויהיה צורך לתקן אותן. חללית הנמצאת במרחק עשרות מיליוני קילומטרים מכדור הארץ לא תעשה סיבוב פרסה עד שתגיע למוסך ששמו כדור הארץ. יהיה צורך לבצע תיקונים אלה במהלך הטיסה. המיר עליה השלום(דרך אגב לא נאמר עליה לא קדיש ולא קדיש יתום לאחר שנכנסה למימיו של האוקיינוס השקט). בשנותיה האחרונות היתה סוג של בית ספר לצורך העניין.

  294. שלמה, מספיק להתחכם. השאלה ברורה: מה יראו שעוני מאדים כאשר יגיעו אליו השדרים מהארץ ומהחללית?

    חיים, שמעתי שאחרי הפשלות של מיר, שינו את שמה ל וייז מיר.

  295. במסגרת פעילותה של תחנת החלל האמריקאית skylab שוגרו אליה 3 חלליות אפולו. זמן הטיסה מהשיגור עד לעגינה בתחנת החלל היה בין 7 ל-8 שעות. מה שאפשר זאת היו טילי השיגור של ארה:ב שעדיין אין כמותם ברוסיה. לשם השוואה טילי השיגור סטורן 5 ששיגרו את חלליות אפולו לירח יכלו להכניס למסלול סביב כדור הארץ מטענים במשקל של 120 טון. חללית הסויוז שוקלת כ -7 טון. עד היום לא שוגרה אף חללית סויוז עם מלוא כמות הדלק במיכלי הדלק. טילי השיגור סויוז לא יכולים לשאת את חלליות הסויוז במלוא משקלם הפוטנציאלי. כדי לפתור בעיה זו חלליות הסויוז היו מוכנסות למסלול סביב כדור הארץ מיד לאחר שיגורן. במשך יומיים היתה בכל טיסה מבוצעת סדרה של תיקוני מסלול קטנים עד שהיו מגיעים לתחנות החלל מיר, סוליוט ולתחנת החלל הנוכחית.הרוסים נקטו בדרך של הגדלת מסלול הטיסה בדרך תוספית. כל פעם מגביהים במקצת את גובה הטיסה.

    הפעם הרוסים נקטו בדרך אחרת הם למדו את חלון הזמן בו תחנת החלל חולפת מעל רוסיה ואז משגרים את חללית.הסויוז. את חלליות האפולו אפשר היה לשגר בכל זמן שהוא. החלליות היו טסות ישירות לתחנת החלל. הרוסים אמנם חוסכים בדרך החדשה דלק, חמצן ומזון אך מגבילים את עצמם לחלונות זמן מסוימים.

  296. 1) התווך הוא אותו תווך, הן לשדר מהחללית והן לשדר מהארץ. מה השאלה?
    2) גם כאן זה אותו תווך, רק בכיוון ההפוך.
    בניו ג’רסי השעה כעת 1:35 בבוקרו של האחד באפריל. איך בקליפורניה?

  297. צבי, אהוד, עדי, עופר, ויתר שועלי היחסות הותיקים.

    יש לנו פה ויכוח עקרוני ביחסות פרטית שניתן להציגו באמצעות השאלות הבאות:

    1. אם חללית שמהירותה חצי c חולפת ליד כדה״א בזמן 0 לפי שעוני החללית והארץ, וברגע המפגש ביניהם שולחת ביחד עם הארץ שדר לקיורויוסיטי במאדים (שלצורך ההדגמה נמצא במרחק שעת אור מהארץ ונייח יחסית לארץ), מה יראה השעון בקיורויוסיטי (המסונכרן עם שעון הארץ) ברגע הגעת השדרים? האם הם יגיעו ביחד?

    2. אם במאדים יש מראה (כמו זו על הירח) והחללית והארץ שולחים קרן לייזר ברגע המפגש ביניהם, מה תהיה השעה בארץ כאשר יחזרו שני הפולסים ממסעם הלוך ושוב בין שתי הפלנטות?

    התוכלו לתת את הדעת לשאלה?

    תודה.

  298. אפשר עוד פרטים על מה איפשר את החיתוך בזמנים? ואולי הסבר למה זה בעצם לא ניסו את זה עד היום?

כתיבת תגובה

האימייל לא יוצג באתר. שדות החובה מסומנים *

אתר זה עושה שימוש באקיזמט למניעת הודעות זבל. לחצו כאן כדי ללמוד איך נתוני התגובה שלכם מעובדים.